SlideShare uma empresa Scribd logo
1 de 80
Baixar para ler offline
4
ExtensivoMEGA 4
4
L
i
v
r
o
Curitiba2019
LivrodoPROFESSOR|Matemática
PG19LP424SDM0_MIOLO_PVE19_4_MAT_LP.indb 1 27/05/2019 16:45:42
Todos os direitos reservados.
SAE DIGITAL S/A.
R. João Domachoski, 5. CEP: 81200-150
Mossunguê – Curitiba – PR
0800 725 9797 | Site: sae.digital
Produção
© 2019 – SAE DIGITAL S/A. É proibida a reprodução, mesmo parcial, por qualquer processo, sem
autorização por escrito dos autores e do detentor dos direitos autorais.
FICHA CATALOGRÁFICA
S132
SAE, Extensivo Mega: matemática, pré-vestibular: livro 4 : livro do professor /
SAE DIGITAL S/A. - 1. ed. - Curitiba, PR : SAE DIGITAL S/A, 2019.
80 p. : il. ; 28 cm.
ISBN 978-85-535-0824-2
1. Matemática - Estudo e ensino (Ensino Médio). 2. Matemática (Ensino Médio)
- Problemas, questões, exercícios. 3. Universidades e faculdades - Vestibular. I. Título:
Matemática, Pré-vestibular: Livro 4 : Livro do Professor. III. Título
CDD: 372.7
CDU: 373.3.016:510
Gerência editorial Luiz Henrique Pereira Mainardes
Coordenação editorial Tassiane Aparecida Sauerbier
Edição Anna Paula Chiarello Marcon, Eliane Peixoto de Lima, Janayna da Costa Goulart, Janile Oliveira,
Rodrigo Zeni Stocco
Revisão Ana Paula Gurki Ferraz, Brunno Freire da Silva Neto, Camille Chiquetti, GabrieleVarão da Costa, Juliana
Basichetti Martins, Katieli Silva, MarcelaVidal Machado, MarileneWojslaw Pereira Dias, Pãmela Leal,
Priscila de Jesus Sousa,Thainara Gabardo,Victor Augusto de LimaTruccolo
Cotejo AndreiaVidal, Laura Akemi, Mariana Chaves, Mariana Passarin, Mellanie Novais, Polyana Fonseca,
Sthefanie Lhorente
Coordenação processos Erica Fujihara
Processos Cleyton Dall’Agnol, Janio Junior, Raul Jungles
Coordenação qualidade Vanessa Marques Cabral dos Santos
Qualidade Bruna Ferreira Rodrigues, Everson de Lara Caetano, Henrique Sossélla, MarileneWojslaw Pereira Dias
Coordenação produção visual Mauricio Ragadalli
Iconografia Antonio Sevilha
Cartografia Júlio Manoel França da Silva
Ilustrações Deny Machado, Kássio Luiz Dias Nery
Arte da capa Kássio Luiz Dias Nery
Projeto gráfico Evandro Pissaia, Rafael Chueire
Diagramação André Lima, Evandro Pissaia, Bruno M. H. Gogolla, Gustavo RibeiroVieira, Jéssica Suelen de Morais,
Jéssica Xavier, Juliana Hiromi Saito, Kássio Luiz Dias Nery, Luisa Piechnik Souza, Maísa Leepkaln,
Mariana Oliveira, Mikhael Gusso, Nadiny da Silva, Ralph Glauber, Silvia Santos,Tarliny da Silva,Thiago
FigueiredoVenâncio
Autores Carolina de Almeida Santos Pinotti, Ednei Leite de Araújo, Maria Fernanda Martini Campagnaro
Créditos da capa Vadym Pasichnyk/Valery Brozhinsky/Shutterstock |Turing Archive/W. Commons
PI_PVE19_L4_MAT_PROF.indd 2 28/05/2019 11:16:54
Livro 7
Revisão 4
Frente A
Frente B
Frente C
Frente A
Frente B
Frente C
Índice
PG19LP424SDM0_MIOLO_PVE19_4_MAT_LP.indb 3 27/05/2019 16:45:43
CONHEÇA OS RECURSOS
DIGITAIS
DIGITAIS
DIGITAIS
PLATAFORMA ADAPTATIVA
ENTENDEU A AULA DE HOJE?
A plataforma adaptativa permite que você resolva exercícios encaminhados
pelo professor e revise o conteúdo visto na semana. Veja como é fácil:
COMO VOCÊ E SUA FAMÍLIA ACOMPANHAM SEU DESEMPENHO?
Quando alguma atividade da plataforma adaptativa for agendada, aluno e responsável serão comunicados
por meio de um aplicativo. Além de enviar um lembrete da tarefa ao aluno, essa ferramenta permite ao
responsável o acompanhamento, em tempo real, do desempenho do estudante e o momento em que o
exercício é concluído.
As notificações serão enviadas quando:
• uma atividade for encaminhada;
• restarem 24 horas para o encerra-
mento do período para a atividade;
• os exercícios forem resolvidos;
• a tarefa não foi realizada.
DIGITAIS
DIGITAIS
DIGITAIS
Selecione a disciplina disponível e resolva
os 4 exercícios propostos na plataforma.
Acesse o site ava.sae.digital
e insira seu login e senha.
Caso seu desempenho esteja abaixo
do esperado, você deverá assistir a
uma videoaula sobre o conteúdo.
Após assistir à videoaula, 3 questões
deverão ser respondidas para finalizar
a tarefa e verificar seu aprendizado.
1
2
3
4
• Acesse a Play Store (Android) ou a App
Store (IOS) de seu smartphone e faça o
download do aplicativo SAE NOTIFICA.
• No aplicativo, insira seu login e senha
(aluno e responsável, conforme cadas-
tro da escola).
Acesse a Play Store (Android) ou a App
e faça o
SAE NOTIFICA.
No aplicativo, insira seu login e senha
(aluno e responsável, conforme cadas-
Dmi
T;
Omelchenko;
photka;
Gts;
GCapture/Shutterstock
PG19LP424SDM0_MIOLO_PVE19_4_MAT_LP.indb 4 27/05/2019 16:45:50
SAE QUESTÕES
ARRASE NO ENEM
Em consonância com o Ano Internacional da Tabela
Periódica dos Elementos Químicos, declarado
pela ONU, apresentamos como Objeto Digital um
simulador da Tabela Periódica, que possibilita ao
aluno e ao professor interagirem de forma dinâmica
e interativa com os 118 elementos químicos.
O QUE O ENEM ESTÁ AVALIANDO?
Com o aplicativo SAE QUESTÕES você poderá aprimorar seus estudos solucionando as questões
presentes nas edições da avaliação do ENEM.
TEMAS DA ATUALIDADE
Acessandooitem“Atualidades”doaplicativo,épossívelacompanharocanal“DOBRADINHASAE”,quetraz,
semanalmente, um professor de redação juntamente com um professor de outra disciplina debatendo
temáticas da atualidade e dando dicas e sugestões de como escrever e estruturar uma boa redação.
QRCODE NAS QUESTÕES ENEM
As questões do ENEM possuem QR codes. Depois de resolver essas questões, confira o vídeo com a resolução
comentada. Para isso, acesse o item “Leitor QRcode” do aplicativo e posicione a câmera de seu smartphone em frente ao
código da questão. Para ter acesso a esse recurso, basta:
1. Baixar o aplicativo SAE QUESTÕES, disponível na Play Store (Android) e também
na App Store (IOS).
2. Fazer um cadastro informando um e-mail válido e elaborar uma senha de acesso.
SIMULADOR – TABELA PERIÓDICA
Quer acessar centenas de aulas para arrasar no ENEM?
Acesse a plataforma adaptativa com seu login e senha e clique em ARRASE NO ENEM. Você vai
encontrar inúmeras videoaulas com conteúdo completo para o ENEM.
E não acaba aí!
Você poderá contar com roteiros de estudos em formato PDF presentes em cada videoaula disponível.
Também é possível acessar as aulas pelos QRcodes presentes nas aberturas dos módulos.
SIMULADOR – TABELA PERIÓDICA
PG19LP424SDM0_MIOLO_PVE19_4_MAT_LP.indb 5 27/05/2019 16:45:53
7
Extensivo
MEGA
Livro
Sumário
VICHAILAO/Shutterstock
VICHAILAO/Shutterstock
Matemática
Frente A
Polinômios II 165
Equações algébricas 169
Equações algébricas: relações de Girard 173
Equações algébricas: transformações e
equações recíprocas 177
Frente B
Operações com arcos II 181
Equações e inequações trigonométricas 185
Juros simples 189
Juros compostos 194
Frente C
Distância entre ponto e
reta e área de um triângulo 199
Equações da circunferência 204
Circunferência: posições relativas 209
Cônicas 215
PG19LP424SDM0_MIOLO_PVE19_4_MAT_LP.indb 164 27/05/2019 16:46:14
Frente A | Livro
Ziga Cetrtic/Shutterstock
Frente A
165
PVE19_7_MAT_A_25
7
VERSÃO 3.3
Método da chave
Dados dois polinômios P(x) e D(x), de graus p e q respec-
tivamente, ao dividirmos P(x) por D(x) encontramos dois poli-
nômios Q(x) e R(x), denominados quociente e resto respecti-
vamente, que satisfazem à:
P(x) = D(x) ⋅ Q(x) + R(x)
Em que:
● o grau de R(x) deve ser menor que o grau de D(x);
● ou R(x) = 0.
Então,
● se gr(P) < gr(D), então Q(x) = 0 e R(x) = P(x).
● se gr(P) ≥ gr(D), a divisão pode ser efetuada pelo seguin-
te algoritmo denominado método da chave:
I. ordenam-se P(x) e D(x) segundo as potências decres-
centes de x, inclusive com os termos do dividendo que
possuem coeficiente 0;
II. divide-se o primeiro termo de P(x) pelo primeiro termo
de D(x), obtendo-se o primeiro termo do quociente;
III. multiplica-se D(x) pelo primeiro termo do quociente e
subtrai-se o resultado de P(x), obtendo-se o primeiro
resto parcial;
IV. com o primeiro resto parcial e o divisor D(x) repetem-se
as operações, obtendo-se o segundo termo do quocien-
te, e assim sucessivamente, até encontrar um resto de
grau menor que o divisor.
9 Exemplo:
Calcule a divisão de (x3
+ 2x -1) por (x2
+ x + 2)
Solução:
x3
+ 0x2
+ 2x − 1 x2
+ x + 2
−x3
− x2
− 2x x − 1
-x2
+ 0x − 1
x2
+ x + 2
x + 1
Dessa forma, Q(x) = x − 1 e R(x) = x + 1.
Observação
O grau do quociente é a diferença dos graus do dividendo e
do divisor.
gr(Q) = gr(P) - gr(D)
No exemplo acima o quociente tem grau 3 - 2 = 1.
Método de Descartes
A divisão de polinômios também pode ser efetuada pelo
método de Descartes, que é uma aplicação da identidade de
polinômios. Nesse método, parte-se da expressão
P(x) = D(x) ⋅ Q(x) + R(x), em que
gr(Q) = gr(P) - gr(D) e gr(R)MAX = gr(D) - 1.
O quociente e o resto são obtidos igualando-se os coefi-
cientes dos dois lados.
9 Exemplos:
1) Divida P(x) = x4
+ 2x3
+ 3x2
+ 4x + 5 por D(x) = x3
+ 1, a fim de
encontrar Q(x) e R(x).
Solução:
Supondo Q(x) = ax + b e R(x) = cx2
+ dx + e, temos:
P(x) = Q(x)·D(x) + R(x)
x4
+ 2x3
+ 3x2
+ 4x + 5 = (ax + b)·(x3
+ 1) + (cx2
+ dx + e) ⇒
⇒ x4
+ 2x3
+ 3x2
+ 4x + 5 = ax4
+ bx3
+ cx2
+ (a + d)x + (b + e)
a = 1
b = 2
c = 3
a + d = 4 ⇒ d = 3
b + e = 5 ⇒ e = 3
⇒ Q(x) = x + 2 e R(x) = 3x2
+ 3x + 3
2) Determine os valores de p e q de modo que x3
− 6x2
+ px − 1 seja
divisível por x2
+ 3x − q.
Solução:
Devemos fazer o resto R(x) = 0 e adotar um quociente
Q(x) = ax + b do primeiro grau. Assim,
x3
− 6x2
+ px − 1 = (x2
+ 3x − q) · (ax + b) ⇒
⇒ x3
− 6x2
+ px − 1 = ax3
+ (b + 3a)x2
+ (3b − aq)x −bq
Igualando os coeficientes dos termos de mesmo grau, temos:
● a = 1
● b + 3a = - 6 ⇒ b + 3 · 1 = - 6
b + 3 = - 6 ⇒ b = - 6 - 3
b = - 9
● 3b - aq = p ⇒ 3 · (- 9) - 1 · q = p
- 27 - q = p ⇒ p + q = - 27
● - bq = - 1 - (- 9) q = - 1
9q = - 1 ⇒ q = -
1
9
● p + q = - 27
p = - 27 - −






1
9
= −
242
9
Logo, p = −
242
9
e q = −
1
9
Método da chave • Método de Descartes •
Dispositivo de Briot-Ruffini
Métodos da chave,
Dispositivo de Briot-Ruffini
e Teorema de d’Alembert
Polinômios II
PG19LP424SDM0_MIOLO_PVE19_4_MAT_LP.indb 165 27/05/2019 16:46:27
Matemática
166
PVE19_7_MAT_A_25
Um polinômio identicamente nulo é aquele que é nulo
para qualquer valor da variável e tem todos os seus coeficien-
tes iguais a zero.
Importante
Se um polinômio de grau n possuir mais de n raízes, ele é
identicamente nulo.
Teorema do Resto (ou de d’Alembert)
O resto da divisão de um polinômio P(x) por ax + b,
a 0, i ua a P
−






b
a
.
Demonstração
Na divisão de P(x) por ax + b o resto deve ter grau zero.
Assim, podemos dizer que a divisão terá um quociente Q(x) e
resto R(x) (R = constante). Logo,
P(x) = (ax + b) ⋅ Q(x) + R(x) ⇔ P(x) = (ax + b) ⋅ Q(x) + R
Fazendo x = -
b
a
,
P
b
a
a
b
a
b Q
b
a
R R P
b
a
−





 = −





 +





 −





 + ⇔ = −






⋅
9 Exemplo:
Calcule o resto de P(x) = x3
+ x2
+ x + 1 por x + 1.
Solução:
Em x + 1 temos a = 1 e b = 1. Então −
b
a
= − = −
1
1
1.
O resto será P(−1) = (−1)3
+ (−1)2
+ (−1) + 1 = 0.
Logo, 0 é o resto de P(x).
Teorema: o polinômio P(x) é divisível por ax + b, com
a 0 se, e somente se, P
−






b
a
= 0.
9 Exemplo:
Determine m para que o polinômio P(x) = x3
+ 2x2
+ mx -10 seja
divisível por x - 2.
Solução:
Em x - 2 temos a = 1 e b = - 2. Então: − = −
−
( ) =
b
a
2
1
2
P(2) = 23
+ 2 ⋅ 22
+ m ⋅ 2 - 10 = 0 ⇒ m = -3
Dispositivo de Briot-Ruffini
Podemos usar o dispositivo de Briot-Ruffini para dividir
um polinômio de grau maior ou igual a 1 por um binômio do
tipo x - a. Esse dispositivo consiste em realizar a divisão efe-
tuando cálculos com os coeficientes.
b er e di idir    2x3
- 5x2
+ 3  - 4 r      - 2
por meio do dispositivo de Briot-Ruffini.
● Posicionamos a raiz do divisor e os coeficientes do divi-
dendo observando a ordem decrescente dos expoentes
de x do polinômio completo:
Raiz do
divisor
Coeficientes do dividendo
2
2
−5 3 −4
● O coeficiente do primeiro termo do quociente é igual ao
coeficiente do primeiro termo do dividendo:
2
=
2
2
−5 3 −4
● Multiplicamos a raiz do divisor pelo coeficiente que foi
repetido e adicionamos o produto encontrado ao se-
gundo coeficiente do dividendo:
2
x
+
2
2
−5
−1
2 · 2 + (−5)
3 −4
● Agora, multiplicamos a raiz do divisor pelo número es-
crito abaixo do segundo coeficiente e adicionamos o
resultado encontrado com o terceiro coeficiente, escre-
vendo o número encontrado abaixo e assim sucessiva-
mente:
2
2
x
+
2
−5
−1
2 · (−1) + 3
3
1
−4
● O último número encontrado é o resto da divisão:
2
2
+
2
−5
−1
2 · 1 + (−4)
3
1
−4
−2
x
Temos, então: Q(x) = 2x2
- x + 1 e R(x) = -2.
9 Exemplo:
Determine a e b para que o polinômio x3
− ax2
+ bx − 10 seja
divisível por (x + 2)(x − 1).
Solução:
Aqui, aplicaremos o dispositivo duas vezes:
1 −a b −10
−2 1 −2 −a 4 + 2a + b −18 − 4a − 2b = 0
1 1 −1 −a a + b + 3 = 0
4 2 18
3
4 2 18
3
6 3
a b
a b
a b
a b
a b
+ = −
+ = −



⇒
+ = −
= − −



⇒ = − =
e .
PG19LP424SDM0_MIOLO_PVE19_4_MAT_LP.indb 166 27/05/2019 16:46:43
Frente A | Livro 167
7
PVE19_7_MAT_A_25
Resolvidos
1. (ESPM) O resto da divisão do polinômio x x
5 2
3 1
− + pelo polinômio
x2
1
− é
a) x - 1.
b) x + 2.
c) 2x - 1.
d) x + 1.
e) x - 2.
9 Solução: E
Dividindo x x
5 2
3 1
− + por x2
1
− , obtemos
x x x
x x x x
x x
x x
x x
x
x
5 2 2
5 3 3
3 2
3
2
2
3 1 1
3
3 1
3 1
3 3
2
− + −
− + + −
− +
− +
− + +
−
−
Portanto, o resto é x − 2.
2. (UEG) A divisão do polinômio x x x
3 2
2 5 6
+ − − por x x
+
( )( )
1 2
−
é igual a
a) x - 3.
b) x + 3.
c) x - 6.
d) x + 6.
9 Solução: B
Aplicando o dispositivo prático de Briot-Ruffini obtemos:
1 2 5 6
1 1 1 6 0
2 1 3 0
- -
- -
Logo, x x x x x x
3 2
2 5 6 1 2 3
+ − − = + − +
( )( )( ) e, portanto, a divisão do
polinômio x x x
3 2
2 5 6
+ − − por ( )( )
x x
+ −
1 2 é igual a x +3.
3. (Ibmec) Se o resto da divisão do polinômio P(x) = x3
+ ax + b
pelo polinômio Q(x) = x2
+ x + 2 é igual a 4, então podemos
afirmar que a + b vale
a) 2.
b) -2.
c) 3.
d) -3.
e) 4.
9 Solução: C
a – 1 = 0. Logo, a = 1.
b + 2 = 4. Logo, b = 2.
x x ax b x x
x x x x
x a x b
x x
a x b
3 2 2
3 2
2
2
0 2
2 1
0 2
2
1
+ + + + +
− − − −
− + − +
+ +
− + +
( )
( ) ( 2
2)
⇒





Portanto, a + b = 3.
Praticando
1. Responda aos itens a seguir.
a) Para que a divisão do polinômio p(x) = x5
- 2x4
- x + k por
q(x) = x - 1 deixe resto zero, o valor de k deve ser igual a quanto?
b) O resto da divisão do polinômio p(x) = x3
+ 2x2
- 3x + k por
x + 1 é igual a 3. Encontre o valor de k.
2. Responda aos itens a seguir.
I. (UFMG) O quociente da divisão de P(x) = 4x4
- 4x3
+ x - 1 por
Q(x) = 4x3
+1 é
a) x - 5.
b) x - 1.
c) x + 5.
d) 4x - 5.
e) 4x + 8.
II. (UFRGS) A divisão de P(x) por x2
+ 1 tem quociente x - 2 e resto
1. O polinômio P(x) é:
a) x2
+ x - 1.
b) x2
+ x + 1.
c) x2
+ x.
d) x3
- 2x2
+ x - 2.
e) x3
- 2x2
+ x - 1.
3. (UFSJ) Considere os polinômios:
p x x x x x r x x e q x
p x
r x
( )= + − − + ( )= + ( )=
( )
( )
4 3 2
3 2 2 12 2
, .
Sobre as raízes da equação q(x) = 0, é correto afirmar que
a) a soma de todas as raízes é igual a -1.
b) duas das raízes são inteiras.
c) duas das raízes são números complexos, um localizado no 1.º
quadrante e outro localizado no 3.º quadrante do plano de
Argand-Gauss.
d) a soma das raízes inteiras é 2.
PG19LP424SDM0_MIOLO_PVE19_4_MAT_LP.indb 167 27/05/2019 16:46:58
Matemática
168
PVE19_7_MAT_A_25
Desenvolvendo Habilidades
1. C5:H21 (FMP-2016) Seja f:  →  a função polinomial definida
por f(x) = x4
- 3x3
+ 3x - 9. O fato de x = 3 ser um zero da função f é
equivalente ao fato de o polinômio x4
- 3x3
+ 3x - 9 ser divisível por
a) x2
- 9.
b) x + 3.
c) 3.
d) x - 3.
e) x.
2. C5:H21 (Unicamp-2017) Considere o polinômio p(x) = xn
+ xm
+1, em
que n > m ≥ 1. Se o resto da divisão de p(x) por x + 1 é igual a 3, então
a) n é par e m é par.
b) n é ímpar e m é ímpar.
c) n é par e m é ímpar.
d) n é ímpar e m é par.
3. C5:H21(Unesp-2014)OpolinômioP(x)=a·x3
+2·x+bédivisívelpor
x - 2 e, quando divisível por x + 3, deixa resto -45. Nessas condições,
os valores de a e b, respectivamente, são
a) 1 e 4.
b) 1 e 12.
c) -1 e 12.
d) 2 e 16.
e) 1 e -12.
4. C5:H21 (CEFET-MG-2016 - adap.) Se uma das raízes do polinômio
P(x) = x4
 - 8x2
 + ax + b é 2 e P(1) = 9, então o valor de a5
- 4b é
a) -64.
b) -28.
c) 16.
d) 24.
e) -16.
5. C5:H21(UFRGS-2012)Se2éraizdupladopolinômiop(x) = 2x4
 - 7x3
 +
+ 3x2
+ 8x - 4, então a soma das outras raízes é
a) -1.
b) -0,5.
c) 0.
d) 0,5.
e) 1.
6. C5:H21 (Unicamp-2016 - adap.) Considere o polinômio cúbico
P(x) = x3
 + x2
 - ax - 3, em que a é um número real. Sabendo que r e
−r são raízes reais de p(x), podemos afirmar que p(1) é igual a
a) 3.
b) 1.
c) -2.
d) -4.
e) 0.
7. C5:H21 (UECE-2017) O resto da divisão de (264
+ 1) por (232
+ 1) é
igual a
a) 1.
b) 0.
c) 4.
d) 2.
e) 3.
8. C5:H21 (UEG-2016) Na divisão do polinômio 6x4
- 2x3
- 8x2
+ 10x - 2
pelo divisor x2
+ 3x - 2, o resto multiplicado por 2 é
a) -222x2
+ 252.
b) 444x2
+ 252.
c) -444x + 252.
d) 222x + 252.
e) -444x2
- 252.
9. C5:H21 (UECE-2016) O resto da divisão de (x2
+ x + 1)2
por x2
- x +1 é
a) 4x.
b) 4(x - 1).
c) 4(x - 2).
d) 4(x - 3).
e) 4(x - 4).
10. C5:H21(FGV-2016)UmdosfatoresdopolinômioP(x)=x3
+2x2
-5x -6
é (x + 3). Outro fator desse polinômio é
a) (x + 8).
b) (x - 5).
c) (x + 4).
d) (x - 1).
e) (x + 1).
Complementares
1. (UERN-2013) O produto entre o maior e o menor dos coeficientes
do quociente da divisão de P(x) = 6x5
+ 3x4
+ 5x3
- 2x2
- 4x + 5 por
D x x x
( ) = −
3 2
3
é
a) 3. b) 4. c) -2. d) -5.
2. (UEPB-2013) Os valores de m e n para os quais a expressão
5 8
2
4 2
2
x x mx n
x
+ + +
+
seja um polinômio são, respectivamente:
a) 2 e -4.
b) 0 e -2.
c) 0 e -4.
d) 2 e 4.
e) 8 e -4.
3. (ESPM-2014) O trinômio x ax b
2
+ + é divisível por x + 2 e por x - 1.
O valor de a - b é
a) 0.
b) 1.
c) 2.
d) 3.
e) 4.
4. (UERN-2012)Ovalordenparaqueadivisãodopolinômio p(x) = 2x3
 +
+5x2
+x+17pord(x)=2x2
+nx+4tenharestoiguala5éumnúmero
a) menor que -6.
b) negativo e maior que -4.
c) positivo e menor que 5.
d) par e maior que 11.
5. (UEL) O polinômio p x x x ax a
( )= + − −
3 2
3 4 é divisível pelo polinô-
mio q x x x
( )= − −
2
4 . Qual o valor de a?
a) a = −2
b) a = −1
c) a = 0
d) a = 1
e) a = 2
6. (IFAL-2011) Dividindo o polinômio p(x) pelo polinômio
( )( )( )
x x x
− − −
2 4 5 obtém-se resto x + 3. Se os restos das divisões
de p(x) por x x
− −
2 4
, e x −5 são, respectivamente, os números
A, B e C, então ABC vale
a) 100.
b) 180.
c) 200.
d) 280.
e) 360.
7. (EsPCEx/AMAN-2015) O polinômio f x x x x
( ) ,
= − + +
5 3 2
1 quando
dividido por q x x x
( ) = − +
3
3 2, deixa resto r(x). Sabendo disso, o
valor numérico de r(-1) é
a) -10.
b) -4.
c) 0.
d) 4.
e) 10.
8. (EPCAr/AFA-2011) Sobre o polinômio A(x) expresso pelo determi-
nante da matriz
x
x
x x
1 1
1 2
1
−










, é incorreto afirmar que
a) não possui raízes comuns com B x x
( )= −
2
1.
b) não possui raízes imaginárias.
c) a soma de suas raízes é igual a uma de suas raízes.
d) é divisível por P x x
( )= +2.
9. (UEPG) Com base nas assertivas a seguir, assinale o que for correto.
(1)	 Se P(x) = (2p + q - 1)x3
+ (p + q)x é um polinômio identica-
mente nulo então p - q = 2.
(2)	 Os polinômios P(x) = (x + a)2
- (x + a)(x - b) e Q(x) = 2x - 3
são idênticos. Então a e b valem, respectivamente, −
3
2
7
2
e .
(3)	 Os polinômios P(x) = 4x3
+ ax2
- 3x; Q(x) = mx2
+ nx e R(x)
= 2x -1 são tais que P(x) = Q(x) · R(x). Então a + m + n = 9.
(4)	 Se f e g são polinômios de grau n então os graus de f + g e f·g
são, respectivamente, 2n e n2
.
(5)	 O polinômio Q(x) = (x - 1)(x - 2)(x - c)(x - d) é divisível por
R(x) = x2
 - 7x + 12. Então c + d = 7.
Soma ( )
10. (Fuvest) Considere o polinômio não nulo
P(x) = a0 + a1x + a2x2
+... + anxn
emquea0,a1,a2,...,an estãoemprogressãogeométricaderazãoq≠0.
a) Calcule P
q
1





 .
b) Mostre que, para n par, o polinômio P(x) não tem raiz real.
GABARITO ONLINE
1. Faça o download do aplicativo SAE Questões ou qualquer aplicativo
de leitura QR Code.
2. Abra o aplicativo e aponte para o QR Code ao lado.
3. O gabarito deste módulo será exibido em sua tela.
GABARITO E SOLUCIONÁRIO ONLINE
1. Faça o download do aplicativo SAE Questões ou qualquer
aplicativo de leitura QR Code.
2. Abra o aplicativo e aponte para um dos QR Codes ao lado para
acessar o gabarito ou o solucionário deste módulo.
PG19LP424SDM0_MIOLO_PVE19_4_MAT_LP.indb 168 27/05/2019 16:47:07
Frente A | Livro
Tupungato/Shutterstock
Frente A
169
PVE19_7_MAT_A_26
7
Denominamos equação algébrica ou equação polino-
mial de grau n toda equação da forma:
anxn
+ an–1 xn–1
+ a 2x 2
+ ... + a1x + a0 = 0
em que a0, a1, ..., an são chamados coeficientes e podem ser
números reais ou complexos, com n e an 0.
O conjunto solução ou conjunto verdade de uma equa-
ção algébrica, no conjunto universo U, é o subconjunto de U
que contém as raízes da equação.
Duas equações são ditas equivalentes em U quando
apresentam o mesmo conjunto solução nesse domínio.
Teorema fundamental
da Álgebra
Carl Friedrich Gauss (1777-1855) demonstrou satisfa-
toriamente pela primeira vez o teorema fundamental da
Álgebra, em 1798.
Nicku/Shutterstock
O teorema afirma que:
T da e ua i ia de rau 1 ad ite a e
uma raiz complexa.
Segundo Gauss, são consequências desse resultado:
1) toda equação polinomial de grau n admite exatamente
n raízes complexas;
2) todo polinômio P(x) = anxn
+ a 1x 1
+ a 2x 2
+ ... +
+ a1x + a0 de grau n pode ser colocado na forma fatorada:
P(x) = an · (x - r1) · (x - r2) · ... · (x - rn),
em que r1, r2, ..., rn são as raízes de P(x);
3) se um polinômio de grau n possuir mais de n raízes,
então ele é identicamente nulo.
9 Exemplo:
Verificar que 1 é raiz de P(x), em que P(x) = x3
- 3x2
+ 4x - 2,
obter as outras raízes e obter a forma fatorada de P(x).
Solução:
Podemos aplicar diretamente o dispositivo de Briot-Ruffini:
1 −3 4 −2
1 1 −2 2 0
Como o resto da divisão por x -1 é 0, então 1 é raiz de P(x).
O quociente é q(x) = x2
- 2x + 2, cujas raízes são 1 i.
Como as raízes são 1, 1+ i e 1 - i, tem-se que a forma fatorada
procurada é P(x) = (x - 1)⋅(x - 1 - i)⋅(x -1 + i).
Multiplicidade de uma raiz
Dizemos que r é raiz de multiplicidade m (m ≥ 1) da equa-
ção P(x) = 0 se, e somente se,
P(x) = (x - r)m
⋅ Q(x) e Q(r) 0,
ou seja, r é raiz de multiplicidade m de P(x) = 0 quando o po-
linômio P é divisível por (x - r)m
e não é divisível por (x - r)m+1
.
Quando m = 1, dizemos que r é uma raiz simples; quando
m = 2, dupla; quando m = 3, tripla; e assim por diante.
9 Exemplos:
1) erificar qual a multiplicidade da raiz −3 na equa o
x4
+ 6x3
+ 11x2
+ 12x + 18 = 0.
Solução:
Para determinar a multiplicidade da raiz –3, dividimos o poli-
nômio x4
+ 6x3
+ 11x2
+ 12x + 18 = 0 sucessivas vezes por 3.
1 6 11 12 18
−3 1 3 2 6 0
−3 1 0 2 0
−3 1 −3 11
Observe que as divisões são exatas nas duas primeiras opera-
ções do dispositivo, isto é, a equação possui duas raízes reais
iguais a 3 e uma raiz diferente de 3.
Fazendo a decomposição de P(x):
P(x) = x4
+ 6x + 11x + 12x + 18
P(x) = (x + 3) · (x + 3x + 2x + 6)
P(x) = (x + 3) · (x + 3) · (x + 2)
P(x) = (x + 3) · (x + 2)
Portanto, –3 é a raiz de multiplicidade 2 ou a raiz dupla da
equação.
Teorema fundamental da Álgebra • Multiplicidade de uma raiz •
Pesquisa de raízes
Equações Polinômicas
ou Algébricas
Equações algébricas
PG19LP424SDM0_MIOLO_PVE19_4_MAT_LP.indb 169 27/05/2019 16:47:16
Matemática
170
PVE19_7_MAT_A_26
2) Qual o grau de uma equa o polinomial P(x) = 0 cujas raízes
são 3, 2, -1 com multiplicidades , 6 e 10, respectivamente
Solução:
P(x) = k ⋅ (x - 3)7
⋅ (x - 2)6
⋅ (x + 1)10
, com k * ⇒ gr(P) = 23
(pois + 6 + 10 = 23).
Pesquisa de raízes
Raízes racionais de equações
com coeficientes inteiros
Se r =
p
q
, com p e q inteiros e primos entre si, é uma raiz
racional da equação de coeficientes inteiros dada por
anxn
+ a 1x 1
+ a 2x 2
+ ... + a1x + a0 = 0 com an 0,
então p é divisor de a0 e q é divisor de an.
9 Exemplo:
Verificar se a equação 2x3
+ x2
+ x − 1 = 0 admite raízes racionais.
Solução:
As possíveis raízes da equação são dadas por:
p
q
x = ⇒ p 1, −1 e q 1, −1, 2, −2
p
q
x = ⇒
p
q
1 1
1
2
1
2
, , ,
− −






p(x) = 2x3
+ x2
+ x − 1
p(1) = 3 p(−1) = −3 p
1
2





 = 0; p
1
2





 = −
3
2
Logo, a única raiz racional da equação é
1
2
.
Raízes complexas de equações
com coeficientes reais
Se um complexo z = a + bi, com a e b e b 0, é raiz
de uma equação algébrica de coeficientes reais, então o con-
jugado z = a – bi também é raiz da equação.
Como consequências desse teorema:
1) toda equação algébrica de coeficientes reais e grau ím-
par admite pelo menos uma raiz real;
2) Teorema do conjugado: se o complexo z é raiz de mul-
tiplicidade m de uma equação algébrica de coeficientes
reais, então o conjugado z também é raiz de multiplici-
dade m da equação.
9 Exemplo:
Resolver a equação x4
+ 4x3
– 17x2
+ 26x 14 = 0 sabendo que
1 – i é uma de suas raízes.
Solução:
Como se trata de uma equação de coeficientes reais, se 1 – i é raiz,
então 1 + i também é raiz.
Aplicando o algoritmo de Briot-Ruffini:
1 4 −1 26 −14
1 − i 1 − i −13 − 6i 7 + 7i 0
1 + i 1 6 − 0
x2
+ 6x − = 0 ⇒ raízes: x = 1 ou x = −
= 1, − , 1+ i, 1− i .
Resolvidos
1. Sabendo que 3 é uma raiz do polinômio p(x) = x4
– 3x2
+ kx – 9,
determine o valor de k.
9 Solução:
Se 3 é uma raiz de p(x), tem-se que p(3) = 0. Logo,
34
– 3(3)2
+ 3k – 9 = 0
81 2 + 3 = 0
3k = 45
k = 15
2. (UFRGS-2014) Considere os polinômios p x x
( ) 3
e q x x x
( ) .
= +
2
O número de soluções da equação p x q x
( ) ( ), no conjunto dos
números reais, é
a) 0.
b) 1.
c) 2.
d) 3.
e) 4.
9 Solução: D
p x q x
x x x
x x x
( ) ( )
( )
=
= +
⋅ − − =
3 2
2
1 0
Tem-se, então, duas equações:
x =0 (já resolvida) ou x x
2
1 0
− − = (com discriminante ∆ = 5, portan-
to, com duas raízes distintas).
Assim, o número de soluções da equação p x q x
( ) ( )
= é 3.
3. (UFPE) Determine o polinômio com coeficientes reais p x ax bx cx
( ) = + +
3 2
,
p x ax bx cx
( ) = + +
3 2
, tal que p x p x x
+
( ) − ( ) =
1 6 2
, e indique a b c
2 2 2
+ + .
9 Solução:
Temos:
p x a x b x c x
ax a b x a b c x a b
( ) ( ) ( ) ( )
( ) ( )
+ = + + + + +
= + + + + + + +
1 1 1 1
3 2 3 2
3 2
3 2
+
+c.
Logo,
p x p x x ax a b x a b c x
( ) ( ) ( ) .
+ − = ⇔ + + + + + =
1 6 3 3 2 6
2 2 2
Para que a identidade se verifique, devemos ter:
3 6
3 2 0
0
2
3
1
a
a b
a b c
a
b
c
=
+ =
+ + =





⇔
=
= −
=





Portanto, p(x) =2x3
- 3x2
+x e a2
+ b2
+ c2
= 22
+ (-3)2
+ 12
= 14.
Praticando
1. Determine quais são as raízes da equação algébrica:
x x x x
−





 +





 −
( ) −
( )=
1
2
2
7
2 21 0
5 3
0 2
e determine a multiplicidade de cada uma delas.
PG19LP424SDM0_MIOLO_PVE19_4_MAT_LP.indb 170 27/05/2019 16:47:34
Frente A | Livro 171
7
PVE19_7_MAT_A_26
2. Uma equação algébrica possui como raízes i2
, 7i, i + 14 e –6 – i. Qual
é o menor grau possível dessa equação algébrica?
3. (Unesp) Sabe-se que, na equação x x x
3 2
4 6 0
+ + − = , uma das
raízes é igual à soma das outras duas. O conjunto solução (S) desta
equação é
a) S = {–3, –2, –1}.
b) S = {–3, –2, +1}.
c) S = {+1, +2, +3}.
d) S = {–1, +2, +3}.
e) S = {–2, +1, +3}.
Desenvolvendo Habilidades
1. C5:H21 (UECE-2017–adap.) Sejam P(x) = x5
+ x4
+ x3
+ x2
+ x +1 um
polinômio e M o conjunto dos números reais k tais que P(k) = 0. O
número de elementos de M é
a) 1.
b) 2.
c) 3.
d) 4.
e) 5.
2. C5:H21 (UECE-2016) O polinômio de menor grau, com coeficientes
inteiros, divisível por 2x – 3, que admite x = 2i como uma das raízes
e P(0) = –12 é
(Dado: i é o número complexo cujo quadrado é igual a –1.)
a) P(x) = 2x3
– 3x2
– 8x – 12.
b) P(x) = 2x3
+ 3x2
– 8x – 12.
c) P(x) = –2x3
– 3x2
– 8x – 12.
d) P(x) = 2x3
– 3x2
+ 8x – 12.
3. C5:H21 (FGV-2013) A equação x−
=
4
16 tem
a) duas raízes reais e duas raízes imaginárias conjugadas.
b) pelo menos duas raízes iguais.
c) uma única raiz imaginária.
d) quatro raízes reais.
e) quatro raízes cujo produto é −
1
4
.
4. C5:H21 (UFRGS-2012) Considere o polinômio:
p(x) = x4
+ 2x3
– 7x2
– 8x + 12.
Se p(2) = 0 e p(–2) = 0, então as raízes do polinômio p(x) são
a) –2, 0, 1 e 2.
b) –2, –1, 2 e 3.
c) –2, –1, 1 e 2.
d) –2, –1, 0 e 2.
e) –3, –2, 1 e 2.
5. C5:H21 (IFCE-2016) Para certos valores reais de p e q, a equação
(x – 1) · (x2
+ p · x + q) = 0 possui apenas uma solução real. Nessas
condições, é verdade que
a) q < 0 < p.
b) p < 0 < q.
c) 0 < p < q.
d) p < q < 0.
e) 0 = q < p.
6. C5:H21 (PUC-Rio-2016) Considere o polinômio p(x) = x2
+ bx + 3 e
assinale a alternativa correta.
a) O polinômio tem pelo menos uma raiz real para todo b .
b) O polinômio tem exatamente uma raiz real para b = 12.
c) O polinômio tem infinitas raízes reais para b = 0.
d) O polinômio não admite raiz real para b = +
1
1
3
.
e) O polinômio tem exatamente três raízes reais para b = .
7. C5:H21 (UFSM-2015) Para avaliar as vendas em 2013, o setor de
planejamento de uma empresa utilizou a função polinomial
N(t) = t3
– 21t2
+ 126t + 304
em que N representa o número de tablets vendidos no mês t, com
t = 1 correspondendo a janeiro, t = 2 correspondendo a fevereiro
e assim por diante.
De acordo com os dados, o número de tablets vendidos foi igual a
480 nos meses de
a) fevereiro, julho e novembro.
b) fevereiro, agosto e novembro.
c) fevereiro, agosto e dezembro.
d) março, agosto e dezembro.
e) março, setembro e dezembro.
8. C5:H21 (Mackenzie-2015) Seja P(x) = 2x3
– 11x2
+ 17x – 6 um poli-
nômio de 3.º grau e 2x – 1 um de seus fatores. A média aritmética
das raízes de P(x) é
a)
7
2
b)
8
2
c)
9
2
d)
10
2
e)
11
6
9. C5:H21(EsPCEx/AMAN-2016)SendoRamaiordasraízesdaequação
11 6
4
2
x
x
x
+
−
= , então o valor de 2R – 2 é
a) 2.
b) 4.
c) 6.
d) 8.
e) 10.
PG19LP424SDM0_MIOLO_PVE19_4_MAT_LP.indb 171 27/05/2019 16:47:48
Matemática
172
PVE19_7_MAT_A_26
Complementares
1. (UEPB-2013) O produto entre as raízes da equação x4
+3x2
+2=0 é:
a) 2.
b) 1.
c) 2.
d) –1.
e) 2i.
2. (UECE-2015) Se os números 2 + i, 2 – i, 1 + 2i, 1 – 2i e 0,5 são as raízes
da equação 2x5
+ px4
+ 42x3
– 78x2
+ 80x + q = 0, então o valor de
p + q + pq é
a) 287.
b) 278.
c) 297.
d) 279.
3. (UECE-2009) Se os polinômios
P x
x n m
nx x
( ) 2
1 1 1
2
e Q(x) = x3
– 4x2
+ x + 4 são idênticos, então o valor de m
n
é:
a) 2.
b) 3.
c) 4.
d) 5.
4. (IME-2013)Ospolinômios P x x ax
( )= + +
3 2
18 e Q 12
3
x x bx
( )= + +
possuem duas raízes comuns. Sabendo que ae bsão números reais,
pode-se afirmar que satisfazem à equação
a) a = b.
b) 2a = b.
c) a = 2b.
d) 2a = 3b.
e) 3a = 2b.
5. (ACAFE-2014) Sobre equações algébricas, considere as seguintes
afirmações:
I. Na equação 2 9 10 3 0
3 2
x x x
− + − = , sabendo que a, b e c são
raízes reais, o valor de log1
9
1 1 1
ab bc ac
+ +





 é −
1
2
.
II. Um recipiente cônico tem 8 dm de altura. Seu espaço interior é
ocupado por uma esfera cujo raio tem a metade da medida do
raio do cone e por 60 dm3
de água. Então, os valores inteiros
da medida do raio do recipiente cônico e do raio da esfera (em
dm) são números múltiplos de 3.
III. A equação x x x
3 2
3 4 12 0
− − + = tem raízes reais a, b e c. Então,
o determinante da matriz
0 0
0
c
b b
a a c








vale –12.
Assinale a alternativa correta.
a) Todas as afirmações estão corretas.
b) As afirmações I e III estão corretas.
c) As afirmações I e II estão corretas.
d) Apenas a afirmação II está correta.
6. (Mackenzie-2012) As raízes da equação x 9x 23x 15
3 2
− + − = 0, co-
locadas em ordem crescente, são os três primeiros termos de uma
progressão aritmética cuja soma dos 20 primeiros termos é
a) 500.
b) 480.
c) 260.
d) 400.
e) 350.
7. (Fuvest-2014) Os coeficientes a, b e c do polinômio:
p x x ax bx c
( ) = + + +
3 2
são reais. Sabendo que –1 e 1 + i, com ,
são raízes da equação p x
( ) 0 e que o resto da divisão de p(x) por
(x – 1) é 8, determine
a) o valor de ;
b) o quociente de p(x) por (x + 1).
i é a unidade imaginária, i2
= –1
8. (UFSC-2011) Assinale a(s) proposição(ões) correta(s).
(1)	 Se 3 5
n
, então log .
5 225
2 2
=
+ n
n
(2)	 Os valores reais de x que satisfazem à equação 4 4 5 2
x x
+ = ⋅
pertencem ao intervalo (2,4].
(3)	 Suponha que “Chevalier de Mére”, um jogador francês do
século XVII, que ganhava a vida apostando seu dinheiro em
jogos de dados, decidiu apostar que vai sair um“3”no lança-
mento de um dado perfeito de seis faces numeradas de 1 a
6. Com relação a esse experimento, há dois resultados possí-
veis: ou sai “3” e Chevalier ganha, ou não sai “3” e ele perde.
Cada um destes resultados – “sai um 3” ou “não sai um 3” –
tem a mesma probabilidade de ocorrer.
(4)	 Para que a função P x x px
( )= +
2
seja divisível por 4x – 1 é
necessário que p seja igual a
1
4
.
(5)	 Sea,becsãoraízesreaisdaequação x x x
3 2
20 125 250 0
− + − = ,
então o valor de log
1 1 1
a b c
+ +





 é nulo.
( )	 Se“A”é o número de arranjos de 6 elementos tomados 2 a 2;
“B”é o número de permutações de 5 elementos e “C”é o nú-
mero de combinações de 5 elementos tomados 3 a 3, então
A B C
+ − =140.
Soma ( )
GABARITO ONLINE
1. Faça o download do aplicativo SAE Questões ou qualquer aplicativo
de leitura QR Code.
2. Abra o aplicativo e aponte para o QR Code ao lado.
3. O gabarito deste módulo será exibido em sua tela.
GABARITO E SOLUCIONÁRIO ONLINE
1. Faça o download do aplicativo SAE Questões ou qualquer
aplicativo de leitura QR Code.
2. Abra o aplicativo e aponte para um dos QR Codes ao lado para
acessar o gabarito ou o solucionário deste módulo.
PG19LP424SDM0_MIOLO_PVE19_4_MAT_LP.indb 172 27/05/2019 16:47:57
Frente A | Livro
SAE DIGITAL S/A
Frente A
173
PVE19_7_MAT_A_27
7
VERSÃO 3.3
Relações de Girard
As relações de Girard são úteis na resolução de equações
polinomiais, pois elas relacionam as raízes e os coeficientes
dessas equações. Por meio dessas relações é possível estabe-
lecer um sistema de equações que permite resolver a equa-
ção inicial, cuja resolução geralmente é mais simples.
Equações do segundo grau
Se a a e ua a bri a a b 0, a 0. A divi-
dirmos a equação por a, obtemos x
b
a
x
c
a
2
0
+ + = .
Por outro lado, sendo x1 e x2 as raízes ax² + bx + c = 0, pode-
mos escrever ax bx c a x x x x x x
2 2
1 2 1 2
0
+ + = − +
( ) + ⋅



 = .
Dessa forma, ao igualarmos as duas equações:
x
b
a
x
c
a
a x x x x x x
2 2
1 2 1 2
+ + = − +
( ) + ⋅



 = 0.
De onde obtemos, portanto,
x x
b
a
x x
c
a
1 2
1 2
+ = −
⋅ =
Equações do terceiro grau
Seja a e ua a bri a a b d 0, a 0.
Ao dividirmos a equação por a, obtemos x
b
a
x
c
a
x
d
a
3 2
0
+ + + = .
Por outro lado, sendo x1, x2 e x3 as raízes de ax³ + bx² + cx +
+ d = 0, podemos escrever:
ax bx cx d a x x x x x x x x x x x x x
3 2 3
1 2 3
2
1 2 1 3 2 3 1 2
+ + + = − + +
( ) + ⋅ + ⋅ + ⋅



 − ⋅ ⋅x
x3
0
=
x x x x x x x x x x x x
1 2 3
2
1 2 1 3 2 3 1 2
− + +
( ) + ⋅ + ⋅ + ⋅ 
 − ⋅ ⋅x
x3
0
= .
Dessa forma, ao igualarmos as duas equações,
x
b
a
x
c
a
x
d
a
a x x x x x x x x x x x x x
3 2 3
1 2 3
2
1 2 1 3 2 3 1
+ + + = − + +
( ) + ⋅ + ⋅ + ⋅



 − ⋅ 2
2 3
⋅x
x x x x x x x x x x x x
1 2 3
2
1 2 1 3 2 3 1
− + +
( ) + ⋅ + ⋅ + ⋅ 
 − ⋅ 2
2 3
⋅x = 0.
De onde obtemos, portanto,
x x x
b
a
x x x x x x
c
a
x x x
d
a
1 2 3
1 2 1 3 2 3
1 2 3
+ + = −
⋅ + ⋅ + ⋅ =
⋅ ⋅ = −
Equações do quarto grau
Caso a equação seja de 4.º grau e x1, x2, x3 e x4 sejam as
raízes da equação algébrica ax4
+ bx³ + cx² + dx + e = 0, com
a   0, te :
x1 + x2 + x3 + x4 = -
b
a
x1x2 + x1x3 + x1x4 + x2x3 + x2x4 + x3x4 =
c
a
x1 x2 x3 + x1 x2 x4 + x1 x3 x4 + x2 x3 x4 = -
d
a
x1 · x2 · x3 · x4 =
e
a
9 Exemplo:
Se x1 , x2 , x3 e x4 são raízes da equação x4
- 2x3
+ 3x2
- x + = 0,
calcule o valor da expressão E =
1 1 1 1
1 2 3 4
x x x x
+ + + .
Solução:
E
x x x x x x x x x x x x
x x x x
=
+ + +
1 2 3 1 2 4 1 3 4 2 3 4
1 2 3 4
Perceba que
a = 1, b = -2, c = 3, d = - e e = .
nt o, note que o numerador i ual a − = −
−
=
d
a
( )
5
1
5 e que o
denominador vale
e
a
7
1
7. Lo o, E
5
7
.
Teorema de Bolzano
Se um polinômio P(x) apresenta valores P(a) e P(b), com
a < b, tais que P(a) · P(b) < 0 (isto é, têm sinais contrários),
então a equação admite um número ímpar (pelo menos
uma) de raízes reais entre a e b.
y
P(a)
P(b)
0 a
b
x
raiz
9 Exemplo:
Seja P(x) = x3
− 3x2
− x + 3.
Solução:
onsiderando, por exemplo, a = 0 e b = 2, em que a b, temos:
P(0) = 3 e P(2) = 23
− 3 · 22
− 2 + 3 = −3.
Relações de Girard • Teorema de Bolzano •
m.d.c. e m.m.c. de polinômios
Relações de Girard e
Teorema de Bolzano
Equações algébricas:
relações de Girard
PG19LP424SDM0_MIOLO_PVE19_4_MAT_LP.indb 173 27/05/2019 16:48:07
Matemática
174
PVE19_7_MAT_A_27
Pelo eorema de Bolzano, existe pelo menos uma raiz entre 0 e 2.
raiz
y
x
4
3
2
1
0
–1
–2
–2
0
2
m.d.c. e m.m.c. de polinômios
O máximo divisor comum (m.d.c.) entre polinômios é o
polinômio unitário (coeficiente dominante 1) formado pelos
fatores comuns aos polinômios elevados aos seus menores
expoentes. O m.d.c. é o polinômio de maior grau que divide
todos aqueles.
As raízes comuns aos polinômios também são raízes de
seu m.d.c., com a menor multiplicidade.
Se o m.d.c. de dois polinômios é 1, diz-se que eles são pri-
mos entre si.
Quando os polinômios não estão na forma fatorada, o
m.d.c. pode ser obtido pelo método das divisões sucessivas.
9 Exemplo:
Obten a o m.d.c. dos polin mios:
p(x) = x4
- 3x3
+ 3x2
- 3x + 2 e q(x) = x2
- 4x + 3.
Solução:
x2
+ x + 4
1
10
x -
3
10 quocientes
x4
- 3x3
+ 3x2
- 3x + 2 x2
- 4x + 3 10x - 10
10x - 10 0 restos
Lo o, m.d.c.(p, q) =
1
10
(10x - 10) = x - 1.
Vale notar que a divisão por 10 se faz necessária para que
o m.d.c. seja um polinômio unitário.
O mínimo múltiplo comum (m.m.c.) entre polinômios é
o polinômio unitário formado por todos os fatores que apare-
cem nos polinômios, comuns ou não, elevados ao seu maior
expoente. O m.m.c. é o polinômio de menor grau múltiplo de
todos aqueles.
Todas as raízes dos polinômios são raízes do seu m.m.c.
9 Exemplo:
ncontre o m.d.c. e o m.m.c. dos polin mios
P(x) = x(x - 1)2
(x - 2)3
e Q(x) = x3
(x - 1)(x - 3)2
.
Solução:
m.d.c. (P, Q) = x(x - 1)
m.m.c. (P, Q) = x3
(x - 1)2
(x - 2)3
(x - 3)2
Resolvidos
1. O polinômio p x x x x
( )= + − +
5 3 4 1
3 2
possui pelo menos uma raiz
no intervalo [–3, 3]?
9 Solução:
im. Observamos que p(3) = · (3) + 3 · (3) - 4 · 3 + 1 = 1 1 0
e que p(-3) = · (-3) + 3 · (-3) - 4 · (-3) + 1 = - . Lo o,
p(3) · p(-3) 0. Pelo eorema de Bolzano, concluímos que existe
raiz no intervalo -3,3 .
2. Sabendo que a, b e c são as raízes da equação x3
- x2
- 1 = 0, forme
uma nova equação cujas raízes sejam os números b + c, c + a e a + b.
9 Solução:
a + b + c = −
−
( )
1
1
= 1
⇒ b + c = 1 − a c + a = 1 − b a + b = 1 − c
⇒ = 1 − x ⇔ x = 1 −
(1 − )3
−(1 − )2
− 1 = 0 ⇒ y3
− 2 2
+ y + 1 = 0
y3
− 2 2
+ y + 1 = 0
3. (UERJ)Umciclistaeumcorredorcomeçam,juntos,umacompetição.
A curva a seguir, cuja equação é e = t3
+ at2
+ bt + c, representa a
posição em metros do ciclista, em função do tempo t, em segundos,
em que a, b, e c são números reais fixos.
3 t(s)
0
e(m)
No instante em que o ciclista parte da posição zero, o corredor inicia
um movimento, descrito pela equação e = 4t, na mesma pista e no
mesmo sentido.
Determine a posição mais afastada da origem na qual o ciclista e o
corredor voltam a se encontrar.
9 Solução:
Por meio da análise do ráfico e da equa o, verificamos que
existem tr s raízes reais: 0 raiz simples e 3 raiz dupla.
nt o, e = t · (t − 3)2
⇒ e = t3
− 6t2
+ 9t.
Para determinar os instantes dos encontros:
t3
− 6t2
+ 9t = 4t ⇔ t3
- 6t2
+ 5t = 0 ⇔ t · (t2
- 6t + ) = 0
⇒ t = 0s t = 1s e t = s.
Posi o dos encontros: 0 m 4 m e 20 m.
Posi o mais afastada = 20 m.
Praticando
1. Mostre que o polinômio p x x x x x
( )= − + + −
4 3 2
5 5 5 6 possui pelo
menos uma raiz entre os valores 0 e 4.
2. Uma das raízes do polinômio p x x x x
( )= + − −
3 2
4 4 é -2. Qual é o
produto entre as outras duas raízes?
PG19LP424SDM0_MIOLO_PVE19_4_MAT_LP.indb 174 27/05/2019 16:48:23
Frente A | Livro 175
7
PVE19_7_MAT_A_27
3. Calculeom.m.c.eom.d.c.entreospolinômios p x x x x x e q x x x
( )= − + + − ( )= + −
4 3 2 2
5 5 5 6 2 15
p x x x x x e q x x x
( )= − + + − ( )= + −
4 3 2 2
5 5 5 6 2 15 sabendo que 1 e -1 são raízes de p(x).
Desenvolvendo Habilidades
1. C5:H21 (UFU-2015) O polinômio de variável real y=p(x)=x3
-ax2
-
- 9x + ar2
é representado graficamente conforme ilustra a figura a
seguir, em que -r, r e a são constantes reais e encontram-se, nessa
ordem, em progressão aritmética (P.A.).
y
–r r a x
(Figura ilustrativa e sem escalas)
Nessas condições, o valor de a é um número
a) primo.
b) ímpar.
c) múltiplo de 5.
d) divisível por 7.
2. C5:H21 (UEG-2013) Se o coeficiente do termo de maior grau de um
polinômiodo4.ºgraué1esuasraízessãox1 =2i,x2 =-2i,x3 =3ex4 =4,
então o polinômio em questão é
a) x4
- 7x3
+ 16x2
- 28x + 48.
b) x4
- 2ix3
+ 2ix2
+ 3x + 4.
c) x4
+ 16x3
+ 4x2
- x + 18.
d) x4
- 28x3
+ 7x2
+ 48x - 28.
3. C5:H21(UDESC-2014)Sejafumpolinômiodegrauquatroquepossui
apenas raízes reais, com coeficiente do termo de maior expoente
igual à razão q da progressão geométrica formada pelas raízes de
f, cuja soma é igual a 15. Sabendo-se que a razão q é igual ao resto
da divisão de p(x) = x2
+ 1 por g(x) = x - 1, então:
a) f(x) = x4
- 15x3
+ 70x2
- 120x + 64.
b) f(x) = 2x4
+ 30x3
+ 140x2
+ 240x + 128.
c) f(x) = -2x4
+ 30x3
- 140x2
+ 240x - 128.
d) f(x) = x4
+ 15x3
+ 70x2
+ 120x + 64.
e) f(x) = 2x4
- 30x3
+ 140x2
- 240x + 128.
4. C5:H21 (UEG-2010) João gosta de brincar com números e fazer ope-
rações com eles. Em determinado momento, ele pensou em três nú-
meros naturais e, em relação a esses números, observou o seguinte:
● a soma desses números é 7;
● o produto deles é 8;
● a soma das três parcelas resultantes dos produtos desses núme-
ros tomados dois a dois é 14.
Assim, os três números pensados por João são raízes da equação
a) x3
- 7x2
+ 14x - 8 = 0.
b) x3
+ 7x2
- 14x + 8 = 0.
c) x3
- 7x2
- 14x - 8 = 0.
d) x3
+ 7x2
- 14x - 8 = 0.
5. C5:H21 (FMJ-2012) A montanha russa de um parque de diversões
tem um perfil que se encontra esboçado em linha cheia no plano
cartesiano da figura.
y
x
A função polinomial P(x), que possui pelo menos uma raiz complexa
não real, capaz de representar essa curva completa, é P(x) = Ax5
+
+Bx4
+ Cx3
+ Dx2
+ Ex + F, com
a) A > 0, F > 0, duas raízes complexas não reais e duas reais, sendo
uma de multiplicidade 1 e outra de multiplicidade 2.
b) A < 0, F > 0, duas raízes complexas não reais e duas reais, sendo
uma de multiplicidade 1 e outra de multiplicidade 2.
c) A > 0, F < 0, duas raízes complexas não reais e duas reais, sendo
uma de multiplicidade 1 e outra de multiplicidade 2.
d) A > 0, F > 0, apenas uma raiz complexa não real e duas reais,
sendo uma negativa e outra positiva.
e) A > 0, F > 0, apenas uma raiz complexa não real e duas reais,
sendo uma negativa e outra nula.
6. C5:H21 (IFAL-2017) Podemos dizer que o polinômio p(x) = x3
- 2x2
-
- 5x + 6
a) tem três raízes reais.
b) tem duas raízes reais e uma imaginária.
c) tem uma raiz real e duas imaginárias.
d) não tem raiz real.
e) tem duas raízes reais e duas imaginárias.
7. C5:H21 (FGV-2017) A equação algébrica x3
- 7x2
+ kx + 216 = 0, em
que k é um número real, possui três raízes reais. Sabendo que o
quadrado de uma das raízes dessa equação é igual ao produto das
outras duas, então o valor de k é igual a
a) -64.
b) -42.
c) -36.
d) 18.
e) 24.
8. C5:H21 (IFCE-2016) Se S é o conjunto dos números reais x para os
quais se verifica a igualdade 2 · (x3
+ 1) = 3 · (x2
+ x), então é verdade
que
a) {-1, 1, 2} S.
b) {-1, 1} S.
c) {1, 2} S.
d) {-1, 2} S.
e) S = .
9. C5:H21(UFJF-2016)Sabendoque1+iéumadasraízesdopolinômio
p(x) = x5
- 2x4
+ 2x3
- x2
+ 2x - 2, é correto afirmar que:
a) o polinômio não possui raízes reais.
b) o polinômio possui exatamente duas raízes racionais.
c) o polinômio possui exatamente duas raízes distintas.
d) o polinômio possui quatro raízes complexas não reais.
e) o polinômio possui exatamente quatro raízes distintas.
PG19LP424SDM0_MIOLO_PVE19_4_MAT_LP.indb 175 27/05/2019 16:48:35
Matemática
176
PVE19_7_MAT_A_27
10. C5:H21 (UNEB-2013 - adap.) Ao desmontar um cubo de Rubik (cubo mágico), uma criança percebeu que ele era formado por vinte e sete
cubinhos menores e que dentre esses existiam oito cubinhos com três faces pintadas, doze com apenas duas faces pintadas, seis com apenas
uma das faces pintadas e apenas um cubinho não possuía nenhuma das faces pintadas.
A tabela a seguir mostra o número de cubinhos, de cada tipo, que podem ser obtidos ao dividir a aresta de um cubo de madeira pintado, em
partes iguais.
Número de
divisões de cada
aresta
Cubinhos com três
faces
pintadas
Cubinhos com
apenas duas faces
pintadas
Cubinhos com
apenas uma face
pintada
Cubinhos sem
nenhuma face
pintada
Total de cubinhos
3 8 12 6 1 27
4 8 24 24 8 64
5 8 36 54 27 125
6 8 48 96 64 216
7 8 60 150 125 343
.
.
.
.
.
.
.
.
.
.
.
.
.
.
.
.
.
.
n 8 P1(n) P2(n) P3(n) n3
Nessas condições, pode-se afirmar que, em , a soma dos inversos das raízes do polinômio P(x) = P3(x) - P1(x) + P2(x) é igual a
a)
1
4
b)
1
2
c)
3
5
d)
2
3
e)
4
5
5. (FGV-2013) A equação 2x3
- 3x2
- 3x + 2 = 0 tem o seguinte conjunto
solução: {-1, a, b}. Podemos afirmar que o valor de a2
+ b2
é
a)
13
4
b)
7
2
c)
15
4
d) 4
e)
17
4
6. (UEPA-2014) Girolamo Cardano (1501-1576) apresentou no
livro Ars Magna, demonstrações sobre como resolver equações
cúbicas. Ele propôs para equações da forma x3
+ px + q = 0 a so-
lução x
q q p q q p
= − + + + − − +
2 4 27 2 4 27
2 3
3
2 3
3
. Sabe-se que Rafael
Bombelli (1526-1572) estendeu às ideias de Cardano e encontrou
uma das raízes da equação x3
- 15x - 4 = 0, o número 4. Nessas
condições, a soma dos inversos das outras raízes dessa equação é
a) 4.
b) 2.
c) 0.
d) -2.
e) -4.
7. (ITA) Seja P(x) um polinômio de grau 5, com coeficientes reais, admi-
tindo 2 e i como raízes. Se P(1) ⋅ P(-1) < 0, então o número de raízes
reais de P(x) pertencentes ao intervalo ]-1, 1[ é
a) 0.
b) 1.
c) 2.
d) 3.
e) 4.
8. (Fuvest) Seja f x ax a x
( ) ( )
= + − +
2
1 1, em que a é um número real
diferente de zero. Determine os valores de a para os quais as raízes
daequação f x
( ) 0 sãoreaiseonúmerox=3pertenceaointervalo
fechado compreendido entre as raízes.
GABARITO ONLINE
1. Faça o download do aplicativo SAE Questões ou qualquer aplicativo
de leitura QR Code.
2. Abra o aplicativo e aponte para o QR Code ao lado.
3. O gabarito deste módulo será exibido em sua tela.
GABARITO E SOLUCIONÁRIO ONLINE
1. Faça o download do aplicativo SAE Questões ou qualquer
aplicativo de leitura QR Code.
2. Abra o aplicativo e aponte para um dos QR Codes ao lado para
acessar o gabarito ou o solucionário deste módulo.
1. (FMP) Seja o polinômio F(x) = x4
- x3
- 16x2
+ 4x + 48. A soma e o
produto de suas raízes são, respectivamente,
a) 1 e 48.
b) 1 e 16.
c) 1 e 4.
d) -1 e 48.
e) -1 e -16.
2. (UFG) Sabe-se que todo polinômio de grau ímpar com coeficien-
tes reais admite pelo menos uma raiz real. Dado o polinômio
p x m m x x kx
( ) [( )( )]
= − + + + +
1 1 1
2 5 2
, com m, k , as condições
sobre m e k, para que o polinômio p(x) não admita raiz real, são:
a) m = 0 e k < -2.
b) m = 1 e k < -2.
c) m = 0 e k > 2.
d) m = -1 e -2 < k < 2.
e) m = 1 e -2 < k < 2.
3. (UEFS) Se a média aritmética das raízes do polinômio
p(x) = 2x2
+ rx + 5 for 7 e a das raízes de q(x) = 3x2
+ sx - 2 for 2 (sendo
r e s constantes), então, a média aritmética das raízes do polinômio
p(x) + q(x) será
a) 4.
b) 4,5.
c) 5.
d) 8,5.
e) 9.
4. (IMBEC-2013) O gráfico a seguir representa a função f(x) = x3
+ 9x2
+
+23x + 15.
x
y
Se os pontos a, b e c são as raízes de f, então 2a
+ 2b
+ 2c
é igual a
a)
21
32
b)
32
43
c)
43
54
d)
54
65
e)
65
76
Complementares
PG19LP424SDM0_MIOLO_PVE19_4_MAT_LP.indb 176 27/05/2019 16:48:43
Frente A | Livro
abeadev/Shutterstock
Frente A
177
PVE19_7_MAT_A_28
7
VERSÃO 3.3
Transformações
Uma transformação de uma equação algébrica P1(x) = 0 é
umaoperaçãocomaqualseobtémumanovaequaçãoP2(y)=0,
cujas raízes estejam relacionadas com as raízes da equação
inicial por meio de uma relação conhecida: y = f(x).
P1(x) = 0 → equação primitiva
P2(y) = 0 → equação transformada
y = f(x) → relação de transformação
Veremos,apartirdeagora,algunstiposdetransformações.
Transformação multiplicativa
É a transformação em que y = k ⋅ x (k 0). Para obter a
equação transformada basta substituir na equação primitiva
x =
y
k
.
y = k ⋅ x ⇒ x =
y
k
9 Exemplo:
Obter a equação cujas raízes são o dobro das raízes da equação
x3
+ 5x2
- 7x + 11 = 0.
Solução:
Aqui, a relação de transformação é y = 2x. Logo, x =
y
2
e substi-
tuindo na equação primitiva:
y y y
y y y
2
5
2
7
2
11 0
1
8
5
4
7
2
11 0
3 2
3 2





 +





 −





+ = ⇒ + − + = .
Multiplicando por 8, temos y3
+10y2
- 28y + 88 = 0. Esta é a equa-
ção transformada.
Transformação aditiva
É a transformação em que y = x + a (a ). Para obter a
equação transformada basta substituir na equação primitiva
x = y - a.
y = x + a ⇒ a
9 Exemplo:
Obter a equação cujas raízes são 2 unidades menores que as raízes
de 2x3
- 5x - 2 = 0.
Solução:
Aqui, a relação de transformação é y = x - 2. Logo, x = y + 2.
Substituindo na equação primitiva: 2(y + 2)3
- 5(y + 2) - 2 = 0 ⇒
2y3
+ 12y2
+ 19y + 4 = 0. Esta é a equação transformada.
Transformada aditiva e
divisão de polinômios
Dada a equação primitiva P1(x) = anxn
+ an–1xn–1
+ an–2xn–2
 
 ... a1x + a0 = 0, tem-se que a transformada aditiva é dada por:
P2(x + a) = Rn ⋅ (x + a)n
+ Rn–1 ⋅ (x + a)n–1
+
+ ... + R1 ⋅ (x + a) + R0 = 0
em que R0, R1, ... , Rn são os restos das divisões sucessivas de
P1 por x + a, que podem ser facilmente obtidos com o auxílio
do algoritmo de Briot-Ruffini.
9 Exemplo:
Dada a equação x3
− 2x2
+ x + 1 = 0, obtenha a transformada pela
relação y = x + 2.
Solução:
Aplicando Briot-Ruffini:
1 – 2 1 1
– 2 1 – 4 9 – 17 R0
– 2 1 – 6 21 R1
– 2 1 – 8 R2
– 2 1 R3
⇒ (x + 2)3
– 8(x + 2)2
+ 21(x + 2) − 1 = 0 ⇒ y3
– 8y2
+ 21y – 17 = 0.
Esta é a equação transformada.
Transformação recíproca
É a transformação em que y =
1
x
, x 0. Para obter a equação
transformada basta substituir na equação primitiva x =
1
y
.
y =
1
x
x =
1
y
9 Exemplo:
Obter a equação cujas raízes são os inversos das raízes da equação
5x3
+ x2
- x + 1 = 0.
Solução:
A relação de transformação é y =
1
x
. Logo, x =
1
y
. Substituindo
na equação primitiva:
5
1 1 1
1 0
3 2
y y y





 +





 −





+ = .Multiplicandocadaladodaigualdade
por y3
, obtém-se
⇒ − + + =
y y y
3 2
5 0, que é a equação transformada.
Transformações • Equações recíprocas
Equações algébricas:
transformações e
equações recíprocas
PG19LP424SDM0_MIOLO_PVE19_4_MAT_LP.indb 177 27/05/2019 16:48:51
Matemática
178
PVE19_7_MAT_A_28
Equações recíprocas
Uma equação polinomial P(x) = 0 é chamada recíproca se, e
somente se, é equivalente à transformada recíproca P
1
x
= 0.
Dada a equação recíproca P(x) = 0, se r é uma raiz de
multiplicidade m, então,
1
r
também é raiz com a mesma
multiplicidade.
Uma equação polinomial P(x) = 0 é recíproca se, e somente
se, os coeficientes equidistantes dos extremos são iguais 2 a
2 ou opostos 2 a 2.
Classificação
Equações recíprocas de 1.ª espécie
São aquelas em que os coeficientes equidistantes dos ex-
tremos são iguais.
Equações recíprocas de 2.ª espécie
Aquelas em que os coeficientes equidistantes dos extre-
mos são simétricos.
Forma normal
Diz-se que uma equação recíproca está na forma normal
se ela é de 1.ª espécie e grau par.
Observação
Se uma equação é recíproca de 2.ª espécie e grau par, então
ela não possui termo central.
Propriedades
● Toda equação P(x) = 0, recíproca de 2.ª espécie e grau
ímpar, admite a raiz 1. A divisão de P(x) por (x -1) con-
duz a uma equação recíproca de 1.ª espécie e grau par.
● Toda equação P(x) = 0, recíproca de 2.ª espécie e grau
par, admite raízes 1 e -1. A divisão de P(x) por (x -1) e
(x +1) conduz a uma equação recíproca de 1.ª espécie e
grau par.
● Toda equação P(x) = 0, recíproca de 1.ª espécie e grau
ímpar, admite a raiz -1. A divisão de P(x) por (x +1)
conduz a uma equação recíproca de 1.ª espécie e
grau par.
Resolução da equação
recíproca normal
Sendo a equação recíproca normal
P(x) = A0x2k
+ A1x2k–1
+...+ A1x + A0 = 0
Dividindo a equação por xk
, tem-se
A x
x
A x
x
A x
x
A
k
k
k
k k k
0 1
1
1 1
1 1 1
0
+





 +





 + + +





 + =
−
− −
...
Fazendo y = x +
1
x
e usando a identidade
x
x
y x
x
x
x
p
p
p
p
p
p
+
+
−
−
+ = ⋅ +





 − +






1
1
1
1
1 1 1
(em que p = 1, 2, 3, ...), temos:
x0
+
1
x0
= 2
x1
+
1
x1
= y
x2
+
1
x2
= y2
- 2
x3
+
1
x3
= y3
- 3y ...
Substituindo as expressões obtidas, tem-se uma equa-
ção em y de grau k. Resolvendo a equação em y, podem-
-se obter os valores de x.
9 Exemplo:
Resolva a equação x4
− 4x3
+ 5x2
− 4x + 1 = 0.
Solução:
Observando os coeficientes, verificamos que se trata de uma equa-
ção recíproca de 1.ª espécie e grau par, ou seja, na forma normal.
Dividindo a equação por x2
:
x x
x x
x
x
x
x
2
2
2
2
4 5
4 1
0
1
4
1
5 0
− + − + = ⇒ +





− +





+ =
Fazendo y = x +
1
x
x2
+
1
x2
= y2
– 2
⇒ (y2
−2) − 4 + = 0 ⇒ y2
− 4 + 3 = 0
⇒ y = 1 ou y = 3
i) x
x
x x x
+ = ⇒ − + = ⇒ =
±
1
1 1 0
1 3
2
2
ii) x
x
x x x
+ = ⇒ − + = ⇒ =
±
1
3 3 1 0
3 5
2
2
Logo, S
i i
=
+ − + −










1 3
2
1 3
2
3 5
2
3 5
2
, , , .
Resolvidos
1. (ITA) É dada a equação polinomial
a c x b c x c a x a b
+ +
( ) + + +
( ) + −
( ) + + +
( )=
2 3 1 4 0
3 2
com a, b, c
reais. Sabendo-se que esta equação é recíproca de primeira espécie
e que 1 é uma raiz, então o produto abc é igual a
a) –2.
b) 4.
c) 6.
d) 9.
e) 12.
9 Solução: E
Sendo
= ≠
a0 0, se a x ax a x a
0
3
1
2
2 3 0
+ + + = é recíproca de primeira
espécie, tem-se a a
0 3
= e a a
1 2
= . Sabendo que 1 é raiz da equação:
a c a b
b c c a
a c a b b c c a
c b
a
+ + = + +
+ + = −
+ + + + + + + + + − =





⇒
− =
+
2 4
3 1
2 4 3 1 0
2
b
b c
a b c
+ = −
+ + = −





2 1
2 5 7
PG19LP424SDM0_MIOLO_PVE19_4_MAT_LP.indb 178 27/05/2019 16:49:08
Frente A | Livro 179
7
PVE19_7_MAT_A_28
⇒
+ + = −
− =
− − =
⇒
+ + = −
− =
− =










⇒
=
a b c
c b
b c
a b c
c b
c
a
2 5 7
2
3 6
2 5 7
2
4 4
4
b
b
c
= −
= −





3
1
Logo, o produto abc é 4 3 1 12
⋅ − ⋅ − =
( ) ( ) .
2. (ITA) Determine a e b para que a equação 6x4
- ax3
+ 62x2
- 35x+
+ b - a = 0 seja recíproca de primeira classe e resolva-a.
9 Solução:
● Recíproca de 1.ª classe ⇒
b – a = 6
–a = –35
⇒
a = 35
b = 41
Então:
6x4
− 3 x3
+ 62 x2
− 3 x + 6 = 0 ( x2
)
⇒ 6x2
– 35x + 62 –
35
x
+
6
x2
= 0
6 x2
+
1
x2
– 35 x2
+
1
x
+ 62 = 0
Fazendo y = x +
1
x
x2
+
1
x2
= y2
– 2
⇒ 6(y2
− 2) − 3 + 62 = 0 ⇒ 6y2
− 3 + 0 = 0
⇒ y =
10
3
ou y =
5
2
● Se y =
5
2
: x +
1
x
=
5
2
⇔ 2x2
− 5x + 2 = 0 ⇔ x = 2 ou x =
1
2
● Se y =
10
3
: x +
1
x
=
10
3
⇔ 3x2
−10x + 3 = 0 ⇔ x = 3 ou x =
1
3
S =
1
3
,
1
2
, 2, 3
Portanto, a = 35, b = 41 e S =
1
3
,
1
2
, 2, 3
Praticando
1. Qual é a equação cujas raízes são k unidades ( k  ) menores que
as raízes da equação x³ = 4?
a) y³ – (k + 4) = 0
b) y³ – (k – 4) = 0
c) y k
− −
( )=
4 0
3
d) y k
− −
( )=
4 0
3
e) y³ + 3ky (y + k) + k³ – 4 = 0
2. (UFF) Resolva a equação 2x6
- 5x5
+ 2x4
- 2x2
+ 5x - 2 = 0.
Desenvolvendo Habilidades
1. C5:H21 (Unioeste-2013) Suponha que P(x) é um polinômio com
coeficientes reais de modo que P(x) tem exatamente 3 raízes e o
coeficiente do termo de maior grau é igual a 1. Considere que o
número real –1 e o número complexo a + bi são duas raízes de P(x).
Com relação ao polinômio P(x), pode-se afirmar que
a) se a ≥
1
2
, então todos os coeficientes são positivos.
b) se a = 0, então todos os coeficientes são positivos.
c) o coeficiente do termo quadrático é sempre nulo.
d) o termo independente é sempre um número negativo.
e) o coeficiente do termo linear é sempre menor que o termo
independente.
2. C5:H21(Fatec)Sabe-seque-1éraizdupladopolinômioP(x)=2x4
+x3
-
- 3x2
- x + 1. As outras raízes são números:
a) imaginários puros.
b) reais negativos.
c) irracionais.
d) racionais.
e) pares.
3. C5:H21 (ITA-2014) Considere o polinômio complexo p(z) = z4
+ az³ +
+5z² – iz – 6, em que a é uma constante complexa. Sabendo que 2i
é uma das raízes de p(z) = 0, as outras três raízes são
a) –3i, –1, 1.
b) –i, i, 1.
c) –i, i, –1.
d) –2i, –1, 1.
e) –2i, –i, i.
PG19LP424SDM0_MIOLO_PVE19_4_MAT_LP.indb 179 27/05/2019 16:49:23
Matemática
180
PVE19_7_MAT_A_28
4. C5:H21 (ITA-2012) Considere um polinômio p(x), de grau 5, com
coeficientes reais. Sabe-se que –2i e i- 3 são duas de suas raízes.
Sabe-se, ainda, que dividindo-se p(x) pelo polinômio q(x) = x – 5
obtém-se resto zero e que p(1) = 20(5 + 2 3). Então, p(–1) é igual a
a) 5(5 – 2 3).
b) 15(5 – 2 3).
c) 30(5 – 2 3).
d) 45(5 – 2 3).
e) 50(5 – 2 3).
Complementares
1. (IFG-2012)RenéDescartes(1596-1650)éconsideradoopaidaFiloso-
fiaModernaetambémumdosfundadoresdaMatemáticaModerna.
Sua principal obra é Discurso do Método, publicado em 1637, que
continha três apêndices, a saber: “A Dióptrica”, “A Geometria” e “Os
Meteoros”.NaparteIIIde“AGeometria”,Descartesapresentadiversas
propriedades sobre polinômios. As afirmativas de I a IV são adapta-
ções das propriedades encontradas em“A Geometria”. Analise-as.
I. Para diminuir a maior potência de uma equação polinomial,
conhecendo-se uma de suas raízes, a, basta dividir a equação
pelo binômio x – a, em que x é a variável.
II. Para saber se o valor a é a raiz de uma equação, divida o poli-
nômio pelo binômio x – a. Se a divisão for exata, então o valor
a é uma raiz.
III. Para aumentar o valor das raízes de um polinômio p(x) em 2
unidades basta fazer a substituição da variável x por x – 2.
IV. Para multiplicar (ou dividir) as raízes de um polinômio
p x a x a x a
n
n
n-1
n-1
( ) = + + … + 0 por um número real k, basta
multiplicar (ou dividir) o an–1 por k, an–2 por k2
, e assim sucessi-
vamente.
É correto afirmar que:
a) apenas a afirmativa I é correta.
b) apenas as afirmativas I e II são corretas.
c) apenas as afirmativas I, II e III são corretas.
d) todas as afirmativas são corretas.
e) nenhuma afirmativa é correta.
2. (CEFET-MG-2014) A equação
x a
x x a x a a
8 8
6 4 2 2 4 6
5
−
− − −
= , para x ≠ a,
possui
I. duas raízes reais para a = 0.
II. somente raízes imaginárias se a ≠ 0.
III. duas raízes reais e distintas para todo a .
IV. duas raízes imaginárias para a = 5.
São corretas apenas as afirmativas
a) I e II.
b) I e III.
c) I e IV.
d) II e III.
e) II e IV.
3. (ITA) Multiplicando por 2 as raízes da equação x3
–2x2
+2x–1= 0 vamos
obter raízes da seguinte equação:
a) 2y3
- 6y2
+ 6y - 4 = 0
b) y3
- 4y2
+ 8y - 8 = 0
c) 8y3
- 8y2
+ 4y - 1 = 0
d) y3
- 8y2
+ 8y + 8 = 0
e) 4y3
- 4y2
- 4y - 8 = 0
4. (ITA) Considere as afirmações:
I. A equação 3x4
- 10x3
+ 10x - 3 = 0 só admite raízes reais.
II. Toda equação recíproca admite um número par de raízes.
III. As raízes da equação x3
+ 4x2
- 4x - 16 = 0 são exatamente o
dobro das raízes de x3
+ 2x2
- x - 2 = 0.
Então:
a) Apenas I é verdadeira.
b) Apenas II é falsa.
c) Apenas III é verdadeira.
d) Todas são verdadeiras.
e) n.d.a.
5. (ITA) Para que 2x4
+ bx3
- bx - 2 = 0 tenha quatro soluções reais
distintas, devemos ter
a) b um número real qualquer.
b) b = 0.
c) b > 0.
d) b < -1.
e) b > 4.
6. (UFF) Uma fábrica utiliza dois tanques para armazenar combustível.
Os níveis de combustível, H1 e H2, em cada tanque, são dados pelas
expressões: H1(t) = 150 t3
- 190 t + 30 e H2(t) = 50 t3
+ 35 t + 30, sendo
t o tempo em hora.
Oníveldecombustíveldeumtanqueéigualaodooutronoinstante
inicial (t = 0) e, também, no instante
a) t = 0,5h.
b) t = 1,0h.
c) t = 1,5h.
d) t = 2,0h.
e) t = 2,5h.
7. (UERJ) As dimensões de um paralelepípedo retângulo são dadas
pelas raízes do polinômio 3x3
-13x2
+ 7x -1. Em relação a esse pa-
ralelepípedo, determine:
a) a razão entre a sua área total e o seu volume.
b) suas dimensões.
8. (ITA) Sabendo-se que a equação de coeficientes reais
x6
- (a + b + c)x5
+ 6x4
+ (a - 2b)x3
- 3cx2
+ 6x - 1 = 0 é uma equação
recíproca de segunda classe, então o número de raízes reais dessa
equação é
a) 0.
b) 2.
c) 3.
d) 4.
e) 6.
9. (ITA) A equação polinomial p(x) = 0 de coeficientes reais e
grau 6 é recíproca de 2.ª espécie e admite i como raiz. Se
p(2) = -
105
8
e p(-2) =
255
8
, então a soma de todas as raízes de p(x)
é igual a
a) 10.
b) 8.
c) 6.
d) 2.
e) 1.
GABARITO ONLINE
1. Faça o download do aplicativo SAE Questões ou qualquer aplicativo
de leitura QR Code.
2. Abra o aplicativo e aponte para o QR Code ao lado.
3. O gabarito deste módulo será exibido em sua tela.
GABARITO E SOLUCIONÁRIO ONLINE
1. Faça o download do aplicativo SAE Questões ou qualquer
aplicativo de leitura QR Code.
2. Abra o aplicativo e aponte para um dos QR Codes ao lado para
acessar o gabarito ou o solucionário deste módulo.
PG19LP424SDM0_MIOLO_PVE19_4_MAT_LP.indb 180 27/05/2019 16:49:31
Frente B | Livro
Matthew Cole/Shutterstock
Frente B
181
PVE19_7_MAT_B_25
7
Arco metade • Transformação em produto
Arco duplo, Arco metade
e Transformação
em produto
Operações com arcos II
Arco metade
Vimos até agora que, conhecendo as relações trigono-
métricas de um arco de medida a, podemos obter algumas
relações para arcos de medidas 2a e 3a.
Encontraremos, agora, as relações trigonométricas para
os arcos de medida
a
2
.
●
a
2
cos
cos
a a
2
1
2
= ±
+
Demonstração
Sabemos que cos cos
cos cos
cos cos
cos
2 2 1
2
2
1
2
2
1
2
2
2
2
x x
a
a
a
a
= −
=





 −





 = +
a
a a
a a
2
1
2
2
1
2





 =
+





 = ±
+
cos
cos
cos
. Fazendo uma mudança
de variáveis e chamando 2x = a, temos x =
a
2
. Assim,
cos cos
cos cos
cos cos
cos
2 2 1
2
2
1
2
2
1
2
2
2
2
x x
a
a
a
a
= −
=





 −





 = +
a
a a
a a
2
1
2
2
1
2





 =
+





 = ±
+
cos
cos
cos
● e
a
2
sen
a a
2
1
2
= ±
− cos
Demonstração
Sabe ue 2 2 2
x – 1. Fazendo a mesma mu-
dança de variáveis propostas no caso anterior:
cos
cos
cos
2 1
1 2
2
2
2
1
2
2
2
2
x sen x
a sen
a
sen
a
a
sen
a
= −
= −











 = −
2
2
1
2
2
1
2





 =
−





 = ±
−
cos
cos
a
sen
a a
● t
a
2
tg
a
sen
a
a
a
a
2
2
2
1
1
= = ±
−
+
cos
cos
cos
9 Exemplo:
Dado cos 45° =
2
2
, calcule tg 22,5°.
Solução:
tg
tg
o
o
o
o
22 5
1 45
1 45
1
2
2
1
2
2
2 2
2
2 2
2
22 5
2 2
2 2
4
,
cos
cos
,
=
−
+
=
−
+
=
−
+
=
−
+
=
−
− +
−
= −
4 2 2
4 2
3 2 2
Transformação em produto
O objetivo é transformar uma soma algébrica de funções
trigonométricas de arcos em um produto de funções trigono-
métricas dos mesmos arcos.
Vimos que:
a b a ⋅ b e a ⋅ e b I
a b a ⋅ b e a ⋅ e b II
e a b e a ⋅ b e b ⋅ a III
e a b e a ⋅ b e b ⋅ a I
Observe algumas operações feitas com estas igualdades:
I II a b a b 2 a ⋅ b
I II a b a b 2 e a ⋅ e b
III I e a b e a b 2 e a ⋅ b
III I e a b e a b 2 e b ⋅ a
Chamando a + b = p e a – b = q, temos:
a
p q
e b
p q
=
+
=
−
2 2
Substituindo, obtemos:
cos cos cos cos
cos cos
p q
p q p q
p q sen
p q
+ =
+





⋅
−






− = −
+

2
2 2
2
2





 ⋅
−






+ =
+





 ⋅
−






sen
p q
senp senq sen
p q p q
2
2
2 2
cos
s
senp senq sen
p q p q
− =
−





 ⋅
+






2
2 2
cos
PG19LP424SDM0_MIOLO_PVE19_4_MAT_LP.indb 181 27/05/2019 16:50:58
Matemática
182
PVE19_7_MAT_B_25
Resolvidos
1. (Mackenzie) Se y = 4cos 15° · cos 75°, então y2
vale:
a) 1
b)
1
4
c)
1
2
d)
3
4
e) 2
9 Solução: A
Temos
y = 4cos(45° – 30°) · cos(45° + 30°) =
=4(cos45°·cos30°+sen45°·sen30°)·(cos45°·cos30°–sen45°·sen30°).
Então,
y = ⋅ ⋅ + ⋅







⋅ ⋅ − ⋅







 =
−
( )⋅ +
( )=
4
2
2
3
2
2
2
1
2
2
2
3
2
2
2
1
2
6 2 6 2
4
36
6 4
4
6 2
4
1
−
=
−
=



⋅ ⋅ − ⋅







 =
−
( )⋅ +
( )=
2
2
3
2
2
2
1
2
6 2 6 2
4
36
6 4
4
6 2
4
1
−
=
−
=
Logo, y2
= 12
= 1.
2. Calcule cos 22° 30’.
9 Solução:
cos cos cos cos
cos
cos
cos
cos
2 2 2 1
1 2
2
1
2 2 2
2
α α α α α
α
α
α
= − ⇒ = −
=
+
⇒ = ±
+
sen
2
2
2
22 5 2 45
22 5
1
2
2
2
22 5
α
α α
Sendo tem se Logo
= ° = °
°= +
+
⇒
, , . ,
cos , cos ,
-
°
°=
+
2 2
2
3. Calcular y = sen2
24° – sen2
6° sabendo que sen18° =
5 1
4
-
.
9 Solução:
sen2
24° – sen2
6° = (sen 24° + sen 6°)(sen 24° – sen 6°) =
= 2
30
2
18
2
2
18°
2
30
2
⋅ ⋅ ⋅ ⋅
sen sen
°
cos
°
cos
°
=
= 2 · sen 15° · cos 9° · 2 · sen 9° · cos 15° =
= 2 · sen 15° · cos 15° · 2 · sen 9° · cos 9° =
= sen 30° · sen 18° =
1
2
5 1
4
5 1
8
⋅
−
=
−
Praticando
1. (UNITAU) Se sen(a–30°) = m, então cos(60° + a) é igual a:
a) 2m.
b) 1m.
c) –1m.
d) –2m.
e) 3m.
2. (UFV) Sabendo que sen 30° =
1
2
, o valor de sen 15° é
a)
3 2
2
−
( )
b)
1
4
c) 1
d)
2 3
2
−
( )
e)
1
2
3. (UFF) O valor de (sen 22,5° + cos 22,5°)2
é:
a) 1
2
2
-
b) 1
2
2
+
c) 2
2
2
+
d) 2
2
2
-
Desenvolvendo Habilidades
1. C5:H21 (Mackenzie-2013) A expressão
cos(a2
– b2
) · cos(b2
) – sen(a2
– 2b2
) · sen(b2
)
é igual a
a) cos a2
+ b2
b) sen b2
c) cos a2
d) sen a + b · a – b
e) cos a + b · a – b
PG19LP424SDM0_MIOLO_PVE19_4_MAT_LP.indb 182 27/05/2019 16:51:15
Frente B | Livro 183
7
PVE19_7_MAT_B_25
2. C5:H21(IFSP-2011)Sabendoque cos θ θ
− =
sen
6
3
,entãoovalorde
sen(2 ) é:
a) –1
b) -
5
9
c)
1
6
d)
1
3
e) 5
6
3. C5:H21(Fatec-2010)Datrigonometriasabe-sequequaisquerquese-
jamosnúmerosreaispeq,senp senq sen
p q p q
+ = ⋅
+





⋅
−






2
2 2
cos .
Logo, a expressão cosx · sen9x é idêntica a
a) sen10x + sen8x
b) 2 · (sen6x + sen2x)
c) 2 · (sen10x + sen8x)
d)
1
2
6 2
⋅ +
( )
sen x sen x
e)
1
2
10 8
⋅ +
( )
sen x sen x
4. C5:H21 (FGV-2012) O valor de y no sistema de equações
=
=
sen y
sen
y
sen
10
1
50°
50 50
1
10°
-
°
° °
° -
+
cos
cos
x
se x
n
10
é
a)
4 3
3
b) 3
c) 3 3
d)
3
3
e)
3
4
5. C5:H21 (UFSM-2013) Para melhorar as condições de acessibilidade
a uma clínica médica, foi construída uma rampa conforme indicado
na figura.
16 m
15°
c
O comprimento horizontal c da rampa, em metros, pode ser ex-
presso por
a) 4 2 3
−
( )
b) 8 2 3
-
c) 8 3
d) 4 2 3
+
( )
e) 8 2 3
+
6. C5:H21 (FGV-2013) Se sen x sen y
+ =
15
3
e cosx + cosy = 1, então,
sec(x – y) é igual a
a)
1
3
b) 1
2
c) 2
d) 3
e) 4
7. C5:H21 (EsPCEx/AMAN-2012) O cosseno do menor ângulo formado
pelos ponteiros de um relógio às 14 horas e 30 minutos vale
a) −
+
( )
3 1
2
b) −
+
( )
2 1
2
c)
1 2
4
+
( )
d) −
−
( )
6 2
4
e)
2 3
4
+
( )
8. C5:H21 (PUC-Rio-2015) Sendo x um arco satisfazendo
2
< <
x e
sen x
( ) =
24
25
, o valor de cos
x
2





 é:
a)
1
25
b) -
1
5
c)
1
5
d) -
3
5
e)
3
5
Complementares
1. (Ibmec) Considere que senx x a
− =
cos , com a > 0. Logo, sen2x é
igual a:
a) 1 – a.
b) a – 1.
c) a.
d) a + 1.
e) 2a.
2. (IME) O valor de y = sen70° cos50° + sen260° cos280° é
a) 3
b) 3
2
c) 3
3
d) 3
4
e) 3
5
PG19LP424SDM0_MIOLO_PVE19_4_MAT_LP.indb 183 27/05/2019 16:51:33
Matemática
184
PVE19_7_MAT_B_25
3. (ITA) Num triângulo ABC o lado AB mede 2cm, a altura relativa ao
lado AB mede 1cm, o ângulo ABC mede 135° e M é o ponto médio
de AB. Então a medida de BAC BMC
+ , em radianos, é igual a
a) 1
5
b)
1
4
c)
1
3
d)
3
8
e)
2
5
4. (UFU) O valor de tg10° · (sec 5° + cossec 5°) · (cos 5° – sen 5°) é igual a
a) 2
b)
1
2
c) 1
d) 2
5. (IME) Assinale a alternativa que apresenta o mesmo valor da expres-
são [4cos2
(9°) – 3] [4cos2
(27°) –3].
a) sen(9°).
b) tg(9°).
c) cos(9°).
d) sec(9°).
e) cossec(9°).
6. (Unesp-2015) Sabendo-se que cos(2x) = cos2
x – sen2
x , para quais
valores de x a função f(x) = cosx + ·
1
2
cos(2x) assume seu valor mí-
nimo no intervalo 0 < x < 2 ?
7. (ITA-2010) Se os números reais e , com + =
4
3
0
, , ma-
ximizam a soma sen + sen , então é igual a
a)
3
3
b)
2
3
c)
3
5
d)
5
8
e)
7
12
8. (UERJ) Um esqueitista treina em três rampas planas de mesmo
comprimento a, mas com inclinações diferentes. As figuras a seguir
representam as trajetórias retilíneas AB = CD = EF, contidas nas retas
de maior declive de cada rampa.
C
A
E
D
B
15º 45º
h1
h2 h3
75º
F
a
a
a
Sabendo que as alturas, em metros, dos pontos de partida A, C e
E são, respectivamente, h1, h2, e h3 conclui-se que h1 + h2 é igual a:
a) h3 3
b) h3 2
c) 2h3
d) h3
GABARITO ONLINE
1. Faça o download do aplicativo SAE Questões ou qualquer aplicativo
de leitura QR Code.
2. Abra o aplicativo e aponte para o QR Code ao lado.
3. O gabarito deste módulo será exibido em sua tela.
GABARITO E SOLUCIONÁRIO ONLINE
1. Faça o download do aplicativo SAE Questões ou qualquer
aplicativo de leitura QR Code.
2. Abra o aplicativo e aponte para um dos QR Codes ao lado para
acessar o gabarito ou o solucionário deste módulo.
PG19LP424SDM0_MIOLO_PVE19_4_MAT_LP.indb 184 27/05/2019 16:51:42
Frente B | Livro
Africa Studio/Shutterstock
Frente B
185
PVE19_7_MAT_B_26
7
Equações trigonométricas •
Inequações trigonométricas
Função tangente,
equações e
inequações
trigonométricas
Relações
fundamentais,
relações auxiliares
e equações
trigonométricas
Função seno,
equações e
inequações
trigonométricas
Função cosseno,
equações e
inequações
trigonométricas
Equações e inequações
trigonométricas
Equações trigonométricas
Para resolvermos equações trigonométricas é fundamen-
tal que tenhamos em mente algumas relações:
● Quando a equação envolve a função seno:
–
sen sen
k
ou
k k
α β
β α π
β π α π
= ⇔
= +
= −
( )+ ∈





2
2 ,
● Quando a equação envolve a função cosseno:
cos cos
( ) ,
α β
β α π
β π α π
= ⇔
= +
= − + ∈





2
2 2
k
ou
k k
2 –
● Quando a equação envolve a função tangente:
+
t t ⇔ = + k , k
9 Exemplos:
1) sen x = sen 60º
x = 60º + 360º · k ou x = 120º + 360º · k; k
2) sen x = sen
5
3
π
⋅
x =
5
3
π
⋅+2k ou x = –
−
2
3
π
+ 2k ; k
3) sen x =
1
2
sen x = sen 30º
x = 30º + 360º · k ou x = 150º + 360º · k; k
4) cos x = cos 45º
x = 45º + 360º · k ou x = 315º + 360º · k; k
5) cos x = cos
−
2
3
π
x =
−
2
3
π
+ 2k ou x =
4
3
π
⋅+ 2k ; k
6) cos x = –1
cos x = cos 180º
x = 180º + 360º · k, k
) tg x = tg 30
x = 30º + 180º · k, k
8) tg x = 1
tg x = tg 13
x = 135º + 180º · k; k
) tg x = tg
π
2
⋅
omo tg
π
2
⋅n o existe, n o existe x que satisfa a igualdade.
Inequações trigonométricas
Nas inequações trigonométricas devemos achar o interva-
lo que satisfaz às necessidades impostas.
9 Exemplos:
Ac e as solu es das inequa es para x [0, 2 .
1) sen x ≥
1
2
⇒ sen x ≥
sen 30º
sen 150º
⇒ S = [30º, 150º]
Para qualquer valor de x a solu o dada por
S k x k k
= + ≤ ≤ + ∈






π
π
π
π
6
2
5
6
2 ,
PG19LP424SDM0_MIOLO_PVE19_4_MAT_LP.indb 185 27/05/2019 16:52:26
Matemática
186
PVE19_7_MAT_B_26
150º 30º
2) cos x < –
1
2
⇒ cos x <
cos 120º
cos 240º
⇒ S = ]120º, 240º[
Para qualquer valor de x a solu o dada por
S k x k k
= + < < + ∈






2
3
2
4
3
2
π
π
π
π ,
120º
240º
3) tg x ≥ 1 ⇒ tg x ≥
tg 45º
tg 225º
⇒ S =





 ∪






π π π π
4 2
5
4
3
2
, ,
Para qualquer valor de x a solu o dada por
S k x k k
= + ≤ < + ∈






π
π
π
π
4 2
,
1
Resolvidos
1. O número de raízes da equação sen x + cos x = 0 no intervalo [0, 2 ] é
a) 0.
b) 1.
c) 2.
d) 3.
e) 4.
9 Solução: C
Tem-se que sen x + cos x = 0 ⇒ sen x = –cos x. Dessa forma,
sen x
x
cos
= –1 ⇒ tg x = –1 ⇒ S =
3
4
7
4
,
Logo, haverá duas soluções.
Perceba que as soluções pertencem aos 1.º e 4.º quadrantes, em que
a tangente é negativa.
2. Determine o conjunto solução da inequação sen x ⋅ cos x > 0, para
x [0, 2 ].
9 Solução:
As soluções serão os quadrantes em que sen x e cos x possuam o
mesmo sinal, ou seja,
S = 0
2
3
2
, ,
3. Ache o conjunto solução da inequação 2sen2
x < senx.
9 Solução:
2sen2
x – sen x < 0 sen x (2sen x –1) < 0 0 < sen x <
1
2
5
6 6
1
2
Logo,
S x k x k
= < < +
/2
6
2 ou
5
6
2 2
+ < < +
k x k k
,
Praticando
1. (PUC-Rio) Os ângulos (em graus) entre 0° e 360° para os quais
sen = cos são:
a) 45° e 90°.
b) 45° e 225°.
c) 180° e 360°.
d) 45º, 90º e 180°.
e) 90º, 180º e 270°.
2. (Unicamp) Seja x real tal que cos x = tg x. O valor de sen x é
a)
3 1
2
-
b)
1 3
2
-
c)
5 1
2
-
d)
1 5
2
-
PG19LP424SDM0_MIOLO_PVE19_4_MAT_LP.indb 186 27/05/2019 16:52:44
Frente B | Livro 187
7
PVE19_7_MAT_B_26
3. Ache o conjunto solução da inequação cos2x + cosx –1.
Desenvolvendo Habilidades
1. C5:H21 (PUC-RS-2016) Se x , então a equação cos(x) = cos(–x)
apresenta o conjunto solução
a) 
b) [–1,1]
c) [0,+ )
d) (– ,0]
e) {–1, 0, 1}
2. C5:H20 (UESPI-2012) Quantas soluções a equação senx
x
=
10
admite
noconjuntodosnúmerosreais?Aseguir,estãoesboçadososgráficos
de senx e
x
10
.
a) 5
b) 6
c) 7
d) 8
e) 9
3. C5:H21 (PUC-Rio-2017) Sabendo que cos(3x) = –1, quais são os
possíveis valores para cos(x)?
a) 0 e
1
2
b)
3
2
1
2
e
c)
1
2
e 1
d) –1 e 5
e) 0 e
3
2
4. C5:H21 (FGV-2012) No intervalo [0, 4 , a equação:
sen3
x – 2sen2
x – 5senx + 6 = 0
tem raízes cuja soma é
a) 2
b) –2
c) 6
d)
2
e) 3
5. C5:H21 (UECE-2014 – adap.) Se p e q são duas soluções da equação
2sen2
x – 3senx + 1 = 0 tais que senp senq, então, o valor da ex-
pressão sen2
p – cos2
q é igual a
a) 0.
b) 0,25.
c) 0,50.
d) 1.
e) 1,5.
6. C5:H21 (UCPel-2011) Sendo x [0, 2 ] e 2sen x – 3cos x = 0, então x
vale
a)
3
b)
2
3
c)
2
5
d)
3
4
e)
5
6
7. C5:H21 (ENaval-2012) A soma dos quadrados das raízes da equação
|senx| = 1 –2sen2
x, quando 0 < x < 2 , vale
a)
49
36
2
b)
49
9
2
c)
7
3
2
d)
14
9
2
e)
49
6
2
8. C5:H21 (CEFET-MG-2014) A solução da inequação
0
2 2
1
1
2
<
+
+
<
sen x sen x
tg x
para x 0
2
, é o conjunto
a) 0
4
,
b) 0
4
,
c) 0
2
,
d) 0
2
,
e)
4 2
,
PG19LP424SDM0_MIOLO_PVE19_4_MAT_LP.indb 187 27/05/2019 16:53:00
Matemática
188
PVE19_7_MAT_B_26
9. C5:H21 (CEFET-MG-2013 – adap.) O conjunto formado pelas raízes
dafunção f x
x x
( )=





⋅






cos cos
2
3
3
2
queestãocontidasnointervalo
[0, é
a)
3
,
b)
3
4
,
c)
3
4
4
3
,
d)
3
3
4
, ,
e)
π π
π
π
3
3
4
4
3
, , ,
{ }
10. C5:H21 (Unesp-2014) O conjunto solução (S) para a inequação
2cos2
x + cos(2x) > 2, em que 0 < x < , é dado por:
a) S x x ou x
= < < < <
0 0
6
5
6
, /
b) S x x
= < <
0
3
2
3
, /
c) S x x ou x
= < < < <
0 0
3
2
3
, /
d) S x x
= < <
0
6
5
6
, /
e) S x
= 0,
Complementares
1. (UFSJ-2012) Sendo x um arco tal que 0 x < 2 e 3 2
⋅( )=
tg x senx, é
correto afirmar que
a) a soma das soluções dessa equação é igual a .
b) as extremidades de todos os arcos x que são solução dessa
equação estão no terceiro quadrante.
c) nesse intervalo, a equação tem dois arcos distintos como so-
luções.
d) para qualquer solução dessa equação, tgx = senx.
2. (PUC-Rio) Assinale o valor de para o qual sen2 = tg .
a)
2
b)
3
c)
2
3
d)
4
3
e)
3
4
3. (UECE-2014–adap.)Usandoaexpressãoclássicadodesenvolvimen-
to da potência (a + b)n
, em que a e b são números reais e n é um
número natural, pode-se resolver facilmente a equação
sen4
x – 4sen3
x + 6sen2
x – 4sen x + 1 = 0.
Então, para os valores de x encontrados, teremos cos x igual a
a) 1
b)
3
2
c)
2
2
d) 0
4. (UFPE)Quantassoluçõesaequaçãotrigonométrica senx x
= −
1 cos
admite, no intervalo 0 80
, ?
5. (UCS) Suponha que, em determinado lugar, a temperatura média
diária,T,em°C,possaserexpressa,emfunçãodotempot,emdiasde-
corridos desde o início do ano, por T t sen
t
= +
-
14 12
2 105
364
.
Segundo esse modelo matemático, a temperatura média máxima
nesse lugar ocorre no mês de
a) julho.
b) setembro.
c) junho.
d) dezembro.
e) março.
6. (Mackenzie-2014) Em , o domínio da função f, definida por
f x
sen x
senx
( )=
2
, é
a) x x k k
/ ,
b) x k x k k
< < +
/ ,
2 2
c) x k x k k
+ +
/ ,
2
2
3
2
2
d) x k x k k x k k
< + + < +
/ ,
2
2
2
3
2
2 2 2
e) x k x k k x k k
+ + < +
/ ,
2
2
2
3
2
2 2 2
7. (UDESC-2012) A soma de todos os valores de x 0 2
, que satisfa-
zem à equação cos2
(2x) – sen2
(x) = cos6
(x) é igual a
a) .
b) 2 .
c) 5 .
d) 3 .
e) 4 .
8. (UECE-2010) O número de soluções da equação
3 3 0
2 2
sen x senx x
− + =
| | cos
que estão no intervalo [0,2 ] é
a) 2.
b) 8.
c) 4.
d) 6.
9. (Fuvest) O triângulo AOB é isósceles, com OA OB
= , e ABCD é um
quadrado. Sendo a medida do ângulo AOB, pode-se garantir que
área do quadrado é maior do que a área do triângulo se
Dados os valores aproximados:
tg14° 0,2493, tg15° 0,2679
tg20° 0,3640, tg28° 0,5317
a) 14° < < 28°.
b) 15° < < 60°.
c) 20° < < 90°.
d) 25° < < 120°.
e) 30° < < 150°.
GABARITO ONLINE
1. Faça o download do aplicativo SAE Questões ou qualquer aplicativo
de leitura QR Code.
2. Abra o aplicativo e aponte para o QR Code ao lado.
3. O gabarito deste módulo será exibido em sua tela.
GABARITO E SOLUCIONÁRIO ONLINE
1. Faça o download do aplicativo SAE Questões ou qualquer
aplicativo de leitura QR Code.
2. Abra o aplicativo e aponte para um dos QR Codes ao lado para
acessar o gabarito ou o solucionário deste módulo.
PG19LP424SDM0_MIOLO_PVE19_4_MAT_LP.indb 188 27/05/2019 16:53:10
Frente B | Livro
retrorocket/Shutterstock
Frente B
189
PVE19_7_MAT_B_27
7
Definições iniciais • Juros simples
Juros simples
Juros simples
Definições iniciais
Podemos definir os juros como o rendimento de uma apli-
cação financeira ou o valor referente ao atraso no pagamento
de uma dívida, por exemplo. Em outras palavras, os juros estão
diretamente ligados a valores financeiros. Dessa forma, antes
de falarmos sobre os juros propriamente ditos, vamos saber um
pouco mais sobre aumentos e descontos de mercadorias.
Aumentos e descontos
O preço inicial de um produto, cujo valor era Pi, foi aumen-
tado em x%. Esse produto tem o preço final (Pf) expresso por
Pf = Pi + Pi ⋅ x% = Pi ⋅ (1 + x%)
Sendo um preço inicial Pi que sofreu um desconto de x%,
o novo preço é:
Pf = Pi – Pi ⋅ x% = Pi ⋅ (1 – x%)
9 Exemplos:
1) Um produto custava R$80,00 e sofreu um aumento de 30%. Qual
o novo preço?
Solução:
80,00 ⋅ (1 + 0,3) = R$104,00.
2) Um produto que custava R$80,00 foi vendido com um desconto
de 30%. Qual o preço de venda?
Solução:
80,00 ⋅ (1 - 0,3) = R$56,00.
3) Um produto que custava R$80,00 passou a custar R$110,00. Qual
o percentual de aumento?
Solução:
80 ⋅ (1 + x) = 110 ⇒ x = 0,375 = 37,5%.
4) Um produto que custava R$80,00 foi vendido com desconto por
R$70,00. Qual o percentual do desconto?
Solução:
80 ⋅ (1 - x) = 70 ⇒ x = 0,125 = 12,5%.
5) Uma mercadoria sofreu dois aumentos sucessivos de 10%. Qual
o aumento resultante?
Solução:
Observe que, nesse caso, os acréscimos são sucessivos. Con-
siderando Pf o preço final, Pi o preço inicial e x1, x2, x3,..., xn as
taxas de acréscimos sucessivos em decimal, temos:
Pf = Pi (1 + x1) ⋅ (1 + x2) + ... + (1 + xn)
Pf = Pi ⋅ (1 + 0,1) ⋅ (1 + 0,1) = 1,21Pi
Aumento: Pf -Pi = 1,21Pi - Pi = 0,21Pi = 21%.
6) Uma mercadoria sofreu um aumento de 20% e posteriormente
um novo desconto de 20%. Qual o desconto final?
Solução:
Pf = Pi ⋅ (1 + 0,2) ⋅ (1 - 0,2) = 0,96Pi
Desconto: Pf - Pi = 0,96Pi - Pi = -0,04Pi = 4%.
Operações sobre mercadorias
São operações que envolvem a compra e venda de
mercadorias e o lucro ou prejuízo oriundos de tais operações.
Vendas com lucro: o preço de venda (V) é obtido pelo pre-
ço de custo (C) mais o lucro (L).
V = C + L
Vendas com prejuízo: o preço de venda é obtido pelo pre-
ço de custo menos o prejuízo.
V = C - P
O lucro ou o prejuízo são comumente calculados com base
no preço de custo, mas podem também ser calculados sobre
o preço de venda. Observe o quadro a seguir, sendo i a taxa
percentual.
Cálculo sobre o
preço de custo
Cálculo sobre o
preço de venda
Com lucro V = C + i ⋅ C V = C + i ⋅ V
Com prejuízo V = C - i ⋅ C V = C - i ⋅ V
9 Exemplos:
1) Uma mercadoria custou R$80,00. Por quanto deve ser vendida
para que haja um lucro de 10% sobre o preço de custo?
Solução:
V = C + i · C ⇒ V = C + 0,10 · C ⇒ V = 1,1 · C ⇒
V = 1,1 · 80 = R$88,00.
2) Uma mercadoria custou R$80,00. Por quanto deve ser vendida
para que haja um lucro de 10% sobre o preço de venda?
Solução:
V = C + i · V ⇒ V = C + 0,10 · V ⇒ V – 0,10 · V = C ⇒
0,9 V = 80 ⇒ V = R$88,89.
3) Uma mercadoria foi vendida por R$180,00 com um prejuízo de
10% sobre o preço de venda. Qual o preço de custo dessa mer-
cadoria?
Solução:
V = C – i · V ⇒ C = V + iV ⇒C = V(1 + i) ⇒ C = 180 · (1 + 0,10) ⇒
C = 180 · 1,10 ⇒ C = R$198,00.
4) Uma calça foi vendida por R$120,00 com um lucro de 20% sobre
o preço de custo. Qual o preço de custo da calça?
Solução:
V = C + i · c ⇒ V = C(1 + i) ⇒ 120 = C(1 + 0,20) ⇒
120 = 1,2 C ⇒ C = R$100,00.
PG19LP424SDM0_MIOLO_PVE19_4_MAT_LP.indb 189 27/05/2019 16:53:36
Matemática
190
PVE19_7_MAT_B_27
Juros simples
O regime de capitalização simples é aquele em que os
juros gerados em cada período são iguais e sobre eles não
incidem novos juros, ou seja, os juros não são capitalizados.
Juros simples é a remuneração recebida pela aplicação
de um capital C a uma taxa de juros de i% durante certo tem-
po t, cuja remuneração é calculada somente sobre o capital
inicial C.
Temos:
J = C ⋅ i ⋅ t
Em que:
● C é o capital inicial aplicado (principal);
● i é a taxa percentual de juros;
● t é o tempo de aplicação;
● J são os juros recebidos.
É fundamental que, na fórmula acima, o tempo t seja ex-
presso na mesma unidade a que estiver referenciada a taxa
de juros i. Dessa forma, se a taxa de juros for ao ano, o tempo
deve ser expresso em anos; já se a taxa de juros for ao mês, o
tempo deverá estar em meses.
Chama-se montante (M) o valor resgatado ao final da
aplicação do capital C.
M = C + J ⇒ M = C (1 + i · t)
No regime de capitalização a juros simples os acréscimos
ao capital em cada período são iguais, ou seja, o montante
cresce segundo uma progressão aritmética, o que pode ser
confirmado pela característica da expressão acima, que é
uma função do 1.º grau em t.
Vale citar que, para o cálculo de juros, normalmente é usa-
do o ano comercial de 360 dias, no qual os meses são sempre
considerados com 30 dias.
9 Exemplos:
1) SeR$3.000,00foramaplicadospor5mesesàtaxadejurossimples
de 4% ao mês, determine os juros recebidos e o montante.
Solução:
= · i · t = 3 000 · 0,04 · = R 600,00
= + = 3 000 + 600 = R 3.600,00
2) Um capital de R$2.000,00 foi aplicado por 7 meses a uma taxa
anualdejurossimplesde24%.Qualomontantedessaaplicação?
Solução:
Como o tempo está em meses e a taxa de juros ao ano, vamos
determinar a taxa de juros mensal:
24
24
12
2
% . .
%
. . % . .
a a a m a m
= =
M = C ⋅ (1 + i ⋅ t)
M = 2 000 ⋅ (1 + 0,02 ⋅ 7) = 2 000 ⋅ 1,14 = R$2.280,00
3) Um capital de R$5.000,00 foi aplicado por 20 dias a juros simples
a 9% ao mês. Qual o montante da aplicação?
Solução:
20 dias =
20
30
mês =
2
3
mês
i = 9% a.m.
M = C · (1 + i · t)
M = ⋅ + ⋅





 = ⋅
5000 1 0 09
2
3
5000 1 06
, ,
M = R$5.300,00
4) O capital de R$500,00 aplicado durante um ano e meio a juros
simples rendeu R$180,00. Qual a taxa mensal?
Solução:
t = 1,5 ano = 18 meses
J C i t i
J
C t
i i
i a m
= ⋅ ⋅ ⇒ =
⋅
=
⋅
⇒ =
=
180
500 18
0 02
2
,
% . .
5) A aplicação de R$3.000,00 a juros simples de 6% ao mês gerou
montante igual a R$3.420,00. Determine o prazo da aplicação.
Solução:
M = C + J ⇒ J = M - C
J = 3 420 - 3 000 = R$420,00
j C i t t
J
C i
t t t meses
t dias
= ⋅ ⋅ ⇒ =
⋅
=
⋅
⇒ = ⇒ =
=
420
3 000 0 06
420
180
7
3
70
,
Resolvidos
1. Em relação ao regime de capitalização de juros simples, resolva:
a) UminvestidoraplicouR$50.000,00aumataxadejurosde6%ao
ano por 7 anos. Imagine que o regime de capitalização é simples
e determine o montante dessa aplicação.
b) Uma pessoa realizou um empréstimo de R$220,00 para pagar
após três meses, a taxa de 12% a.t. Determine quanto essa
pessoa pagou de juros, sabendo que o regime de capitalização
utilizado foi o simples.
9 Solução:
a) M = montante
Logo, M = 50000 · (1 + 0,06 · 7) = R$71.000,00
b)
t meses
i
C
J C i t R
=
=
=





⇒ = ⋅ ⋅ = ⋅ ⋅ =
3
0 04
220
220 0 04 3 26 40
, , $ ,
2. (FGV)OSr.OliveiraaplicouR$20.000,00numacadernetadepoupan-
ça e R$30.000,00 num fundo de ações por 1 ano. Neste período, a
caderneta de poupança rendeu 8% e o fundo de ações apenas 2%.
a) Qual a taxa de rendimento global do Sr. Oliveira no período?
b) Quanto ele deveria ter aplicado no fundo de ações (mantida a
aplicação de R$20.000,00 na caderneta de poupança) para que
sua taxa global fosse de 6% ao ano?
9 Solução:
a) Calculando os juros que a caderneta de poupança e o fundo de
ações renderam, temos:
• Caderneta de poupança: R$20.000,00 ⋅ 0,08 = R$1.600,00
• Fundo de ações: R$30.000,00 ⋅ 0,02 = R$600,00
A taxa de rendimento global será:
i =
+
+
= = =
1600 600
20000 30000
2200
50000
0 044 4 4
, , %
PG19LP424SDM0_MIOLO_PVE19_4_MAT_LP.indb 190 27/05/2019 16:53:55
Frente B | Livro 191
7
PVE19_7_MAT_B_27
b) 8 20000 2
20000
6
1600 0 02
20000
0 06
% % x
%
,
,
⋅ + ⋅
+
=
+
+
=
x
x
x
1200 + 0,06x = 1600 + 0,02x
0,04x = 400
x = 10 000
3. (FGV)Benedito,ummotoristadetáxiquepercorre5040kmpormês,
analisa a hipótese de adquirir um veículo equipado com tecnologia
flex fuel, bicombustível.
No folheto de propaganda a montadora explica que o veículo
bicombustível tanto pode usar álcool como gasolina, em qualquer
proporção, apresentando a seguinte tabela de consumo, de acordo
com as proporções de combustíveis utilizadas:
Combustível Consumo
(km por litro)
Álcool Gasolina
– 100% 18
40% 60% 16
60% 40% 15
70% 30% 14
100% – 10
a) Considerando que atualmente a gasolina custa R$2,00 por litro
e que o preço do litro de álcool é 45% do preço do litro de ga-
solina, que proporção de combustíveis Benedito deveria utilizar
no veículo equipado com tecnologia flex fuel, para que tivesse
o menor gasto mensal possível?
b) Para comprar o carro bicombustível, Benedito despenderá
R$3.000,00 a mais do que gastaria se adquirisse o mesmo
modelo com motor movido a gasolina, que faz 18 km por litro.
Nas duas hipóteses, o seu carro atual entrará como parte do
pagamento. O nosso motorista está em dúvida, pois se comprar
o carro a gasolina poderá aplicar os R$3.000,00 em um fundo
de investimento que garante um rendimento de 30% de juros
no período de 3 anos. Supondo que os preços dos combustíveis
mantenham-se nos níveis atuais nos próximos 3 anos, qual a
aquisição que proporcionará maior ganho a Benedito?
9 Solução:
a) Supondo que Benedito possa usar apenas as opções apresenta-
das na tabela do enunciado, podemos montar a tabela a seguir
para estimar o gasto em cada uma das situações.
Preço por
litro (R$)
Consumo
mensal (L)
Gasto
mensal (R$)
0 · 0,90 + 1 · 2,00 = 2,00 040 : 18 = 280 2,00 · 280 = 560,00
0,4 · 0,90 + 0,6 · 2,00 =1,56 040 : 16 = 31 1,56 · 315 = 491,40
0,6 · 0,90 + 0,4 · 2,00 =1,34 040 : 1 = 336 1,34 · 336 = 450,24
0,7 · 0,90 + 0,3 · 2,00 =1,23 040 : 14 = 360 1,23 · 360 = 442,80
1 · 0,90 + 0 · 2,00 = 0,90 040 : 10 = 04 0,90 · 504 = 453,60
A análise dos resultados mostra que seu gasto mensal será o me-
norpossívelcomaproporçãode70%deálcoole30%degasolina.
b) Nas condições apresentadas, se Benedito adquirir um veículo
com tecnologia flex fuel, ele poderá economizar, no máximo,
560,00–442,80=117,20reaispormêsnogastocomcombustível.
Isso corresponde a 117,20 ⋅ 36 = 4.219,20 reais em 3 anos.
Descontando R$3.000,00, que gastaria a mais na aquisição, ele
economizaria R$1.219,20. Nesse mesmo período, a aplicação de
R$3.000,00 renderia R$900,00 (30% de R$3.000,00). Logo, a me-
lhor aquisição é a do veículo flex fuel.
Praticando
1. UmcapitaldeR$540,00éaplicadoporumanoaumataxade5%a.m.
Imaginando o regime de capitalização simples, determine o valor
dos juros obtidos.
2. Determine o valor resultante de uma aplicação de R$180,00, por 8
meses, sendo a taxa de juros simples igual a 72% a.s.
3. (FGV – adap.) Fábio recebeu um empréstimo bancário de R$10.000,00
paraserpagoemduasparcelasanuais,aserempagasrespectivamente
nofinaldoprimeiroedosegundoano,sendocobradosjurossimplesà
taxa de 20% ao ano. Sabendo que o valor da 1.ª parcela foi R$4.000,00,
podemos concluir que o valor da 2.ª foi de
a) R$8.800,00.
b) R$8.000,00.
c) R$9.600,00.
d) R$6.000,00.
e) R$10.000,00.
PG19LP424SDM0_MIOLO_PVE19_4_MAT_LP.indb 191 27/05/2019 16:54:09
Matemática
192
PVE19_7_MAT_B_27
Desenvolvendo Habilidades
1. C5:H22 (FMP-2016) A seguir são apresentados termos gerais que
definem cinco sequências de números reais, para n .
an = 80 ∙ (24)n
bn = 80 ∙ (1,30)n
cn = 80 ∙ (0,3)n
dn = 80 + 24n
en = 80 + (2,4)n
Um dos termos gerais apresentados indica o valor devido n meses
após a tomada de um empréstimo de R$80,00, calculado após a
incidência de uma taxa mensal de juros simples de 30% sobre o
valor do empréstimo. Esse termo geral é
a) an
b) bn
c) cn
d) dn
e) en
2. C5:H21 (UERJ-2015 – adap.) Considere uma mercadoria que teve
seu preço elevado de x reais para y reais. Para saber o percentual
de aumento, um cliente dividiu y por x obtendo quociente igual a
2,08 e resto igual a zero.
Em relação ao valor de x, o aumento percentual é equivalente a
a) 10,8%.
b) 20,8%.
c) 108,0%.
d) 208,0%.
e) 280,0%.
3. C5:H21 (Enem-2013) O contribuinte que vende mais de
R$20mil de ações em Bolsa de Valores em um mês de-
verá pagar Imposto de Renda. O pagamento para a Re-
ceita Federal consistirá em 15% do lucro obtido com a
venda das ações.
(Disponível em: <www.folha.uol.com.br> Acesso em. 26 abr. 2010. Adaptado).
Um contribuinte que vende por R$34 mil um lote de ações que
custou R$26 mil terá de pagar de Imposto de Renda à Receita
Federal o valor de
a) R$ 900,00.
b) R$ 1.200,00.
c) R$ 2.100,00.
d) R$ 3.900,00.
e) R$ 5.100,00.
4. C5:H22 (Enem-2010) Uma empresa possui um sistema de
controle de qualidade que classifica o seu desempenho
financeiro anual, tendo como base o do ano anterior. Os
conceitossão:insuficiente,quandoocrescimentoémenor
que 1%; regular, quando o crescimento é maior ou igual a 1% e menor
que 5%; bom, quando o crescimento é maior ou igual a 5% e menor
que 10%; ótimo, quando é maior ou igual a 10% e menor que 20%; e
excelente, quando é maior ou igual a 20%. Essa empresa apresentou
lucro de R$132.000,00 em 2008 e de R$145.000,00 em 2009.
De acordo com esse sistema de controle de qualidade, o desempe-
nho financeiro dessa empresa no ano de 2009 deve ser considerado
a) insuficiente.
b) regular.
c) bom.
d) ótimo.
e) excelente.
5. C5:H21 (IFSC-2017) Analise as seguintes situações:
1. Seu João fez um empréstimo de R$1.000,00 no Banco A, a uma
taxa de juros simples; após 4 meses, pagou um montante de
R$1.320,00 e quitou sua dívida.
2. Dona Maria fez um empréstimo de R$1.200,00 no Banco B, a
uma taxa de juros simples; após 5 meses, pagou um montante
de R$1.800,00 e quitou a dívida.
Assinale a alternativa correta.
A taxa mensal de juros simples cobrada pelo Banco A e pelo Banco
B, respectivamente, é:
a) 8% a.m. e 10% a.m.
b) 18% a.m. e 13% a.m.
c) 6,4% a.m. e 12,5% a.m.
d) 13% a.m. e 18% a.m.
e) 10% a.m. e 8% a.m.
6. C5:H21 (UFSM-2015) A chegada da televisão no Brasil facilitou o
acesso à informação. Com o avanço da tecnologia, os aparelhos
estão cada dia mais modernos e consequentemente mais caros.
Um consumidor deseja adquirir uma televisão com tecnologia de
última geração. Enquanto aguarda o preço da televisão baixar, ele
aplica o capital disponível de R$3.000,00 a juros simples de 0,8% ao
mês em uma instituição financeira, por um período de 18 meses.
O montante, ao final desse período, é igual a
a) R$7.320,00.
b) R$5.400,00.
c) R$4.320,00.
d) R$3.432,00.
e) R$3.240,00.
7. C5:H21(UEPA-2015)Umagricultorfinancioujuntoaumacooperativa
os insumos utilizados na lavoura em 2014. Pagou 20% do valor dos
insumos no ato da compra, utilizando parte do lucro obtido no ano
anterior, e financiou o restante em 10 meses a uma taxa de 2% ao
mês a juros simples. Observou que havia gastado o montante de
R$208.800,00 com a parte financiada. Neste caso, o valor financiado
dos insumos pelo agricultor foi de
a) R$217.500,00.
b) R$174.000,00.
c) R$164.000,00.
d) R$144.500,00.
e) R$136.000,00.
8. C5:H21 (Enem-2011) Uma pessoa aplicou certa quantia
em ações. No primeiro mês, ela perdeu 30% do total do
investimento e, no segundo mês, recuperou 20% do que
havia perdido. Depois desses dois meses, resolveu tirar
o montante de R$3.800,00 gerado pela aplicação.
A quantia inicial que essa pessoa aplicou em ações corresponde
ao valor de
a) R$4.222,22.
b) R$4.523,80.
c) R$5.000,00.
d) R$13.300,00.
e) R$17.100,00.
9. C5:H21 (Enem-2015)Umfornecedorvendiacaixasdeleite
a um supermercado por R$1,50 a unidade. O supermerca-
docostumavacomprar3000caixasdeleitepormêsdesse
fornecedor.Umaforteseca,ocorridanaregiãoondeoleite
éproduzido,forçouofornecedoraencareceropreçodevendaem40%.
Osupermercadodecidiuentãocortarem20%acompramensaldessas
caixas de leite. Após essas mudanças, o fornecedor verificou que sua
receita nas vendas ao supermercado tinha aumentado. O aumento da
receita nas vendas do fornecedor, em reais, foi de
a) 540.
b) 600.
c) 900.
d) 1260.
e) 1500.
10. C6:H25 (Enem-2011) Um jovem investidor precisa esco-
lher qual  investimento lhe trará maior retorno financei-
ro em uma aplicação de R$500,00. Para isso, pesquisa o
rendimento e o imposto a ser pago em dois investimen-
tos: poupança  e CDB (certificado de depósito bancário). As infor-
mações obtidas estão resumidas no quadro:
Rendimento
Mensal (%)
IR (Imposto de
Renda)
Poupança 0,560 Isento
CDB 0,876 4% (sobre o ganho)
Paraojoveminvestidor,aofinaldeummês,aaplicaçãomaisvantajosaé
a) a poupança, pois totalizará um montante de R$502,80.
b) a poupança, pois totalizará um montante de R$500,56.
c) o CDB, pois totalizará um montante de R$504,38.
d) o CDB, pois totalizará um montante de R$504,21.
e) o CDB, pois totalizará um montante de R$500,87.
PG19LP424SDM0_MIOLO_PVE19_4_MAT_LP.indb 192 27/05/2019 16:54:10
Divisão de polinômios: métodos da chave e de Descartes
Divisão de polinômios: métodos da chave e de Descartes
Divisão de polinômios: métodos da chave e de Descartes
Divisão de polinômios: métodos da chave e de Descartes
Divisão de polinômios: métodos da chave e de Descartes
Divisão de polinômios: métodos da chave e de Descartes
Divisão de polinômios: métodos da chave e de Descartes
Divisão de polinômios: métodos da chave e de Descartes
Divisão de polinômios: métodos da chave e de Descartes
Divisão de polinômios: métodos da chave e de Descartes
Divisão de polinômios: métodos da chave e de Descartes
Divisão de polinômios: métodos da chave e de Descartes
Divisão de polinômios: métodos da chave e de Descartes
Divisão de polinômios: métodos da chave e de Descartes
Divisão de polinômios: métodos da chave e de Descartes
Divisão de polinômios: métodos da chave e de Descartes
Divisão de polinômios: métodos da chave e de Descartes
Divisão de polinômios: métodos da chave e de Descartes
Divisão de polinômios: métodos da chave e de Descartes
Divisão de polinômios: métodos da chave e de Descartes
Divisão de polinômios: métodos da chave e de Descartes
Divisão de polinômios: métodos da chave e de Descartes
Divisão de polinômios: métodos da chave e de Descartes
Divisão de polinômios: métodos da chave e de Descartes
Divisão de polinômios: métodos da chave e de Descartes
Divisão de polinômios: métodos da chave e de Descartes
Divisão de polinômios: métodos da chave e de Descartes
Divisão de polinômios: métodos da chave e de Descartes
Divisão de polinômios: métodos da chave e de Descartes
Divisão de polinômios: métodos da chave e de Descartes
Divisão de polinômios: métodos da chave e de Descartes
Divisão de polinômios: métodos da chave e de Descartes
Divisão de polinômios: métodos da chave e de Descartes
Divisão de polinômios: métodos da chave e de Descartes
Divisão de polinômios: métodos da chave e de Descartes
Divisão de polinômios: métodos da chave e de Descartes
Divisão de polinômios: métodos da chave e de Descartes
Divisão de polinômios: métodos da chave e de Descartes
Divisão de polinômios: métodos da chave e de Descartes
Divisão de polinômios: métodos da chave e de Descartes
Divisão de polinômios: métodos da chave e de Descartes
Divisão de polinômios: métodos da chave e de Descartes
Divisão de polinômios: métodos da chave e de Descartes
Divisão de polinômios: métodos da chave e de Descartes
Divisão de polinômios: métodos da chave e de Descartes
Divisão de polinômios: métodos da chave e de Descartes

Mais conteúdo relacionado

Semelhante a Divisão de polinômios: métodos da chave e de Descartes

Ppt avaliação diagnóstica em processo
Ppt avaliação diagnóstica em processoPpt avaliação diagnóstica em processo
Ppt avaliação diagnóstica em processoGisele Ferreira
 
Plano de trabalho - Teorema de Pitágoras
Plano de trabalho - Teorema de PitágorasPlano de trabalho - Teorema de Pitágoras
Plano de trabalho - Teorema de PitágorasLuciane Oliveira
 
7º ano pet 3 - laboratório de matemática ok
7º ano   pet 3 - laboratório de matemática ok7º ano   pet 3 - laboratório de matemática ok
7º ano pet 3 - laboratório de matemática okThaysBotelho2
 
Execução do projeto de aprendizagem
Execução do projeto de aprendizagemExecução do projeto de aprendizagem
Execução do projeto de aprendizagemsilviamatematica
 
Livro Bianchini - 9º Ano.pdf
Livro Bianchini - 9º Ano.pdfLivro Bianchini - 9º Ano.pdf
Livro Bianchini - 9º Ano.pdfCleuvânia Dias
 
Apostila revisão fundamental II
Apostila revisão fundamental IIApostila revisão fundamental II
Apostila revisão fundamental IIqcavalcante
 
Atps estatística pedro
Atps estatística pedroAtps estatística pedro
Atps estatística pedroPeter San
 
Matemática 1º ano
Matemática   1º anoMatemática   1º ano
Matemática 1º anoEdpositivo
 
Livro aprender mais_matematica_ens_medio3
Livro aprender mais_matematica_ens_medio3Livro aprender mais_matematica_ens_medio3
Livro aprender mais_matematica_ens_medio3elannialins
 
Apostila verao 19 passos 1
Apostila verao 19 passos 1Apostila verao 19 passos 1
Apostila verao 19 passos 1Otávio Sales
 
modelo de apresentação estado de PE.pptx
modelo de apresentação estado de PE.pptxmodelo de apresentação estado de PE.pptx
modelo de apresentação estado de PE.pptxGyl Souza
 
Universidade federal de campina grande programa institucional de bolsas de in...
Universidade federal de campina grande programa institucional de bolsas de in...Universidade federal de campina grande programa institucional de bolsas de in...
Universidade federal de campina grande programa institucional de bolsas de in...qcavalcante
 

Semelhante a Divisão de polinômios: métodos da chave e de Descartes (19)

Ppt avaliação diagnóstica em processo
Ppt avaliação diagnóstica em processoPpt avaliação diagnóstica em processo
Ppt avaliação diagnóstica em processo
 
Plano de trabalho - Teorema de Pitágoras
Plano de trabalho - Teorema de PitágorasPlano de trabalho - Teorema de Pitágoras
Plano de trabalho - Teorema de Pitágoras
 
Função poliomial do 1 grau gabarito
Função poliomial do 1 grau gabaritoFunção poliomial do 1 grau gabarito
Função poliomial do 1 grau gabarito
 
Função poliomial do 1 grau gabarito
Função poliomial do 1 grau gabaritoFunção poliomial do 1 grau gabarito
Função poliomial do 1 grau gabarito
 
AULA Limites
AULA LimitesAULA Limites
AULA Limites
 
JLL08082012.pdf
JLL08082012.pdfJLL08082012.pdf
JLL08082012.pdf
 
7º ano pet 3 - laboratório de matemática ok
7º ano   pet 3 - laboratório de matemática ok7º ano   pet 3 - laboratório de matemática ok
7º ano pet 3 - laboratório de matemática ok
 
Execução do projeto de aprendizagem
Execução do projeto de aprendizagemExecução do projeto de aprendizagem
Execução do projeto de aprendizagem
 
Livro Bianchini - 9º Ano.pdf
Livro Bianchini - 9º Ano.pdfLivro Bianchini - 9º Ano.pdf
Livro Bianchini - 9º Ano.pdf
 
Apostila revisão fundamental II
Apostila revisão fundamental IIApostila revisão fundamental II
Apostila revisão fundamental II
 
Atps estatística pedro
Atps estatística pedroAtps estatística pedro
Atps estatística pedro
 
Matemática 1º ano
Matemática   1º anoMatemática   1º ano
Matemática 1º ano
 
Porcentagem e juros
Porcentagem e jurosPorcentagem e juros
Porcentagem e juros
 
Livro aprender mais_matematica_ens_medio3
Livro aprender mais_matematica_ens_medio3Livro aprender mais_matematica_ens_medio3
Livro aprender mais_matematica_ens_medio3
 
Apostila verao 19 passos 1
Apostila verao 19 passos 1Apostila verao 19 passos 1
Apostila verao 19 passos 1
 
modelo de apresentação estado de PE.pptx
modelo de apresentação estado de PE.pptxmodelo de apresentação estado de PE.pptx
modelo de apresentação estado de PE.pptx
 
Universidade federal de campina grande programa institucional de bolsas de in...
Universidade federal de campina grande programa institucional de bolsas de in...Universidade federal de campina grande programa institucional de bolsas de in...
Universidade federal de campina grande programa institucional de bolsas de in...
 
Portfólio 3 revezamento
Portfólio 3 revezamentoPortfólio 3 revezamento
Portfólio 3 revezamento
 
Monografia José Matemática 2010
Monografia José Matemática 2010Monografia José Matemática 2010
Monografia José Matemática 2010
 

Divisão de polinômios: métodos da chave e de Descartes

  • 2. Todos os direitos reservados. SAE DIGITAL S/A. R. João Domachoski, 5. CEP: 81200-150 Mossunguê – Curitiba – PR 0800 725 9797 | Site: sae.digital Produção © 2019 – SAE DIGITAL S/A. É proibida a reprodução, mesmo parcial, por qualquer processo, sem autorização por escrito dos autores e do detentor dos direitos autorais. FICHA CATALOGRÁFICA S132 SAE, Extensivo Mega: matemática, pré-vestibular: livro 4 : livro do professor / SAE DIGITAL S/A. - 1. ed. - Curitiba, PR : SAE DIGITAL S/A, 2019. 80 p. : il. ; 28 cm. ISBN 978-85-535-0824-2 1. Matemática - Estudo e ensino (Ensino Médio). 2. Matemática (Ensino Médio) - Problemas, questões, exercícios. 3. Universidades e faculdades - Vestibular. I. Título: Matemática, Pré-vestibular: Livro 4 : Livro do Professor. III. Título CDD: 372.7 CDU: 373.3.016:510 Gerência editorial Luiz Henrique Pereira Mainardes Coordenação editorial Tassiane Aparecida Sauerbier Edição Anna Paula Chiarello Marcon, Eliane Peixoto de Lima, Janayna da Costa Goulart, Janile Oliveira, Rodrigo Zeni Stocco Revisão Ana Paula Gurki Ferraz, Brunno Freire da Silva Neto, Camille Chiquetti, GabrieleVarão da Costa, Juliana Basichetti Martins, Katieli Silva, MarcelaVidal Machado, MarileneWojslaw Pereira Dias, Pãmela Leal, Priscila de Jesus Sousa,Thainara Gabardo,Victor Augusto de LimaTruccolo Cotejo AndreiaVidal, Laura Akemi, Mariana Chaves, Mariana Passarin, Mellanie Novais, Polyana Fonseca, Sthefanie Lhorente Coordenação processos Erica Fujihara Processos Cleyton Dall’Agnol, Janio Junior, Raul Jungles Coordenação qualidade Vanessa Marques Cabral dos Santos Qualidade Bruna Ferreira Rodrigues, Everson de Lara Caetano, Henrique Sossélla, MarileneWojslaw Pereira Dias Coordenação produção visual Mauricio Ragadalli Iconografia Antonio Sevilha Cartografia Júlio Manoel França da Silva Ilustrações Deny Machado, Kássio Luiz Dias Nery Arte da capa Kássio Luiz Dias Nery Projeto gráfico Evandro Pissaia, Rafael Chueire Diagramação André Lima, Evandro Pissaia, Bruno M. H. Gogolla, Gustavo RibeiroVieira, Jéssica Suelen de Morais, Jéssica Xavier, Juliana Hiromi Saito, Kássio Luiz Dias Nery, Luisa Piechnik Souza, Maísa Leepkaln, Mariana Oliveira, Mikhael Gusso, Nadiny da Silva, Ralph Glauber, Silvia Santos,Tarliny da Silva,Thiago FigueiredoVenâncio Autores Carolina de Almeida Santos Pinotti, Ednei Leite de Araújo, Maria Fernanda Martini Campagnaro Créditos da capa Vadym Pasichnyk/Valery Brozhinsky/Shutterstock |Turing Archive/W. Commons PI_PVE19_L4_MAT_PROF.indd 2 28/05/2019 11:16:54
  • 3. Livro 7 Revisão 4 Frente A Frente B Frente C Frente A Frente B Frente C Índice PG19LP424SDM0_MIOLO_PVE19_4_MAT_LP.indb 3 27/05/2019 16:45:43
  • 4. CONHEÇA OS RECURSOS DIGITAIS DIGITAIS DIGITAIS PLATAFORMA ADAPTATIVA ENTENDEU A AULA DE HOJE? A plataforma adaptativa permite que você resolva exercícios encaminhados pelo professor e revise o conteúdo visto na semana. Veja como é fácil: COMO VOCÊ E SUA FAMÍLIA ACOMPANHAM SEU DESEMPENHO? Quando alguma atividade da plataforma adaptativa for agendada, aluno e responsável serão comunicados por meio de um aplicativo. Além de enviar um lembrete da tarefa ao aluno, essa ferramenta permite ao responsável o acompanhamento, em tempo real, do desempenho do estudante e o momento em que o exercício é concluído. As notificações serão enviadas quando: • uma atividade for encaminhada; • restarem 24 horas para o encerra- mento do período para a atividade; • os exercícios forem resolvidos; • a tarefa não foi realizada. DIGITAIS DIGITAIS DIGITAIS Selecione a disciplina disponível e resolva os 4 exercícios propostos na plataforma. Acesse o site ava.sae.digital e insira seu login e senha. Caso seu desempenho esteja abaixo do esperado, você deverá assistir a uma videoaula sobre o conteúdo. Após assistir à videoaula, 3 questões deverão ser respondidas para finalizar a tarefa e verificar seu aprendizado. 1 2 3 4 • Acesse a Play Store (Android) ou a App Store (IOS) de seu smartphone e faça o download do aplicativo SAE NOTIFICA. • No aplicativo, insira seu login e senha (aluno e responsável, conforme cadas- tro da escola). Acesse a Play Store (Android) ou a App e faça o SAE NOTIFICA. No aplicativo, insira seu login e senha (aluno e responsável, conforme cadas- Dmi T; Omelchenko; photka; Gts; GCapture/Shutterstock PG19LP424SDM0_MIOLO_PVE19_4_MAT_LP.indb 4 27/05/2019 16:45:50
  • 5. SAE QUESTÕES ARRASE NO ENEM Em consonância com o Ano Internacional da Tabela Periódica dos Elementos Químicos, declarado pela ONU, apresentamos como Objeto Digital um simulador da Tabela Periódica, que possibilita ao aluno e ao professor interagirem de forma dinâmica e interativa com os 118 elementos químicos. O QUE O ENEM ESTÁ AVALIANDO? Com o aplicativo SAE QUESTÕES você poderá aprimorar seus estudos solucionando as questões presentes nas edições da avaliação do ENEM. TEMAS DA ATUALIDADE Acessandooitem“Atualidades”doaplicativo,épossívelacompanharocanal“DOBRADINHASAE”,quetraz, semanalmente, um professor de redação juntamente com um professor de outra disciplina debatendo temáticas da atualidade e dando dicas e sugestões de como escrever e estruturar uma boa redação. QRCODE NAS QUESTÕES ENEM As questões do ENEM possuem QR codes. Depois de resolver essas questões, confira o vídeo com a resolução comentada. Para isso, acesse o item “Leitor QRcode” do aplicativo e posicione a câmera de seu smartphone em frente ao código da questão. Para ter acesso a esse recurso, basta: 1. Baixar o aplicativo SAE QUESTÕES, disponível na Play Store (Android) e também na App Store (IOS). 2. Fazer um cadastro informando um e-mail válido e elaborar uma senha de acesso. SIMULADOR – TABELA PERIÓDICA Quer acessar centenas de aulas para arrasar no ENEM? Acesse a plataforma adaptativa com seu login e senha e clique em ARRASE NO ENEM. Você vai encontrar inúmeras videoaulas com conteúdo completo para o ENEM. E não acaba aí! Você poderá contar com roteiros de estudos em formato PDF presentes em cada videoaula disponível. Também é possível acessar as aulas pelos QRcodes presentes nas aberturas dos módulos. SIMULADOR – TABELA PERIÓDICA PG19LP424SDM0_MIOLO_PVE19_4_MAT_LP.indb 5 27/05/2019 16:45:53
  • 6. 7 Extensivo MEGA Livro Sumário VICHAILAO/Shutterstock VICHAILAO/Shutterstock Matemática Frente A Polinômios II 165 Equações algébricas 169 Equações algébricas: relações de Girard 173 Equações algébricas: transformações e equações recíprocas 177 Frente B Operações com arcos II 181 Equações e inequações trigonométricas 185 Juros simples 189 Juros compostos 194 Frente C Distância entre ponto e reta e área de um triângulo 199 Equações da circunferência 204 Circunferência: posições relativas 209 Cônicas 215 PG19LP424SDM0_MIOLO_PVE19_4_MAT_LP.indb 164 27/05/2019 16:46:14
  • 7. Frente A | Livro Ziga Cetrtic/Shutterstock Frente A 165 PVE19_7_MAT_A_25 7 VERSÃO 3.3 Método da chave Dados dois polinômios P(x) e D(x), de graus p e q respec- tivamente, ao dividirmos P(x) por D(x) encontramos dois poli- nômios Q(x) e R(x), denominados quociente e resto respecti- vamente, que satisfazem à: P(x) = D(x) ⋅ Q(x) + R(x) Em que: ● o grau de R(x) deve ser menor que o grau de D(x); ● ou R(x) = 0. Então, ● se gr(P) < gr(D), então Q(x) = 0 e R(x) = P(x). ● se gr(P) ≥ gr(D), a divisão pode ser efetuada pelo seguin- te algoritmo denominado método da chave: I. ordenam-se P(x) e D(x) segundo as potências decres- centes de x, inclusive com os termos do dividendo que possuem coeficiente 0; II. divide-se o primeiro termo de P(x) pelo primeiro termo de D(x), obtendo-se o primeiro termo do quociente; III. multiplica-se D(x) pelo primeiro termo do quociente e subtrai-se o resultado de P(x), obtendo-se o primeiro resto parcial; IV. com o primeiro resto parcial e o divisor D(x) repetem-se as operações, obtendo-se o segundo termo do quocien- te, e assim sucessivamente, até encontrar um resto de grau menor que o divisor. 9 Exemplo: Calcule a divisão de (x3 + 2x -1) por (x2 + x + 2) Solução: x3 + 0x2 + 2x − 1 x2 + x + 2 −x3 − x2 − 2x x − 1 -x2 + 0x − 1 x2 + x + 2 x + 1 Dessa forma, Q(x) = x − 1 e R(x) = x + 1. Observação O grau do quociente é a diferença dos graus do dividendo e do divisor. gr(Q) = gr(P) - gr(D) No exemplo acima o quociente tem grau 3 - 2 = 1. Método de Descartes A divisão de polinômios também pode ser efetuada pelo método de Descartes, que é uma aplicação da identidade de polinômios. Nesse método, parte-se da expressão P(x) = D(x) ⋅ Q(x) + R(x), em que gr(Q) = gr(P) - gr(D) e gr(R)MAX = gr(D) - 1. O quociente e o resto são obtidos igualando-se os coefi- cientes dos dois lados. 9 Exemplos: 1) Divida P(x) = x4 + 2x3 + 3x2 + 4x + 5 por D(x) = x3 + 1, a fim de encontrar Q(x) e R(x). Solução: Supondo Q(x) = ax + b e R(x) = cx2 + dx + e, temos: P(x) = Q(x)·D(x) + R(x) x4 + 2x3 + 3x2 + 4x + 5 = (ax + b)·(x3 + 1) + (cx2 + dx + e) ⇒ ⇒ x4 + 2x3 + 3x2 + 4x + 5 = ax4 + bx3 + cx2 + (a + d)x + (b + e) a = 1 b = 2 c = 3 a + d = 4 ⇒ d = 3 b + e = 5 ⇒ e = 3 ⇒ Q(x) = x + 2 e R(x) = 3x2 + 3x + 3 2) Determine os valores de p e q de modo que x3 − 6x2 + px − 1 seja divisível por x2 + 3x − q. Solução: Devemos fazer o resto R(x) = 0 e adotar um quociente Q(x) = ax + b do primeiro grau. Assim, x3 − 6x2 + px − 1 = (x2 + 3x − q) · (ax + b) ⇒ ⇒ x3 − 6x2 + px − 1 = ax3 + (b + 3a)x2 + (3b − aq)x −bq Igualando os coeficientes dos termos de mesmo grau, temos: ● a = 1 ● b + 3a = - 6 ⇒ b + 3 · 1 = - 6 b + 3 = - 6 ⇒ b = - 6 - 3 b = - 9 ● 3b - aq = p ⇒ 3 · (- 9) - 1 · q = p - 27 - q = p ⇒ p + q = - 27 ● - bq = - 1 - (- 9) q = - 1 9q = - 1 ⇒ q = - 1 9 ● p + q = - 27 p = - 27 - −       1 9 = − 242 9 Logo, p = − 242 9 e q = − 1 9 Método da chave • Método de Descartes • Dispositivo de Briot-Ruffini Métodos da chave, Dispositivo de Briot-Ruffini e Teorema de d’Alembert Polinômios II PG19LP424SDM0_MIOLO_PVE19_4_MAT_LP.indb 165 27/05/2019 16:46:27
  • 8. Matemática 166 PVE19_7_MAT_A_25 Um polinômio identicamente nulo é aquele que é nulo para qualquer valor da variável e tem todos os seus coeficien- tes iguais a zero. Importante Se um polinômio de grau n possuir mais de n raízes, ele é identicamente nulo. Teorema do Resto (ou de d’Alembert) O resto da divisão de um polinômio P(x) por ax + b, a 0, i ua a P −       b a . Demonstração Na divisão de P(x) por ax + b o resto deve ter grau zero. Assim, podemos dizer que a divisão terá um quociente Q(x) e resto R(x) (R = constante). Logo, P(x) = (ax + b) ⋅ Q(x) + R(x) ⇔ P(x) = (ax + b) ⋅ Q(x) + R Fazendo x = - b a , P b a a b a b Q b a R R P b a −       = −       +       −       + ⇔ = −       ⋅ 9 Exemplo: Calcule o resto de P(x) = x3 + x2 + x + 1 por x + 1. Solução: Em x + 1 temos a = 1 e b = 1. Então − b a = − = − 1 1 1. O resto será P(−1) = (−1)3 + (−1)2 + (−1) + 1 = 0. Logo, 0 é o resto de P(x). Teorema: o polinômio P(x) é divisível por ax + b, com a 0 se, e somente se, P −       b a = 0. 9 Exemplo: Determine m para que o polinômio P(x) = x3 + 2x2 + mx -10 seja divisível por x - 2. Solução: Em x - 2 temos a = 1 e b = - 2. Então: − = − − ( ) = b a 2 1 2 P(2) = 23 + 2 ⋅ 22 + m ⋅ 2 - 10 = 0 ⇒ m = -3 Dispositivo de Briot-Ruffini Podemos usar o dispositivo de Briot-Ruffini para dividir um polinômio de grau maior ou igual a 1 por um binômio do tipo x - a. Esse dispositivo consiste em realizar a divisão efe- tuando cálculos com os coeficientes. b er e di idir    2x3 - 5x2 + 3  - 4 r      - 2 por meio do dispositivo de Briot-Ruffini. ● Posicionamos a raiz do divisor e os coeficientes do divi- dendo observando a ordem decrescente dos expoentes de x do polinômio completo: Raiz do divisor Coeficientes do dividendo 2 2 −5 3 −4 ● O coeficiente do primeiro termo do quociente é igual ao coeficiente do primeiro termo do dividendo: 2 = 2 2 −5 3 −4 ● Multiplicamos a raiz do divisor pelo coeficiente que foi repetido e adicionamos o produto encontrado ao se- gundo coeficiente do dividendo: 2 x + 2 2 −5 −1 2 · 2 + (−5) 3 −4 ● Agora, multiplicamos a raiz do divisor pelo número es- crito abaixo do segundo coeficiente e adicionamos o resultado encontrado com o terceiro coeficiente, escre- vendo o número encontrado abaixo e assim sucessiva- mente: 2 2 x + 2 −5 −1 2 · (−1) + 3 3 1 −4 ● O último número encontrado é o resto da divisão: 2 2 + 2 −5 −1 2 · 1 + (−4) 3 1 −4 −2 x Temos, então: Q(x) = 2x2 - x + 1 e R(x) = -2. 9 Exemplo: Determine a e b para que o polinômio x3 − ax2 + bx − 10 seja divisível por (x + 2)(x − 1). Solução: Aqui, aplicaremos o dispositivo duas vezes: 1 −a b −10 −2 1 −2 −a 4 + 2a + b −18 − 4a − 2b = 0 1 1 −1 −a a + b + 3 = 0 4 2 18 3 4 2 18 3 6 3 a b a b a b a b a b + = − + = −    ⇒ + = − = − −    ⇒ = − = e . PG19LP424SDM0_MIOLO_PVE19_4_MAT_LP.indb 166 27/05/2019 16:46:43
  • 9. Frente A | Livro 167 7 PVE19_7_MAT_A_25 Resolvidos 1. (ESPM) O resto da divisão do polinômio x x 5 2 3 1 − + pelo polinômio x2 1 − é a) x - 1. b) x + 2. c) 2x - 1. d) x + 1. e) x - 2. 9 Solução: E Dividindo x x 5 2 3 1 − + por x2 1 − , obtemos x x x x x x x x x x x x x x x 5 2 2 5 3 3 3 2 3 2 2 3 1 1 3 3 1 3 1 3 3 2 − + − − + + − − + − + − + + − − Portanto, o resto é x − 2. 2. (UEG) A divisão do polinômio x x x 3 2 2 5 6 + − − por x x + ( )( ) 1 2 − é igual a a) x - 3. b) x + 3. c) x - 6. d) x + 6. 9 Solução: B Aplicando o dispositivo prático de Briot-Ruffini obtemos: 1 2 5 6 1 1 1 6 0 2 1 3 0 - - - - Logo, x x x x x x 3 2 2 5 6 1 2 3 + − − = + − + ( )( )( ) e, portanto, a divisão do polinômio x x x 3 2 2 5 6 + − − por ( )( ) x x + − 1 2 é igual a x +3. 3. (Ibmec) Se o resto da divisão do polinômio P(x) = x3 + ax + b pelo polinômio Q(x) = x2 + x + 2 é igual a 4, então podemos afirmar que a + b vale a) 2. b) -2. c) 3. d) -3. e) 4. 9 Solução: C a – 1 = 0. Logo, a = 1. b + 2 = 4. Logo, b = 2. x x ax b x x x x x x x a x b x x a x b 3 2 2 3 2 2 2 0 2 2 1 0 2 2 1 + + + + + − − − − − + − + + + − + + ( ) ( ) ( 2 2) ⇒      Portanto, a + b = 3. Praticando 1. Responda aos itens a seguir. a) Para que a divisão do polinômio p(x) = x5 - 2x4 - x + k por q(x) = x - 1 deixe resto zero, o valor de k deve ser igual a quanto? b) O resto da divisão do polinômio p(x) = x3 + 2x2 - 3x + k por x + 1 é igual a 3. Encontre o valor de k. 2. Responda aos itens a seguir. I. (UFMG) O quociente da divisão de P(x) = 4x4 - 4x3 + x - 1 por Q(x) = 4x3 +1 é a) x - 5. b) x - 1. c) x + 5. d) 4x - 5. e) 4x + 8. II. (UFRGS) A divisão de P(x) por x2 + 1 tem quociente x - 2 e resto 1. O polinômio P(x) é: a) x2 + x - 1. b) x2 + x + 1. c) x2 + x. d) x3 - 2x2 + x - 2. e) x3 - 2x2 + x - 1. 3. (UFSJ) Considere os polinômios: p x x x x x r x x e q x p x r x ( )= + − − + ( )= + ( )= ( ) ( ) 4 3 2 3 2 2 12 2 , . Sobre as raízes da equação q(x) = 0, é correto afirmar que a) a soma de todas as raízes é igual a -1. b) duas das raízes são inteiras. c) duas das raízes são números complexos, um localizado no 1.º quadrante e outro localizado no 3.º quadrante do plano de Argand-Gauss. d) a soma das raízes inteiras é 2. PG19LP424SDM0_MIOLO_PVE19_4_MAT_LP.indb 167 27/05/2019 16:46:58
  • 10. Matemática 168 PVE19_7_MAT_A_25 Desenvolvendo Habilidades 1. C5:H21 (FMP-2016) Seja f:  →  a função polinomial definida por f(x) = x4 - 3x3 + 3x - 9. O fato de x = 3 ser um zero da função f é equivalente ao fato de o polinômio x4 - 3x3 + 3x - 9 ser divisível por a) x2 - 9. b) x + 3. c) 3. d) x - 3. e) x. 2. C5:H21 (Unicamp-2017) Considere o polinômio p(x) = xn + xm +1, em que n > m ≥ 1. Se o resto da divisão de p(x) por x + 1 é igual a 3, então a) n é par e m é par. b) n é ímpar e m é ímpar. c) n é par e m é ímpar. d) n é ímpar e m é par. 3. C5:H21(Unesp-2014)OpolinômioP(x)=a·x3 +2·x+bédivisívelpor x - 2 e, quando divisível por x + 3, deixa resto -45. Nessas condições, os valores de a e b, respectivamente, são a) 1 e 4. b) 1 e 12. c) -1 e 12. d) 2 e 16. e) 1 e -12. 4. C5:H21 (CEFET-MG-2016 - adap.) Se uma das raízes do polinômio P(x) = x4  - 8x2  + ax + b é 2 e P(1) = 9, então o valor de a5 - 4b é a) -64. b) -28. c) 16. d) 24. e) -16. 5. C5:H21(UFRGS-2012)Se2éraizdupladopolinômiop(x) = 2x4  - 7x3  + + 3x2 + 8x - 4, então a soma das outras raízes é a) -1. b) -0,5. c) 0. d) 0,5. e) 1. 6. C5:H21 (Unicamp-2016 - adap.) Considere o polinômio cúbico P(x) = x3  + x2  - ax - 3, em que a é um número real. Sabendo que r e −r são raízes reais de p(x), podemos afirmar que p(1) é igual a a) 3. b) 1. c) -2. d) -4. e) 0. 7. C5:H21 (UECE-2017) O resto da divisão de (264 + 1) por (232 + 1) é igual a a) 1. b) 0. c) 4. d) 2. e) 3. 8. C5:H21 (UEG-2016) Na divisão do polinômio 6x4 - 2x3 - 8x2 + 10x - 2 pelo divisor x2 + 3x - 2, o resto multiplicado por 2 é a) -222x2 + 252. b) 444x2 + 252. c) -444x + 252. d) 222x + 252. e) -444x2 - 252. 9. C5:H21 (UECE-2016) O resto da divisão de (x2 + x + 1)2 por x2 - x +1 é a) 4x. b) 4(x - 1). c) 4(x - 2). d) 4(x - 3). e) 4(x - 4). 10. C5:H21(FGV-2016)UmdosfatoresdopolinômioP(x)=x3 +2x2 -5x -6 é (x + 3). Outro fator desse polinômio é a) (x + 8). b) (x - 5). c) (x + 4). d) (x - 1). e) (x + 1). Complementares 1. (UERN-2013) O produto entre o maior e o menor dos coeficientes do quociente da divisão de P(x) = 6x5 + 3x4 + 5x3 - 2x2 - 4x + 5 por D x x x ( ) = − 3 2 3 é a) 3. b) 4. c) -2. d) -5. 2. (UEPB-2013) Os valores de m e n para os quais a expressão 5 8 2 4 2 2 x x mx n x + + + + seja um polinômio são, respectivamente: a) 2 e -4. b) 0 e -2. c) 0 e -4. d) 2 e 4. e) 8 e -4. 3. (ESPM-2014) O trinômio x ax b 2 + + é divisível por x + 2 e por x - 1. O valor de a - b é a) 0. b) 1. c) 2. d) 3. e) 4. 4. (UERN-2012)Ovalordenparaqueadivisãodopolinômio p(x) = 2x3  + +5x2 +x+17pord(x)=2x2 +nx+4tenharestoiguala5éumnúmero a) menor que -6. b) negativo e maior que -4. c) positivo e menor que 5. d) par e maior que 11. 5. (UEL) O polinômio p x x x ax a ( )= + − − 3 2 3 4 é divisível pelo polinô- mio q x x x ( )= − − 2 4 . Qual o valor de a? a) a = −2 b) a = −1 c) a = 0 d) a = 1 e) a = 2 6. (IFAL-2011) Dividindo o polinômio p(x) pelo polinômio ( )( )( ) x x x − − − 2 4 5 obtém-se resto x + 3. Se os restos das divisões de p(x) por x x − − 2 4 , e x −5 são, respectivamente, os números A, B e C, então ABC vale a) 100. b) 180. c) 200. d) 280. e) 360. 7. (EsPCEx/AMAN-2015) O polinômio f x x x x ( ) , = − + + 5 3 2 1 quando dividido por q x x x ( ) = − + 3 3 2, deixa resto r(x). Sabendo disso, o valor numérico de r(-1) é a) -10. b) -4. c) 0. d) 4. e) 10. 8. (EPCAr/AFA-2011) Sobre o polinômio A(x) expresso pelo determi- nante da matriz x x x x 1 1 1 2 1 −           , é incorreto afirmar que a) não possui raízes comuns com B x x ( )= − 2 1. b) não possui raízes imaginárias. c) a soma de suas raízes é igual a uma de suas raízes. d) é divisível por P x x ( )= +2. 9. (UEPG) Com base nas assertivas a seguir, assinale o que for correto. (1) Se P(x) = (2p + q - 1)x3 + (p + q)x é um polinômio identica- mente nulo então p - q = 2. (2) Os polinômios P(x) = (x + a)2 - (x + a)(x - b) e Q(x) = 2x - 3 são idênticos. Então a e b valem, respectivamente, − 3 2 7 2 e . (3) Os polinômios P(x) = 4x3 + ax2 - 3x; Q(x) = mx2 + nx e R(x) = 2x -1 são tais que P(x) = Q(x) · R(x). Então a + m + n = 9. (4) Se f e g são polinômios de grau n então os graus de f + g e f·g são, respectivamente, 2n e n2 . (5) O polinômio Q(x) = (x - 1)(x - 2)(x - c)(x - d) é divisível por R(x) = x2  - 7x + 12. Então c + d = 7. Soma ( ) 10. (Fuvest) Considere o polinômio não nulo P(x) = a0 + a1x + a2x2 +... + anxn emquea0,a1,a2,...,an estãoemprogressãogeométricaderazãoq≠0. a) Calcule P q 1       . b) Mostre que, para n par, o polinômio P(x) não tem raiz real. GABARITO ONLINE 1. Faça o download do aplicativo SAE Questões ou qualquer aplicativo de leitura QR Code. 2. Abra o aplicativo e aponte para o QR Code ao lado. 3. O gabarito deste módulo será exibido em sua tela. GABARITO E SOLUCIONÁRIO ONLINE 1. Faça o download do aplicativo SAE Questões ou qualquer aplicativo de leitura QR Code. 2. Abra o aplicativo e aponte para um dos QR Codes ao lado para acessar o gabarito ou o solucionário deste módulo. PG19LP424SDM0_MIOLO_PVE19_4_MAT_LP.indb 168 27/05/2019 16:47:07
  • 11. Frente A | Livro Tupungato/Shutterstock Frente A 169 PVE19_7_MAT_A_26 7 Denominamos equação algébrica ou equação polino- mial de grau n toda equação da forma: anxn + an–1 xn–1 + a 2x 2 + ... + a1x + a0 = 0 em que a0, a1, ..., an são chamados coeficientes e podem ser números reais ou complexos, com n e an 0. O conjunto solução ou conjunto verdade de uma equa- ção algébrica, no conjunto universo U, é o subconjunto de U que contém as raízes da equação. Duas equações são ditas equivalentes em U quando apresentam o mesmo conjunto solução nesse domínio. Teorema fundamental da Álgebra Carl Friedrich Gauss (1777-1855) demonstrou satisfa- toriamente pela primeira vez o teorema fundamental da Álgebra, em 1798. Nicku/Shutterstock O teorema afirma que: T da e ua i ia de rau 1 ad ite a e uma raiz complexa. Segundo Gauss, são consequências desse resultado: 1) toda equação polinomial de grau n admite exatamente n raízes complexas; 2) todo polinômio P(x) = anxn + a 1x 1 + a 2x 2 + ... + + a1x + a0 de grau n pode ser colocado na forma fatorada: P(x) = an · (x - r1) · (x - r2) · ... · (x - rn), em que r1, r2, ..., rn são as raízes de P(x); 3) se um polinômio de grau n possuir mais de n raízes, então ele é identicamente nulo. 9 Exemplo: Verificar que 1 é raiz de P(x), em que P(x) = x3 - 3x2 + 4x - 2, obter as outras raízes e obter a forma fatorada de P(x). Solução: Podemos aplicar diretamente o dispositivo de Briot-Ruffini: 1 −3 4 −2 1 1 −2 2 0 Como o resto da divisão por x -1 é 0, então 1 é raiz de P(x). O quociente é q(x) = x2 - 2x + 2, cujas raízes são 1 i. Como as raízes são 1, 1+ i e 1 - i, tem-se que a forma fatorada procurada é P(x) = (x - 1)⋅(x - 1 - i)⋅(x -1 + i). Multiplicidade de uma raiz Dizemos que r é raiz de multiplicidade m (m ≥ 1) da equa- ção P(x) = 0 se, e somente se, P(x) = (x - r)m ⋅ Q(x) e Q(r) 0, ou seja, r é raiz de multiplicidade m de P(x) = 0 quando o po- linômio P é divisível por (x - r)m e não é divisível por (x - r)m+1 . Quando m = 1, dizemos que r é uma raiz simples; quando m = 2, dupla; quando m = 3, tripla; e assim por diante. 9 Exemplos: 1) erificar qual a multiplicidade da raiz −3 na equa o x4 + 6x3 + 11x2 + 12x + 18 = 0. Solução: Para determinar a multiplicidade da raiz –3, dividimos o poli- nômio x4 + 6x3 + 11x2 + 12x + 18 = 0 sucessivas vezes por 3. 1 6 11 12 18 −3 1 3 2 6 0 −3 1 0 2 0 −3 1 −3 11 Observe que as divisões são exatas nas duas primeiras opera- ções do dispositivo, isto é, a equação possui duas raízes reais iguais a 3 e uma raiz diferente de 3. Fazendo a decomposição de P(x): P(x) = x4 + 6x + 11x + 12x + 18 P(x) = (x + 3) · (x + 3x + 2x + 6) P(x) = (x + 3) · (x + 3) · (x + 2) P(x) = (x + 3) · (x + 2) Portanto, –3 é a raiz de multiplicidade 2 ou a raiz dupla da equação. Teorema fundamental da Álgebra • Multiplicidade de uma raiz • Pesquisa de raízes Equações Polinômicas ou Algébricas Equações algébricas PG19LP424SDM0_MIOLO_PVE19_4_MAT_LP.indb 169 27/05/2019 16:47:16
  • 12. Matemática 170 PVE19_7_MAT_A_26 2) Qual o grau de uma equa o polinomial P(x) = 0 cujas raízes são 3, 2, -1 com multiplicidades , 6 e 10, respectivamente Solução: P(x) = k ⋅ (x - 3)7 ⋅ (x - 2)6 ⋅ (x + 1)10 , com k * ⇒ gr(P) = 23 (pois + 6 + 10 = 23). Pesquisa de raízes Raízes racionais de equações com coeficientes inteiros Se r = p q , com p e q inteiros e primos entre si, é uma raiz racional da equação de coeficientes inteiros dada por anxn + a 1x 1 + a 2x 2 + ... + a1x + a0 = 0 com an 0, então p é divisor de a0 e q é divisor de an. 9 Exemplo: Verificar se a equação 2x3 + x2 + x − 1 = 0 admite raízes racionais. Solução: As possíveis raízes da equação são dadas por: p q x = ⇒ p 1, −1 e q 1, −1, 2, −2 p q x = ⇒ p q 1 1 1 2 1 2 , , , − −       p(x) = 2x3 + x2 + x − 1 p(1) = 3 p(−1) = −3 p 1 2       = 0; p 1 2       = − 3 2 Logo, a única raiz racional da equação é 1 2 . Raízes complexas de equações com coeficientes reais Se um complexo z = a + bi, com a e b e b 0, é raiz de uma equação algébrica de coeficientes reais, então o con- jugado z = a – bi também é raiz da equação. Como consequências desse teorema: 1) toda equação algébrica de coeficientes reais e grau ím- par admite pelo menos uma raiz real; 2) Teorema do conjugado: se o complexo z é raiz de mul- tiplicidade m de uma equação algébrica de coeficientes reais, então o conjugado z também é raiz de multiplici- dade m da equação. 9 Exemplo: Resolver a equação x4 + 4x3 – 17x2 + 26x 14 = 0 sabendo que 1 – i é uma de suas raízes. Solução: Como se trata de uma equação de coeficientes reais, se 1 – i é raiz, então 1 + i também é raiz. Aplicando o algoritmo de Briot-Ruffini: 1 4 −1 26 −14 1 − i 1 − i −13 − 6i 7 + 7i 0 1 + i 1 6 − 0 x2 + 6x − = 0 ⇒ raízes: x = 1 ou x = − = 1, − , 1+ i, 1− i . Resolvidos 1. Sabendo que 3 é uma raiz do polinômio p(x) = x4 – 3x2 + kx – 9, determine o valor de k. 9 Solução: Se 3 é uma raiz de p(x), tem-se que p(3) = 0. Logo, 34 – 3(3)2 + 3k – 9 = 0 81 2 + 3 = 0 3k = 45 k = 15 2. (UFRGS-2014) Considere os polinômios p x x ( ) 3 e q x x x ( ) . = + 2 O número de soluções da equação p x q x ( ) ( ), no conjunto dos números reais, é a) 0. b) 1. c) 2. d) 3. e) 4. 9 Solução: D p x q x x x x x x x ( ) ( ) ( ) = = + ⋅ − − = 3 2 2 1 0 Tem-se, então, duas equações: x =0 (já resolvida) ou x x 2 1 0 − − = (com discriminante ∆ = 5, portan- to, com duas raízes distintas). Assim, o número de soluções da equação p x q x ( ) ( ) = é 3. 3. (UFPE) Determine o polinômio com coeficientes reais p x ax bx cx ( ) = + + 3 2 , p x ax bx cx ( ) = + + 3 2 , tal que p x p x x + ( ) − ( ) = 1 6 2 , e indique a b c 2 2 2 + + . 9 Solução: Temos: p x a x b x c x ax a b x a b c x a b ( ) ( ) ( ) ( ) ( ) ( ) + = + + + + + = + + + + + + + 1 1 1 1 3 2 3 2 3 2 3 2 + +c. Logo, p x p x x ax a b x a b c x ( ) ( ) ( ) . + − = ⇔ + + + + + = 1 6 3 3 2 6 2 2 2 Para que a identidade se verifique, devemos ter: 3 6 3 2 0 0 2 3 1 a a b a b c a b c = + = + + =      ⇔ = = − =      Portanto, p(x) =2x3 - 3x2 +x e a2 + b2 + c2 = 22 + (-3)2 + 12 = 14. Praticando 1. Determine quais são as raízes da equação algébrica: x x x x −       +       − ( ) − ( )= 1 2 2 7 2 21 0 5 3 0 2 e determine a multiplicidade de cada uma delas. PG19LP424SDM0_MIOLO_PVE19_4_MAT_LP.indb 170 27/05/2019 16:47:34
  • 13. Frente A | Livro 171 7 PVE19_7_MAT_A_26 2. Uma equação algébrica possui como raízes i2 , 7i, i + 14 e –6 – i. Qual é o menor grau possível dessa equação algébrica? 3. (Unesp) Sabe-se que, na equação x x x 3 2 4 6 0 + + − = , uma das raízes é igual à soma das outras duas. O conjunto solução (S) desta equação é a) S = {–3, –2, –1}. b) S = {–3, –2, +1}. c) S = {+1, +2, +3}. d) S = {–1, +2, +3}. e) S = {–2, +1, +3}. Desenvolvendo Habilidades 1. C5:H21 (UECE-2017–adap.) Sejam P(x) = x5 + x4 + x3 + x2 + x +1 um polinômio e M o conjunto dos números reais k tais que P(k) = 0. O número de elementos de M é a) 1. b) 2. c) 3. d) 4. e) 5. 2. C5:H21 (UECE-2016) O polinômio de menor grau, com coeficientes inteiros, divisível por 2x – 3, que admite x = 2i como uma das raízes e P(0) = –12 é (Dado: i é o número complexo cujo quadrado é igual a –1.) a) P(x) = 2x3 – 3x2 – 8x – 12. b) P(x) = 2x3 + 3x2 – 8x – 12. c) P(x) = –2x3 – 3x2 – 8x – 12. d) P(x) = 2x3 – 3x2 + 8x – 12. 3. C5:H21 (FGV-2013) A equação x− = 4 16 tem a) duas raízes reais e duas raízes imaginárias conjugadas. b) pelo menos duas raízes iguais. c) uma única raiz imaginária. d) quatro raízes reais. e) quatro raízes cujo produto é − 1 4 . 4. C5:H21 (UFRGS-2012) Considere o polinômio: p(x) = x4 + 2x3 – 7x2 – 8x + 12. Se p(2) = 0 e p(–2) = 0, então as raízes do polinômio p(x) são a) –2, 0, 1 e 2. b) –2, –1, 2 e 3. c) –2, –1, 1 e 2. d) –2, –1, 0 e 2. e) –3, –2, 1 e 2. 5. C5:H21 (IFCE-2016) Para certos valores reais de p e q, a equação (x – 1) · (x2 + p · x + q) = 0 possui apenas uma solução real. Nessas condições, é verdade que a) q < 0 < p. b) p < 0 < q. c) 0 < p < q. d) p < q < 0. e) 0 = q < p. 6. C5:H21 (PUC-Rio-2016) Considere o polinômio p(x) = x2 + bx + 3 e assinale a alternativa correta. a) O polinômio tem pelo menos uma raiz real para todo b . b) O polinômio tem exatamente uma raiz real para b = 12. c) O polinômio tem infinitas raízes reais para b = 0. d) O polinômio não admite raiz real para b = + 1 1 3 . e) O polinômio tem exatamente três raízes reais para b = . 7. C5:H21 (UFSM-2015) Para avaliar as vendas em 2013, o setor de planejamento de uma empresa utilizou a função polinomial N(t) = t3 – 21t2 + 126t + 304 em que N representa o número de tablets vendidos no mês t, com t = 1 correspondendo a janeiro, t = 2 correspondendo a fevereiro e assim por diante. De acordo com os dados, o número de tablets vendidos foi igual a 480 nos meses de a) fevereiro, julho e novembro. b) fevereiro, agosto e novembro. c) fevereiro, agosto e dezembro. d) março, agosto e dezembro. e) março, setembro e dezembro. 8. C5:H21 (Mackenzie-2015) Seja P(x) = 2x3 – 11x2 + 17x – 6 um poli- nômio de 3.º grau e 2x – 1 um de seus fatores. A média aritmética das raízes de P(x) é a) 7 2 b) 8 2 c) 9 2 d) 10 2 e) 11 6 9. C5:H21(EsPCEx/AMAN-2016)SendoRamaiordasraízesdaequação 11 6 4 2 x x x + − = , então o valor de 2R – 2 é a) 2. b) 4. c) 6. d) 8. e) 10. PG19LP424SDM0_MIOLO_PVE19_4_MAT_LP.indb 171 27/05/2019 16:47:48
  • 14. Matemática 172 PVE19_7_MAT_A_26 Complementares 1. (UEPB-2013) O produto entre as raízes da equação x4 +3x2 +2=0 é: a) 2. b) 1. c) 2. d) –1. e) 2i. 2. (UECE-2015) Se os números 2 + i, 2 – i, 1 + 2i, 1 – 2i e 0,5 são as raízes da equação 2x5 + px4 + 42x3 – 78x2 + 80x + q = 0, então o valor de p + q + pq é a) 287. b) 278. c) 297. d) 279. 3. (UECE-2009) Se os polinômios P x x n m nx x ( ) 2 1 1 1 2 e Q(x) = x3 – 4x2 + x + 4 são idênticos, então o valor de m n é: a) 2. b) 3. c) 4. d) 5. 4. (IME-2013)Ospolinômios P x x ax ( )= + + 3 2 18 e Q 12 3 x x bx ( )= + + possuem duas raízes comuns. Sabendo que ae bsão números reais, pode-se afirmar que satisfazem à equação a) a = b. b) 2a = b. c) a = 2b. d) 2a = 3b. e) 3a = 2b. 5. (ACAFE-2014) Sobre equações algébricas, considere as seguintes afirmações: I. Na equação 2 9 10 3 0 3 2 x x x − + − = , sabendo que a, b e c são raízes reais, o valor de log1 9 1 1 1 ab bc ac + +       é − 1 2 . II. Um recipiente cônico tem 8 dm de altura. Seu espaço interior é ocupado por uma esfera cujo raio tem a metade da medida do raio do cone e por 60 dm3 de água. Então, os valores inteiros da medida do raio do recipiente cônico e do raio da esfera (em dm) são números múltiplos de 3. III. A equação x x x 3 2 3 4 12 0 − − + = tem raízes reais a, b e c. Então, o determinante da matriz 0 0 0 c b b a a c         vale –12. Assinale a alternativa correta. a) Todas as afirmações estão corretas. b) As afirmações I e III estão corretas. c) As afirmações I e II estão corretas. d) Apenas a afirmação II está correta. 6. (Mackenzie-2012) As raízes da equação x 9x 23x 15 3 2 − + − = 0, co- locadas em ordem crescente, são os três primeiros termos de uma progressão aritmética cuja soma dos 20 primeiros termos é a) 500. b) 480. c) 260. d) 400. e) 350. 7. (Fuvest-2014) Os coeficientes a, b e c do polinômio: p x x ax bx c ( ) = + + + 3 2 são reais. Sabendo que –1 e 1 + i, com , são raízes da equação p x ( ) 0 e que o resto da divisão de p(x) por (x – 1) é 8, determine a) o valor de ; b) o quociente de p(x) por (x + 1). i é a unidade imaginária, i2 = –1 8. (UFSC-2011) Assinale a(s) proposição(ões) correta(s). (1) Se 3 5 n , então log . 5 225 2 2 = + n n (2) Os valores reais de x que satisfazem à equação 4 4 5 2 x x + = ⋅ pertencem ao intervalo (2,4]. (3) Suponha que “Chevalier de Mére”, um jogador francês do século XVII, que ganhava a vida apostando seu dinheiro em jogos de dados, decidiu apostar que vai sair um“3”no lança- mento de um dado perfeito de seis faces numeradas de 1 a 6. Com relação a esse experimento, há dois resultados possí- veis: ou sai “3” e Chevalier ganha, ou não sai “3” e ele perde. Cada um destes resultados – “sai um 3” ou “não sai um 3” – tem a mesma probabilidade de ocorrer. (4) Para que a função P x x px ( )= + 2 seja divisível por 4x – 1 é necessário que p seja igual a 1 4 . (5) Sea,becsãoraízesreaisdaequação x x x 3 2 20 125 250 0 − + − = , então o valor de log 1 1 1 a b c + +       é nulo. ( ) Se“A”é o número de arranjos de 6 elementos tomados 2 a 2; “B”é o número de permutações de 5 elementos e “C”é o nú- mero de combinações de 5 elementos tomados 3 a 3, então A B C + − =140. Soma ( ) GABARITO ONLINE 1. Faça o download do aplicativo SAE Questões ou qualquer aplicativo de leitura QR Code. 2. Abra o aplicativo e aponte para o QR Code ao lado. 3. O gabarito deste módulo será exibido em sua tela. GABARITO E SOLUCIONÁRIO ONLINE 1. Faça o download do aplicativo SAE Questões ou qualquer aplicativo de leitura QR Code. 2. Abra o aplicativo e aponte para um dos QR Codes ao lado para acessar o gabarito ou o solucionário deste módulo. PG19LP424SDM0_MIOLO_PVE19_4_MAT_LP.indb 172 27/05/2019 16:47:57
  • 15. Frente A | Livro SAE DIGITAL S/A Frente A 173 PVE19_7_MAT_A_27 7 VERSÃO 3.3 Relações de Girard As relações de Girard são úteis na resolução de equações polinomiais, pois elas relacionam as raízes e os coeficientes dessas equações. Por meio dessas relações é possível estabe- lecer um sistema de equações que permite resolver a equa- ção inicial, cuja resolução geralmente é mais simples. Equações do segundo grau Se a a e ua a bri a a b 0, a 0. A divi- dirmos a equação por a, obtemos x b a x c a 2 0 + + = . Por outro lado, sendo x1 e x2 as raízes ax² + bx + c = 0, pode- mos escrever ax bx c a x x x x x x 2 2 1 2 1 2 0 + + = − + ( ) + ⋅     = . Dessa forma, ao igualarmos as duas equações: x b a x c a a x x x x x x 2 2 1 2 1 2 + + = − + ( ) + ⋅     = 0. De onde obtemos, portanto, x x b a x x c a 1 2 1 2 + = − ⋅ = Equações do terceiro grau Seja a e ua a bri a a b d 0, a 0. Ao dividirmos a equação por a, obtemos x b a x c a x d a 3 2 0 + + + = . Por outro lado, sendo x1, x2 e x3 as raízes de ax³ + bx² + cx + + d = 0, podemos escrever: ax bx cx d a x x x x x x x x x x x x x 3 2 3 1 2 3 2 1 2 1 3 2 3 1 2 + + + = − + + ( ) + ⋅ + ⋅ + ⋅     − ⋅ ⋅x x3 0 = x x x x x x x x x x x x 1 2 3 2 1 2 1 3 2 3 1 2 − + + ( ) + ⋅ + ⋅ + ⋅   − ⋅ ⋅x x3 0 = . Dessa forma, ao igualarmos as duas equações, x b a x c a x d a a x x x x x x x x x x x x x 3 2 3 1 2 3 2 1 2 1 3 2 3 1 + + + = − + + ( ) + ⋅ + ⋅ + ⋅     − ⋅ 2 2 3 ⋅x x x x x x x x x x x x x 1 2 3 2 1 2 1 3 2 3 1 − + + ( ) + ⋅ + ⋅ + ⋅   − ⋅ 2 2 3 ⋅x = 0. De onde obtemos, portanto, x x x b a x x x x x x c a x x x d a 1 2 3 1 2 1 3 2 3 1 2 3 + + = − ⋅ + ⋅ + ⋅ = ⋅ ⋅ = − Equações do quarto grau Caso a equação seja de 4.º grau e x1, x2, x3 e x4 sejam as raízes da equação algébrica ax4 + bx³ + cx² + dx + e = 0, com a   0, te : x1 + x2 + x3 + x4 = - b a x1x2 + x1x3 + x1x4 + x2x3 + x2x4 + x3x4 = c a x1 x2 x3 + x1 x2 x4 + x1 x3 x4 + x2 x3 x4 = - d a x1 · x2 · x3 · x4 = e a 9 Exemplo: Se x1 , x2 , x3 e x4 são raízes da equação x4 - 2x3 + 3x2 - x + = 0, calcule o valor da expressão E = 1 1 1 1 1 2 3 4 x x x x + + + . Solução: E x x x x x x x x x x x x x x x x = + + + 1 2 3 1 2 4 1 3 4 2 3 4 1 2 3 4 Perceba que a = 1, b = -2, c = 3, d = - e e = . nt o, note que o numerador i ual a − = − − = d a ( ) 5 1 5 e que o denominador vale e a 7 1 7. Lo o, E 5 7 . Teorema de Bolzano Se um polinômio P(x) apresenta valores P(a) e P(b), com a < b, tais que P(a) · P(b) < 0 (isto é, têm sinais contrários), então a equação admite um número ímpar (pelo menos uma) de raízes reais entre a e b. y P(a) P(b) 0 a b x raiz 9 Exemplo: Seja P(x) = x3 − 3x2 − x + 3. Solução: onsiderando, por exemplo, a = 0 e b = 2, em que a b, temos: P(0) = 3 e P(2) = 23 − 3 · 22 − 2 + 3 = −3. Relações de Girard • Teorema de Bolzano • m.d.c. e m.m.c. de polinômios Relações de Girard e Teorema de Bolzano Equações algébricas: relações de Girard PG19LP424SDM0_MIOLO_PVE19_4_MAT_LP.indb 173 27/05/2019 16:48:07
  • 16. Matemática 174 PVE19_7_MAT_A_27 Pelo eorema de Bolzano, existe pelo menos uma raiz entre 0 e 2. raiz y x 4 3 2 1 0 –1 –2 –2 0 2 m.d.c. e m.m.c. de polinômios O máximo divisor comum (m.d.c.) entre polinômios é o polinômio unitário (coeficiente dominante 1) formado pelos fatores comuns aos polinômios elevados aos seus menores expoentes. O m.d.c. é o polinômio de maior grau que divide todos aqueles. As raízes comuns aos polinômios também são raízes de seu m.d.c., com a menor multiplicidade. Se o m.d.c. de dois polinômios é 1, diz-se que eles são pri- mos entre si. Quando os polinômios não estão na forma fatorada, o m.d.c. pode ser obtido pelo método das divisões sucessivas. 9 Exemplo: Obten a o m.d.c. dos polin mios: p(x) = x4 - 3x3 + 3x2 - 3x + 2 e q(x) = x2 - 4x + 3. Solução: x2 + x + 4 1 10 x - 3 10 quocientes x4 - 3x3 + 3x2 - 3x + 2 x2 - 4x + 3 10x - 10 10x - 10 0 restos Lo o, m.d.c.(p, q) = 1 10 (10x - 10) = x - 1. Vale notar que a divisão por 10 se faz necessária para que o m.d.c. seja um polinômio unitário. O mínimo múltiplo comum (m.m.c.) entre polinômios é o polinômio unitário formado por todos os fatores que apare- cem nos polinômios, comuns ou não, elevados ao seu maior expoente. O m.m.c. é o polinômio de menor grau múltiplo de todos aqueles. Todas as raízes dos polinômios são raízes do seu m.m.c. 9 Exemplo: ncontre o m.d.c. e o m.m.c. dos polin mios P(x) = x(x - 1)2 (x - 2)3 e Q(x) = x3 (x - 1)(x - 3)2 . Solução: m.d.c. (P, Q) = x(x - 1) m.m.c. (P, Q) = x3 (x - 1)2 (x - 2)3 (x - 3)2 Resolvidos 1. O polinômio p x x x x ( )= + − + 5 3 4 1 3 2 possui pelo menos uma raiz no intervalo [–3, 3]? 9 Solução: im. Observamos que p(3) = · (3) + 3 · (3) - 4 · 3 + 1 = 1 1 0 e que p(-3) = · (-3) + 3 · (-3) - 4 · (-3) + 1 = - . Lo o, p(3) · p(-3) 0. Pelo eorema de Bolzano, concluímos que existe raiz no intervalo -3,3 . 2. Sabendo que a, b e c são as raízes da equação x3 - x2 - 1 = 0, forme uma nova equação cujas raízes sejam os números b + c, c + a e a + b. 9 Solução: a + b + c = − − ( ) 1 1 = 1 ⇒ b + c = 1 − a c + a = 1 − b a + b = 1 − c ⇒ = 1 − x ⇔ x = 1 − (1 − )3 −(1 − )2 − 1 = 0 ⇒ y3 − 2 2 + y + 1 = 0 y3 − 2 2 + y + 1 = 0 3. (UERJ)Umciclistaeumcorredorcomeçam,juntos,umacompetição. A curva a seguir, cuja equação é e = t3 + at2 + bt + c, representa a posição em metros do ciclista, em função do tempo t, em segundos, em que a, b, e c são números reais fixos. 3 t(s) 0 e(m) No instante em que o ciclista parte da posição zero, o corredor inicia um movimento, descrito pela equação e = 4t, na mesma pista e no mesmo sentido. Determine a posição mais afastada da origem na qual o ciclista e o corredor voltam a se encontrar. 9 Solução: Por meio da análise do ráfico e da equa o, verificamos que existem tr s raízes reais: 0 raiz simples e 3 raiz dupla. nt o, e = t · (t − 3)2 ⇒ e = t3 − 6t2 + 9t. Para determinar os instantes dos encontros: t3 − 6t2 + 9t = 4t ⇔ t3 - 6t2 + 5t = 0 ⇔ t · (t2 - 6t + ) = 0 ⇒ t = 0s t = 1s e t = s. Posi o dos encontros: 0 m 4 m e 20 m. Posi o mais afastada = 20 m. Praticando 1. Mostre que o polinômio p x x x x x ( )= − + + − 4 3 2 5 5 5 6 possui pelo menos uma raiz entre os valores 0 e 4. 2. Uma das raízes do polinômio p x x x x ( )= + − − 3 2 4 4 é -2. Qual é o produto entre as outras duas raízes? PG19LP424SDM0_MIOLO_PVE19_4_MAT_LP.indb 174 27/05/2019 16:48:23
  • 17. Frente A | Livro 175 7 PVE19_7_MAT_A_27 3. Calculeom.m.c.eom.d.c.entreospolinômios p x x x x x e q x x x ( )= − + + − ( )= + − 4 3 2 2 5 5 5 6 2 15 p x x x x x e q x x x ( )= − + + − ( )= + − 4 3 2 2 5 5 5 6 2 15 sabendo que 1 e -1 são raízes de p(x). Desenvolvendo Habilidades 1. C5:H21 (UFU-2015) O polinômio de variável real y=p(x)=x3 -ax2 - - 9x + ar2 é representado graficamente conforme ilustra a figura a seguir, em que -r, r e a são constantes reais e encontram-se, nessa ordem, em progressão aritmética (P.A.). y –r r a x (Figura ilustrativa e sem escalas) Nessas condições, o valor de a é um número a) primo. b) ímpar. c) múltiplo de 5. d) divisível por 7. 2. C5:H21 (UEG-2013) Se o coeficiente do termo de maior grau de um polinômiodo4.ºgraué1esuasraízessãox1 =2i,x2 =-2i,x3 =3ex4 =4, então o polinômio em questão é a) x4 - 7x3 + 16x2 - 28x + 48. b) x4 - 2ix3 + 2ix2 + 3x + 4. c) x4 + 16x3 + 4x2 - x + 18. d) x4 - 28x3 + 7x2 + 48x - 28. 3. C5:H21(UDESC-2014)Sejafumpolinômiodegrauquatroquepossui apenas raízes reais, com coeficiente do termo de maior expoente igual à razão q da progressão geométrica formada pelas raízes de f, cuja soma é igual a 15. Sabendo-se que a razão q é igual ao resto da divisão de p(x) = x2 + 1 por g(x) = x - 1, então: a) f(x) = x4 - 15x3 + 70x2 - 120x + 64. b) f(x) = 2x4 + 30x3 + 140x2 + 240x + 128. c) f(x) = -2x4 + 30x3 - 140x2 + 240x - 128. d) f(x) = x4 + 15x3 + 70x2 + 120x + 64. e) f(x) = 2x4 - 30x3 + 140x2 - 240x + 128. 4. C5:H21 (UEG-2010) João gosta de brincar com números e fazer ope- rações com eles. Em determinado momento, ele pensou em três nú- meros naturais e, em relação a esses números, observou o seguinte: ● a soma desses números é 7; ● o produto deles é 8; ● a soma das três parcelas resultantes dos produtos desses núme- ros tomados dois a dois é 14. Assim, os três números pensados por João são raízes da equação a) x3 - 7x2 + 14x - 8 = 0. b) x3 + 7x2 - 14x + 8 = 0. c) x3 - 7x2 - 14x - 8 = 0. d) x3 + 7x2 - 14x - 8 = 0. 5. C5:H21 (FMJ-2012) A montanha russa de um parque de diversões tem um perfil que se encontra esboçado em linha cheia no plano cartesiano da figura. y x A função polinomial P(x), que possui pelo menos uma raiz complexa não real, capaz de representar essa curva completa, é P(x) = Ax5 + +Bx4 + Cx3 + Dx2 + Ex + F, com a) A > 0, F > 0, duas raízes complexas não reais e duas reais, sendo uma de multiplicidade 1 e outra de multiplicidade 2. b) A < 0, F > 0, duas raízes complexas não reais e duas reais, sendo uma de multiplicidade 1 e outra de multiplicidade 2. c) A > 0, F < 0, duas raízes complexas não reais e duas reais, sendo uma de multiplicidade 1 e outra de multiplicidade 2. d) A > 0, F > 0, apenas uma raiz complexa não real e duas reais, sendo uma negativa e outra positiva. e) A > 0, F > 0, apenas uma raiz complexa não real e duas reais, sendo uma negativa e outra nula. 6. C5:H21 (IFAL-2017) Podemos dizer que o polinômio p(x) = x3 - 2x2 - - 5x + 6 a) tem três raízes reais. b) tem duas raízes reais e uma imaginária. c) tem uma raiz real e duas imaginárias. d) não tem raiz real. e) tem duas raízes reais e duas imaginárias. 7. C5:H21 (FGV-2017) A equação algébrica x3 - 7x2 + kx + 216 = 0, em que k é um número real, possui três raízes reais. Sabendo que o quadrado de uma das raízes dessa equação é igual ao produto das outras duas, então o valor de k é igual a a) -64. b) -42. c) -36. d) 18. e) 24. 8. C5:H21 (IFCE-2016) Se S é o conjunto dos números reais x para os quais se verifica a igualdade 2 · (x3 + 1) = 3 · (x2 + x), então é verdade que a) {-1, 1, 2} S. b) {-1, 1} S. c) {1, 2} S. d) {-1, 2} S. e) S = . 9. C5:H21(UFJF-2016)Sabendoque1+iéumadasraízesdopolinômio p(x) = x5 - 2x4 + 2x3 - x2 + 2x - 2, é correto afirmar que: a) o polinômio não possui raízes reais. b) o polinômio possui exatamente duas raízes racionais. c) o polinômio possui exatamente duas raízes distintas. d) o polinômio possui quatro raízes complexas não reais. e) o polinômio possui exatamente quatro raízes distintas. PG19LP424SDM0_MIOLO_PVE19_4_MAT_LP.indb 175 27/05/2019 16:48:35
  • 18. Matemática 176 PVE19_7_MAT_A_27 10. C5:H21 (UNEB-2013 - adap.) Ao desmontar um cubo de Rubik (cubo mágico), uma criança percebeu que ele era formado por vinte e sete cubinhos menores e que dentre esses existiam oito cubinhos com três faces pintadas, doze com apenas duas faces pintadas, seis com apenas uma das faces pintadas e apenas um cubinho não possuía nenhuma das faces pintadas. A tabela a seguir mostra o número de cubinhos, de cada tipo, que podem ser obtidos ao dividir a aresta de um cubo de madeira pintado, em partes iguais. Número de divisões de cada aresta Cubinhos com três faces pintadas Cubinhos com apenas duas faces pintadas Cubinhos com apenas uma face pintada Cubinhos sem nenhuma face pintada Total de cubinhos 3 8 12 6 1 27 4 8 24 24 8 64 5 8 36 54 27 125 6 8 48 96 64 216 7 8 60 150 125 343 . . . . . . . . . . . . . . . . . . n 8 P1(n) P2(n) P3(n) n3 Nessas condições, pode-se afirmar que, em , a soma dos inversos das raízes do polinômio P(x) = P3(x) - P1(x) + P2(x) é igual a a) 1 4 b) 1 2 c) 3 5 d) 2 3 e) 4 5 5. (FGV-2013) A equação 2x3 - 3x2 - 3x + 2 = 0 tem o seguinte conjunto solução: {-1, a, b}. Podemos afirmar que o valor de a2 + b2 é a) 13 4 b) 7 2 c) 15 4 d) 4 e) 17 4 6. (UEPA-2014) Girolamo Cardano (1501-1576) apresentou no livro Ars Magna, demonstrações sobre como resolver equações cúbicas. Ele propôs para equações da forma x3 + px + q = 0 a so- lução x q q p q q p = − + + + − − + 2 4 27 2 4 27 2 3 3 2 3 3 . Sabe-se que Rafael Bombelli (1526-1572) estendeu às ideias de Cardano e encontrou uma das raízes da equação x3 - 15x - 4 = 0, o número 4. Nessas condições, a soma dos inversos das outras raízes dessa equação é a) 4. b) 2. c) 0. d) -2. e) -4. 7. (ITA) Seja P(x) um polinômio de grau 5, com coeficientes reais, admi- tindo 2 e i como raízes. Se P(1) ⋅ P(-1) < 0, então o número de raízes reais de P(x) pertencentes ao intervalo ]-1, 1[ é a) 0. b) 1. c) 2. d) 3. e) 4. 8. (Fuvest) Seja f x ax a x ( ) ( ) = + − + 2 1 1, em que a é um número real diferente de zero. Determine os valores de a para os quais as raízes daequação f x ( ) 0 sãoreaiseonúmerox=3pertenceaointervalo fechado compreendido entre as raízes. GABARITO ONLINE 1. Faça o download do aplicativo SAE Questões ou qualquer aplicativo de leitura QR Code. 2. Abra o aplicativo e aponte para o QR Code ao lado. 3. O gabarito deste módulo será exibido em sua tela. GABARITO E SOLUCIONÁRIO ONLINE 1. Faça o download do aplicativo SAE Questões ou qualquer aplicativo de leitura QR Code. 2. Abra o aplicativo e aponte para um dos QR Codes ao lado para acessar o gabarito ou o solucionário deste módulo. 1. (FMP) Seja o polinômio F(x) = x4 - x3 - 16x2 + 4x + 48. A soma e o produto de suas raízes são, respectivamente, a) 1 e 48. b) 1 e 16. c) 1 e 4. d) -1 e 48. e) -1 e -16. 2. (UFG) Sabe-se que todo polinômio de grau ímpar com coeficien- tes reais admite pelo menos uma raiz real. Dado o polinômio p x m m x x kx ( ) [( )( )] = − + + + + 1 1 1 2 5 2 , com m, k , as condições sobre m e k, para que o polinômio p(x) não admita raiz real, são: a) m = 0 e k < -2. b) m = 1 e k < -2. c) m = 0 e k > 2. d) m = -1 e -2 < k < 2. e) m = 1 e -2 < k < 2. 3. (UEFS) Se a média aritmética das raízes do polinômio p(x) = 2x2 + rx + 5 for 7 e a das raízes de q(x) = 3x2 + sx - 2 for 2 (sendo r e s constantes), então, a média aritmética das raízes do polinômio p(x) + q(x) será a) 4. b) 4,5. c) 5. d) 8,5. e) 9. 4. (IMBEC-2013) O gráfico a seguir representa a função f(x) = x3 + 9x2 + +23x + 15. x y Se os pontos a, b e c são as raízes de f, então 2a + 2b + 2c é igual a a) 21 32 b) 32 43 c) 43 54 d) 54 65 e) 65 76 Complementares PG19LP424SDM0_MIOLO_PVE19_4_MAT_LP.indb 176 27/05/2019 16:48:43
  • 19. Frente A | Livro abeadev/Shutterstock Frente A 177 PVE19_7_MAT_A_28 7 VERSÃO 3.3 Transformações Uma transformação de uma equação algébrica P1(x) = 0 é umaoperaçãocomaqualseobtémumanovaequaçãoP2(y)=0, cujas raízes estejam relacionadas com as raízes da equação inicial por meio de uma relação conhecida: y = f(x). P1(x) = 0 → equação primitiva P2(y) = 0 → equação transformada y = f(x) → relação de transformação Veremos,apartirdeagora,algunstiposdetransformações. Transformação multiplicativa É a transformação em que y = k ⋅ x (k 0). Para obter a equação transformada basta substituir na equação primitiva x = y k . y = k ⋅ x ⇒ x = y k 9 Exemplo: Obter a equação cujas raízes são o dobro das raízes da equação x3 + 5x2 - 7x + 11 = 0. Solução: Aqui, a relação de transformação é y = 2x. Logo, x = y 2 e substi- tuindo na equação primitiva: y y y y y y 2 5 2 7 2 11 0 1 8 5 4 7 2 11 0 3 2 3 2       +       −      + = ⇒ + − + = . Multiplicando por 8, temos y3 +10y2 - 28y + 88 = 0. Esta é a equa- ção transformada. Transformação aditiva É a transformação em que y = x + a (a ). Para obter a equação transformada basta substituir na equação primitiva x = y - a. y = x + a ⇒ a 9 Exemplo: Obter a equação cujas raízes são 2 unidades menores que as raízes de 2x3 - 5x - 2 = 0. Solução: Aqui, a relação de transformação é y = x - 2. Logo, x = y + 2. Substituindo na equação primitiva: 2(y + 2)3 - 5(y + 2) - 2 = 0 ⇒ 2y3 + 12y2 + 19y + 4 = 0. Esta é a equação transformada. Transformada aditiva e divisão de polinômios Dada a equação primitiva P1(x) = anxn + an–1xn–1 + an–2xn–2    ... a1x + a0 = 0, tem-se que a transformada aditiva é dada por: P2(x + a) = Rn ⋅ (x + a)n + Rn–1 ⋅ (x + a)n–1 + + ... + R1 ⋅ (x + a) + R0 = 0 em que R0, R1, ... , Rn são os restos das divisões sucessivas de P1 por x + a, que podem ser facilmente obtidos com o auxílio do algoritmo de Briot-Ruffini. 9 Exemplo: Dada a equação x3 − 2x2 + x + 1 = 0, obtenha a transformada pela relação y = x + 2. Solução: Aplicando Briot-Ruffini: 1 – 2 1 1 – 2 1 – 4 9 – 17 R0 – 2 1 – 6 21 R1 – 2 1 – 8 R2 – 2 1 R3 ⇒ (x + 2)3 – 8(x + 2)2 + 21(x + 2) − 1 = 0 ⇒ y3 – 8y2 + 21y – 17 = 0. Esta é a equação transformada. Transformação recíproca É a transformação em que y = 1 x , x 0. Para obter a equação transformada basta substituir na equação primitiva x = 1 y . y = 1 x x = 1 y 9 Exemplo: Obter a equação cujas raízes são os inversos das raízes da equação 5x3 + x2 - x + 1 = 0. Solução: A relação de transformação é y = 1 x . Logo, x = 1 y . Substituindo na equação primitiva: 5 1 1 1 1 0 3 2 y y y       +       −      + = .Multiplicandocadaladodaigualdade por y3 , obtém-se ⇒ − + + = y y y 3 2 5 0, que é a equação transformada. Transformações • Equações recíprocas Equações algébricas: transformações e equações recíprocas PG19LP424SDM0_MIOLO_PVE19_4_MAT_LP.indb 177 27/05/2019 16:48:51
  • 20. Matemática 178 PVE19_7_MAT_A_28 Equações recíprocas Uma equação polinomial P(x) = 0 é chamada recíproca se, e somente se, é equivalente à transformada recíproca P 1 x = 0. Dada a equação recíproca P(x) = 0, se r é uma raiz de multiplicidade m, então, 1 r também é raiz com a mesma multiplicidade. Uma equação polinomial P(x) = 0 é recíproca se, e somente se, os coeficientes equidistantes dos extremos são iguais 2 a 2 ou opostos 2 a 2. Classificação Equações recíprocas de 1.ª espécie São aquelas em que os coeficientes equidistantes dos ex- tremos são iguais. Equações recíprocas de 2.ª espécie Aquelas em que os coeficientes equidistantes dos extre- mos são simétricos. Forma normal Diz-se que uma equação recíproca está na forma normal se ela é de 1.ª espécie e grau par. Observação Se uma equação é recíproca de 2.ª espécie e grau par, então ela não possui termo central. Propriedades ● Toda equação P(x) = 0, recíproca de 2.ª espécie e grau ímpar, admite a raiz 1. A divisão de P(x) por (x -1) con- duz a uma equação recíproca de 1.ª espécie e grau par. ● Toda equação P(x) = 0, recíproca de 2.ª espécie e grau par, admite raízes 1 e -1. A divisão de P(x) por (x -1) e (x +1) conduz a uma equação recíproca de 1.ª espécie e grau par. ● Toda equação P(x) = 0, recíproca de 1.ª espécie e grau ímpar, admite a raiz -1. A divisão de P(x) por (x +1) conduz a uma equação recíproca de 1.ª espécie e grau par. Resolução da equação recíproca normal Sendo a equação recíproca normal P(x) = A0x2k + A1x2k–1 +...+ A1x + A0 = 0 Dividindo a equação por xk , tem-se A x x A x x A x x A k k k k k k 0 1 1 1 1 1 1 1 0 +       +       + + +       + = − − − ... Fazendo y = x + 1 x e usando a identidade x x y x x x x p p p p p p + + − − + = ⋅ +       − +       1 1 1 1 1 1 1 (em que p = 1, 2, 3, ...), temos: x0 + 1 x0 = 2 x1 + 1 x1 = y x2 + 1 x2 = y2 - 2 x3 + 1 x3 = y3 - 3y ... Substituindo as expressões obtidas, tem-se uma equa- ção em y de grau k. Resolvendo a equação em y, podem- -se obter os valores de x. 9 Exemplo: Resolva a equação x4 − 4x3 + 5x2 − 4x + 1 = 0. Solução: Observando os coeficientes, verificamos que se trata de uma equa- ção recíproca de 1.ª espécie e grau par, ou seja, na forma normal. Dividindo a equação por x2 : x x x x x x x x 2 2 2 2 4 5 4 1 0 1 4 1 5 0 − + − + = ⇒ +      − +      + = Fazendo y = x + 1 x x2 + 1 x2 = y2 – 2 ⇒ (y2 −2) − 4 + = 0 ⇒ y2 − 4 + 3 = 0 ⇒ y = 1 ou y = 3 i) x x x x x + = ⇒ − + = ⇒ = ± 1 1 1 0 1 3 2 2 ii) x x x x x + = ⇒ − + = ⇒ = ± 1 3 3 1 0 3 5 2 2 Logo, S i i = + − + −           1 3 2 1 3 2 3 5 2 3 5 2 , , , . Resolvidos 1. (ITA) É dada a equação polinomial a c x b c x c a x a b + + ( ) + + + ( ) + − ( ) + + + ( )= 2 3 1 4 0 3 2 com a, b, c reais. Sabendo-se que esta equação é recíproca de primeira espécie e que 1 é uma raiz, então o produto abc é igual a a) –2. b) 4. c) 6. d) 9. e) 12. 9 Solução: E Sendo = ≠ a0 0, se a x ax a x a 0 3 1 2 2 3 0 + + + = é recíproca de primeira espécie, tem-se a a 0 3 = e a a 1 2 = . Sabendo que 1 é raiz da equação: a c a b b c c a a c a b b c c a c b a + + = + + + + = − + + + + + + + + + − =      ⇒ − = + 2 4 3 1 2 4 3 1 0 2 b b c a b c + = − + + = −      2 1 2 5 7 PG19LP424SDM0_MIOLO_PVE19_4_MAT_LP.indb 178 27/05/2019 16:49:08
  • 21. Frente A | Livro 179 7 PVE19_7_MAT_A_28 ⇒ + + = − − = − − = ⇒ + + = − − = − =           ⇒ = a b c c b b c a b c c b c a 2 5 7 2 3 6 2 5 7 2 4 4 4 b b c = − = −      3 1 Logo, o produto abc é 4 3 1 12 ⋅ − ⋅ − = ( ) ( ) . 2. (ITA) Determine a e b para que a equação 6x4 - ax3 + 62x2 - 35x+ + b - a = 0 seja recíproca de primeira classe e resolva-a. 9 Solução: ● Recíproca de 1.ª classe ⇒ b – a = 6 –a = –35 ⇒ a = 35 b = 41 Então: 6x4 − 3 x3 + 62 x2 − 3 x + 6 = 0 ( x2 ) ⇒ 6x2 – 35x + 62 – 35 x + 6 x2 = 0 6 x2 + 1 x2 – 35 x2 + 1 x + 62 = 0 Fazendo y = x + 1 x x2 + 1 x2 = y2 – 2 ⇒ 6(y2 − 2) − 3 + 62 = 0 ⇒ 6y2 − 3 + 0 = 0 ⇒ y = 10 3 ou y = 5 2 ● Se y = 5 2 : x + 1 x = 5 2 ⇔ 2x2 − 5x + 2 = 0 ⇔ x = 2 ou x = 1 2 ● Se y = 10 3 : x + 1 x = 10 3 ⇔ 3x2 −10x + 3 = 0 ⇔ x = 3 ou x = 1 3 S = 1 3 , 1 2 , 2, 3 Portanto, a = 35, b = 41 e S = 1 3 , 1 2 , 2, 3 Praticando 1. Qual é a equação cujas raízes são k unidades ( k  ) menores que as raízes da equação x³ = 4? a) y³ – (k + 4) = 0 b) y³ – (k – 4) = 0 c) y k − − ( )= 4 0 3 d) y k − − ( )= 4 0 3 e) y³ + 3ky (y + k) + k³ – 4 = 0 2. (UFF) Resolva a equação 2x6 - 5x5 + 2x4 - 2x2 + 5x - 2 = 0. Desenvolvendo Habilidades 1. C5:H21 (Unioeste-2013) Suponha que P(x) é um polinômio com coeficientes reais de modo que P(x) tem exatamente 3 raízes e o coeficiente do termo de maior grau é igual a 1. Considere que o número real –1 e o número complexo a + bi são duas raízes de P(x). Com relação ao polinômio P(x), pode-se afirmar que a) se a ≥ 1 2 , então todos os coeficientes são positivos. b) se a = 0, então todos os coeficientes são positivos. c) o coeficiente do termo quadrático é sempre nulo. d) o termo independente é sempre um número negativo. e) o coeficiente do termo linear é sempre menor que o termo independente. 2. C5:H21(Fatec)Sabe-seque-1éraizdupladopolinômioP(x)=2x4 +x3 - - 3x2 - x + 1. As outras raízes são números: a) imaginários puros. b) reais negativos. c) irracionais. d) racionais. e) pares. 3. C5:H21 (ITA-2014) Considere o polinômio complexo p(z) = z4 + az³ + +5z² – iz – 6, em que a é uma constante complexa. Sabendo que 2i é uma das raízes de p(z) = 0, as outras três raízes são a) –3i, –1, 1. b) –i, i, 1. c) –i, i, –1. d) –2i, –1, 1. e) –2i, –i, i. PG19LP424SDM0_MIOLO_PVE19_4_MAT_LP.indb 179 27/05/2019 16:49:23
  • 22. Matemática 180 PVE19_7_MAT_A_28 4. C5:H21 (ITA-2012) Considere um polinômio p(x), de grau 5, com coeficientes reais. Sabe-se que –2i e i- 3 são duas de suas raízes. Sabe-se, ainda, que dividindo-se p(x) pelo polinômio q(x) = x – 5 obtém-se resto zero e que p(1) = 20(5 + 2 3). Então, p(–1) é igual a a) 5(5 – 2 3). b) 15(5 – 2 3). c) 30(5 – 2 3). d) 45(5 – 2 3). e) 50(5 – 2 3). Complementares 1. (IFG-2012)RenéDescartes(1596-1650)éconsideradoopaidaFiloso- fiaModernaetambémumdosfundadoresdaMatemáticaModerna. Sua principal obra é Discurso do Método, publicado em 1637, que continha três apêndices, a saber: “A Dióptrica”, “A Geometria” e “Os Meteoros”.NaparteIIIde“AGeometria”,Descartesapresentadiversas propriedades sobre polinômios. As afirmativas de I a IV são adapta- ções das propriedades encontradas em“A Geometria”. Analise-as. I. Para diminuir a maior potência de uma equação polinomial, conhecendo-se uma de suas raízes, a, basta dividir a equação pelo binômio x – a, em que x é a variável. II. Para saber se o valor a é a raiz de uma equação, divida o poli- nômio pelo binômio x – a. Se a divisão for exata, então o valor a é uma raiz. III. Para aumentar o valor das raízes de um polinômio p(x) em 2 unidades basta fazer a substituição da variável x por x – 2. IV. Para multiplicar (ou dividir) as raízes de um polinômio p x a x a x a n n n-1 n-1 ( ) = + + … + 0 por um número real k, basta multiplicar (ou dividir) o an–1 por k, an–2 por k2 , e assim sucessi- vamente. É correto afirmar que: a) apenas a afirmativa I é correta. b) apenas as afirmativas I e II são corretas. c) apenas as afirmativas I, II e III são corretas. d) todas as afirmativas são corretas. e) nenhuma afirmativa é correta. 2. (CEFET-MG-2014) A equação x a x x a x a a 8 8 6 4 2 2 4 6 5 − − − − = , para x ≠ a, possui I. duas raízes reais para a = 0. II. somente raízes imaginárias se a ≠ 0. III. duas raízes reais e distintas para todo a . IV. duas raízes imaginárias para a = 5. São corretas apenas as afirmativas a) I e II. b) I e III. c) I e IV. d) II e III. e) II e IV. 3. (ITA) Multiplicando por 2 as raízes da equação x3 –2x2 +2x–1= 0 vamos obter raízes da seguinte equação: a) 2y3 - 6y2 + 6y - 4 = 0 b) y3 - 4y2 + 8y - 8 = 0 c) 8y3 - 8y2 + 4y - 1 = 0 d) y3 - 8y2 + 8y + 8 = 0 e) 4y3 - 4y2 - 4y - 8 = 0 4. (ITA) Considere as afirmações: I. A equação 3x4 - 10x3 + 10x - 3 = 0 só admite raízes reais. II. Toda equação recíproca admite um número par de raízes. III. As raízes da equação x3 + 4x2 - 4x - 16 = 0 são exatamente o dobro das raízes de x3 + 2x2 - x - 2 = 0. Então: a) Apenas I é verdadeira. b) Apenas II é falsa. c) Apenas III é verdadeira. d) Todas são verdadeiras. e) n.d.a. 5. (ITA) Para que 2x4 + bx3 - bx - 2 = 0 tenha quatro soluções reais distintas, devemos ter a) b um número real qualquer. b) b = 0. c) b > 0. d) b < -1. e) b > 4. 6. (UFF) Uma fábrica utiliza dois tanques para armazenar combustível. Os níveis de combustível, H1 e H2, em cada tanque, são dados pelas expressões: H1(t) = 150 t3 - 190 t + 30 e H2(t) = 50 t3 + 35 t + 30, sendo t o tempo em hora. Oníveldecombustíveldeumtanqueéigualaodooutronoinstante inicial (t = 0) e, também, no instante a) t = 0,5h. b) t = 1,0h. c) t = 1,5h. d) t = 2,0h. e) t = 2,5h. 7. (UERJ) As dimensões de um paralelepípedo retângulo são dadas pelas raízes do polinômio 3x3 -13x2 + 7x -1. Em relação a esse pa- ralelepípedo, determine: a) a razão entre a sua área total e o seu volume. b) suas dimensões. 8. (ITA) Sabendo-se que a equação de coeficientes reais x6 - (a + b + c)x5 + 6x4 + (a - 2b)x3 - 3cx2 + 6x - 1 = 0 é uma equação recíproca de segunda classe, então o número de raízes reais dessa equação é a) 0. b) 2. c) 3. d) 4. e) 6. 9. (ITA) A equação polinomial p(x) = 0 de coeficientes reais e grau 6 é recíproca de 2.ª espécie e admite i como raiz. Se p(2) = - 105 8 e p(-2) = 255 8 , então a soma de todas as raízes de p(x) é igual a a) 10. b) 8. c) 6. d) 2. e) 1. GABARITO ONLINE 1. Faça o download do aplicativo SAE Questões ou qualquer aplicativo de leitura QR Code. 2. Abra o aplicativo e aponte para o QR Code ao lado. 3. O gabarito deste módulo será exibido em sua tela. GABARITO E SOLUCIONÁRIO ONLINE 1. Faça o download do aplicativo SAE Questões ou qualquer aplicativo de leitura QR Code. 2. Abra o aplicativo e aponte para um dos QR Codes ao lado para acessar o gabarito ou o solucionário deste módulo. PG19LP424SDM0_MIOLO_PVE19_4_MAT_LP.indb 180 27/05/2019 16:49:31
  • 23. Frente B | Livro Matthew Cole/Shutterstock Frente B 181 PVE19_7_MAT_B_25 7 Arco metade • Transformação em produto Arco duplo, Arco metade e Transformação em produto Operações com arcos II Arco metade Vimos até agora que, conhecendo as relações trigono- métricas de um arco de medida a, podemos obter algumas relações para arcos de medidas 2a e 3a. Encontraremos, agora, as relações trigonométricas para os arcos de medida a 2 . ● a 2 cos cos a a 2 1 2 = ± + Demonstração Sabemos que cos cos cos cos cos cos cos 2 2 1 2 2 1 2 2 1 2 2 2 2 x x a a a a = − =       −       = + a a a a a 2 1 2 2 1 2       = +       = ± + cos cos cos . Fazendo uma mudança de variáveis e chamando 2x = a, temos x = a 2 . Assim, cos cos cos cos cos cos cos 2 2 1 2 2 1 2 2 1 2 2 2 2 x x a a a a = − =       −       = + a a a a a 2 1 2 2 1 2       = +       = ± + cos cos cos ● e a 2 sen a a 2 1 2 = ± − cos Demonstração Sabe ue 2 2 2 x – 1. Fazendo a mesma mu- dança de variáveis propostas no caso anterior: cos cos cos 2 1 1 2 2 2 2 1 2 2 2 2 x sen x a sen a sen a a sen a = − = −             = − 2 2 1 2 2 1 2       = −       = ± − cos cos a sen a a ● t a 2 tg a sen a a a a 2 2 2 1 1 = = ± − + cos cos cos 9 Exemplo: Dado cos 45° = 2 2 , calcule tg 22,5°. Solução: tg tg o o o o 22 5 1 45 1 45 1 2 2 1 2 2 2 2 2 2 2 2 22 5 2 2 2 2 4 , cos cos , = − + = − + = − + = − + = − − + − = − 4 2 2 4 2 3 2 2 Transformação em produto O objetivo é transformar uma soma algébrica de funções trigonométricas de arcos em um produto de funções trigono- métricas dos mesmos arcos. Vimos que: a b a ⋅ b e a ⋅ e b I a b a ⋅ b e a ⋅ e b II e a b e a ⋅ b e b ⋅ a III e a b e a ⋅ b e b ⋅ a I Observe algumas operações feitas com estas igualdades: I II a b a b 2 a ⋅ b I II a b a b 2 e a ⋅ e b III I e a b e a b 2 e a ⋅ b III I e a b e a b 2 e b ⋅ a Chamando a + b = p e a – b = q, temos: a p q e b p q = + = − 2 2 Substituindo, obtemos: cos cos cos cos cos cos p q p q p q p q sen p q + = +      ⋅ −       − = − +  2 2 2 2 2       ⋅ −       + = +       ⋅ −       sen p q senp senq sen p q p q 2 2 2 2 cos s senp senq sen p q p q − = −       ⋅ +       2 2 2 cos PG19LP424SDM0_MIOLO_PVE19_4_MAT_LP.indb 181 27/05/2019 16:50:58
  • 24. Matemática 182 PVE19_7_MAT_B_25 Resolvidos 1. (Mackenzie) Se y = 4cos 15° · cos 75°, então y2 vale: a) 1 b) 1 4 c) 1 2 d) 3 4 e) 2 9 Solução: A Temos y = 4cos(45° – 30°) · cos(45° + 30°) = =4(cos45°·cos30°+sen45°·sen30°)·(cos45°·cos30°–sen45°·sen30°). Então, y = ⋅ ⋅ + ⋅        ⋅ ⋅ − ⋅         = − ( )⋅ + ( )= 4 2 2 3 2 2 2 1 2 2 2 3 2 2 2 1 2 6 2 6 2 4 36 6 4 4 6 2 4 1 − = − =    ⋅ ⋅ − ⋅         = − ( )⋅ + ( )= 2 2 3 2 2 2 1 2 6 2 6 2 4 36 6 4 4 6 2 4 1 − = − = Logo, y2 = 12 = 1. 2. Calcule cos 22° 30’. 9 Solução: cos cos cos cos cos cos cos cos 2 2 2 1 1 2 2 1 2 2 2 2 α α α α α α α α = − ⇒ = − = + ⇒ = ± + sen 2 2 2 22 5 2 45 22 5 1 2 2 2 22 5 α α α Sendo tem se Logo = ° = ° °= + + ⇒ , , . , cos , cos , - ° °= + 2 2 2 3. Calcular y = sen2 24° – sen2 6° sabendo que sen18° = 5 1 4 - . 9 Solução: sen2 24° – sen2 6° = (sen 24° + sen 6°)(sen 24° – sen 6°) = = 2 30 2 18 2 2 18° 2 30 2 ⋅ ⋅ ⋅ ⋅ sen sen ° cos ° cos ° = = 2 · sen 15° · cos 9° · 2 · sen 9° · cos 15° = = 2 · sen 15° · cos 15° · 2 · sen 9° · cos 9° = = sen 30° · sen 18° = 1 2 5 1 4 5 1 8 ⋅ − = − Praticando 1. (UNITAU) Se sen(a–30°) = m, então cos(60° + a) é igual a: a) 2m. b) 1m. c) –1m. d) –2m. e) 3m. 2. (UFV) Sabendo que sen 30° = 1 2 , o valor de sen 15° é a) 3 2 2 − ( ) b) 1 4 c) 1 d) 2 3 2 − ( ) e) 1 2 3. (UFF) O valor de (sen 22,5° + cos 22,5°)2 é: a) 1 2 2 - b) 1 2 2 + c) 2 2 2 + d) 2 2 2 - Desenvolvendo Habilidades 1. C5:H21 (Mackenzie-2013) A expressão cos(a2 – b2 ) · cos(b2 ) – sen(a2 – 2b2 ) · sen(b2 ) é igual a a) cos a2 + b2 b) sen b2 c) cos a2 d) sen a + b · a – b e) cos a + b · a – b PG19LP424SDM0_MIOLO_PVE19_4_MAT_LP.indb 182 27/05/2019 16:51:15
  • 25. Frente B | Livro 183 7 PVE19_7_MAT_B_25 2. C5:H21(IFSP-2011)Sabendoque cos θ θ − = sen 6 3 ,entãoovalorde sen(2 ) é: a) –1 b) - 5 9 c) 1 6 d) 1 3 e) 5 6 3. C5:H21(Fatec-2010)Datrigonometriasabe-sequequaisquerquese- jamosnúmerosreaispeq,senp senq sen p q p q + = ⋅ +      ⋅ −       2 2 2 cos . Logo, a expressão cosx · sen9x é idêntica a a) sen10x + sen8x b) 2 · (sen6x + sen2x) c) 2 · (sen10x + sen8x) d) 1 2 6 2 ⋅ + ( ) sen x sen x e) 1 2 10 8 ⋅ + ( ) sen x sen x 4. C5:H21 (FGV-2012) O valor de y no sistema de equações = = sen y sen y sen 10 1 50° 50 50 1 10° - ° ° ° ° - + cos cos x se x n 10 é a) 4 3 3 b) 3 c) 3 3 d) 3 3 e) 3 4 5. C5:H21 (UFSM-2013) Para melhorar as condições de acessibilidade a uma clínica médica, foi construída uma rampa conforme indicado na figura. 16 m 15° c O comprimento horizontal c da rampa, em metros, pode ser ex- presso por a) 4 2 3 − ( ) b) 8 2 3 - c) 8 3 d) 4 2 3 + ( ) e) 8 2 3 + 6. C5:H21 (FGV-2013) Se sen x sen y + = 15 3 e cosx + cosy = 1, então, sec(x – y) é igual a a) 1 3 b) 1 2 c) 2 d) 3 e) 4 7. C5:H21 (EsPCEx/AMAN-2012) O cosseno do menor ângulo formado pelos ponteiros de um relógio às 14 horas e 30 minutos vale a) − + ( ) 3 1 2 b) − + ( ) 2 1 2 c) 1 2 4 + ( ) d) − − ( ) 6 2 4 e) 2 3 4 + ( ) 8. C5:H21 (PUC-Rio-2015) Sendo x um arco satisfazendo 2 < < x e sen x ( ) = 24 25 , o valor de cos x 2       é: a) 1 25 b) - 1 5 c) 1 5 d) - 3 5 e) 3 5 Complementares 1. (Ibmec) Considere que senx x a − = cos , com a > 0. Logo, sen2x é igual a: a) 1 – a. b) a – 1. c) a. d) a + 1. e) 2a. 2. (IME) O valor de y = sen70° cos50° + sen260° cos280° é a) 3 b) 3 2 c) 3 3 d) 3 4 e) 3 5 PG19LP424SDM0_MIOLO_PVE19_4_MAT_LP.indb 183 27/05/2019 16:51:33
  • 26. Matemática 184 PVE19_7_MAT_B_25 3. (ITA) Num triângulo ABC o lado AB mede 2cm, a altura relativa ao lado AB mede 1cm, o ângulo ABC mede 135° e M é o ponto médio de AB. Então a medida de BAC BMC + , em radianos, é igual a a) 1 5 b) 1 4 c) 1 3 d) 3 8 e) 2 5 4. (UFU) O valor de tg10° · (sec 5° + cossec 5°) · (cos 5° – sen 5°) é igual a a) 2 b) 1 2 c) 1 d) 2 5. (IME) Assinale a alternativa que apresenta o mesmo valor da expres- são [4cos2 (9°) – 3] [4cos2 (27°) –3]. a) sen(9°). b) tg(9°). c) cos(9°). d) sec(9°). e) cossec(9°). 6. (Unesp-2015) Sabendo-se que cos(2x) = cos2 x – sen2 x , para quais valores de x a função f(x) = cosx + · 1 2 cos(2x) assume seu valor mí- nimo no intervalo 0 < x < 2 ? 7. (ITA-2010) Se os números reais e , com + = 4 3 0 , , ma- ximizam a soma sen + sen , então é igual a a) 3 3 b) 2 3 c) 3 5 d) 5 8 e) 7 12 8. (UERJ) Um esqueitista treina em três rampas planas de mesmo comprimento a, mas com inclinações diferentes. As figuras a seguir representam as trajetórias retilíneas AB = CD = EF, contidas nas retas de maior declive de cada rampa. C A E D B 15º 45º h1 h2 h3 75º F a a a Sabendo que as alturas, em metros, dos pontos de partida A, C e E são, respectivamente, h1, h2, e h3 conclui-se que h1 + h2 é igual a: a) h3 3 b) h3 2 c) 2h3 d) h3 GABARITO ONLINE 1. Faça o download do aplicativo SAE Questões ou qualquer aplicativo de leitura QR Code. 2. Abra o aplicativo e aponte para o QR Code ao lado. 3. O gabarito deste módulo será exibido em sua tela. GABARITO E SOLUCIONÁRIO ONLINE 1. Faça o download do aplicativo SAE Questões ou qualquer aplicativo de leitura QR Code. 2. Abra o aplicativo e aponte para um dos QR Codes ao lado para acessar o gabarito ou o solucionário deste módulo. PG19LP424SDM0_MIOLO_PVE19_4_MAT_LP.indb 184 27/05/2019 16:51:42
  • 27. Frente B | Livro Africa Studio/Shutterstock Frente B 185 PVE19_7_MAT_B_26 7 Equações trigonométricas • Inequações trigonométricas Função tangente, equações e inequações trigonométricas Relações fundamentais, relações auxiliares e equações trigonométricas Função seno, equações e inequações trigonométricas Função cosseno, equações e inequações trigonométricas Equações e inequações trigonométricas Equações trigonométricas Para resolvermos equações trigonométricas é fundamen- tal que tenhamos em mente algumas relações: ● Quando a equação envolve a função seno: – sen sen k ou k k α β β α π β π α π = ⇔ = + = − ( )+ ∈      2 2 , ● Quando a equação envolve a função cosseno: cos cos ( ) , α β β α π β π α π = ⇔ = + = − + ∈      2 2 2 k ou k k 2 – ● Quando a equação envolve a função tangente: + t t ⇔ = + k , k 9 Exemplos: 1) sen x = sen 60º x = 60º + 360º · k ou x = 120º + 360º · k; k 2) sen x = sen 5 3 π ⋅ x = 5 3 π ⋅+2k ou x = – − 2 3 π + 2k ; k 3) sen x = 1 2 sen x = sen 30º x = 30º + 360º · k ou x = 150º + 360º · k; k 4) cos x = cos 45º x = 45º + 360º · k ou x = 315º + 360º · k; k 5) cos x = cos − 2 3 π x = − 2 3 π + 2k ou x = 4 3 π ⋅+ 2k ; k 6) cos x = –1 cos x = cos 180º x = 180º + 360º · k, k ) tg x = tg 30 x = 30º + 180º · k, k 8) tg x = 1 tg x = tg 13 x = 135º + 180º · k; k ) tg x = tg π 2 ⋅ omo tg π 2 ⋅n o existe, n o existe x que satisfa a igualdade. Inequações trigonométricas Nas inequações trigonométricas devemos achar o interva- lo que satisfaz às necessidades impostas. 9 Exemplos: Ac e as solu es das inequa es para x [0, 2 . 1) sen x ≥ 1 2 ⇒ sen x ≥ sen 30º sen 150º ⇒ S = [30º, 150º] Para qualquer valor de x a solu o dada por S k x k k = + ≤ ≤ + ∈       π π π π 6 2 5 6 2 , PG19LP424SDM0_MIOLO_PVE19_4_MAT_LP.indb 185 27/05/2019 16:52:26
  • 28. Matemática 186 PVE19_7_MAT_B_26 150º 30º 2) cos x < – 1 2 ⇒ cos x < cos 120º cos 240º ⇒ S = ]120º, 240º[ Para qualquer valor de x a solu o dada por S k x k k = + < < + ∈       2 3 2 4 3 2 π π π π , 120º 240º 3) tg x ≥ 1 ⇒ tg x ≥ tg 45º tg 225º ⇒ S =       ∪       π π π π 4 2 5 4 3 2 , , Para qualquer valor de x a solu o dada por S k x k k = + ≤ < + ∈       π π π π 4 2 , 1 Resolvidos 1. O número de raízes da equação sen x + cos x = 0 no intervalo [0, 2 ] é a) 0. b) 1. c) 2. d) 3. e) 4. 9 Solução: C Tem-se que sen x + cos x = 0 ⇒ sen x = –cos x. Dessa forma, sen x x cos = –1 ⇒ tg x = –1 ⇒ S = 3 4 7 4 , Logo, haverá duas soluções. Perceba que as soluções pertencem aos 1.º e 4.º quadrantes, em que a tangente é negativa. 2. Determine o conjunto solução da inequação sen x ⋅ cos x > 0, para x [0, 2 ]. 9 Solução: As soluções serão os quadrantes em que sen x e cos x possuam o mesmo sinal, ou seja, S = 0 2 3 2 , , 3. Ache o conjunto solução da inequação 2sen2 x < senx. 9 Solução: 2sen2 x – sen x < 0 sen x (2sen x –1) < 0 0 < sen x < 1 2 5 6 6 1 2 Logo, S x k x k = < < + /2 6 2 ou 5 6 2 2 + < < + k x k k , Praticando 1. (PUC-Rio) Os ângulos (em graus) entre 0° e 360° para os quais sen = cos são: a) 45° e 90°. b) 45° e 225°. c) 180° e 360°. d) 45º, 90º e 180°. e) 90º, 180º e 270°. 2. (Unicamp) Seja x real tal que cos x = tg x. O valor de sen x é a) 3 1 2 - b) 1 3 2 - c) 5 1 2 - d) 1 5 2 - PG19LP424SDM0_MIOLO_PVE19_4_MAT_LP.indb 186 27/05/2019 16:52:44
  • 29. Frente B | Livro 187 7 PVE19_7_MAT_B_26 3. Ache o conjunto solução da inequação cos2x + cosx –1. Desenvolvendo Habilidades 1. C5:H21 (PUC-RS-2016) Se x , então a equação cos(x) = cos(–x) apresenta o conjunto solução a)  b) [–1,1] c) [0,+ ) d) (– ,0] e) {–1, 0, 1} 2. C5:H20 (UESPI-2012) Quantas soluções a equação senx x = 10 admite noconjuntodosnúmerosreais?Aseguir,estãoesboçadososgráficos de senx e x 10 . a) 5 b) 6 c) 7 d) 8 e) 9 3. C5:H21 (PUC-Rio-2017) Sabendo que cos(3x) = –1, quais são os possíveis valores para cos(x)? a) 0 e 1 2 b) 3 2 1 2 e c) 1 2 e 1 d) –1 e 5 e) 0 e 3 2 4. C5:H21 (FGV-2012) No intervalo [0, 4 , a equação: sen3 x – 2sen2 x – 5senx + 6 = 0 tem raízes cuja soma é a) 2 b) –2 c) 6 d) 2 e) 3 5. C5:H21 (UECE-2014 – adap.) Se p e q são duas soluções da equação 2sen2 x – 3senx + 1 = 0 tais que senp senq, então, o valor da ex- pressão sen2 p – cos2 q é igual a a) 0. b) 0,25. c) 0,50. d) 1. e) 1,5. 6. C5:H21 (UCPel-2011) Sendo x [0, 2 ] e 2sen x – 3cos x = 0, então x vale a) 3 b) 2 3 c) 2 5 d) 3 4 e) 5 6 7. C5:H21 (ENaval-2012) A soma dos quadrados das raízes da equação |senx| = 1 –2sen2 x, quando 0 < x < 2 , vale a) 49 36 2 b) 49 9 2 c) 7 3 2 d) 14 9 2 e) 49 6 2 8. C5:H21 (CEFET-MG-2014) A solução da inequação 0 2 2 1 1 2 < + + < sen x sen x tg x para x 0 2 , é o conjunto a) 0 4 , b) 0 4 , c) 0 2 , d) 0 2 , e) 4 2 , PG19LP424SDM0_MIOLO_PVE19_4_MAT_LP.indb 187 27/05/2019 16:53:00
  • 30. Matemática 188 PVE19_7_MAT_B_26 9. C5:H21 (CEFET-MG-2013 – adap.) O conjunto formado pelas raízes dafunção f x x x ( )=      ⋅       cos cos 2 3 3 2 queestãocontidasnointervalo [0, é a) 3 , b) 3 4 , c) 3 4 4 3 , d) 3 3 4 , , e) π π π π 3 3 4 4 3 , , , { } 10. C5:H21 (Unesp-2014) O conjunto solução (S) para a inequação 2cos2 x + cos(2x) > 2, em que 0 < x < , é dado por: a) S x x ou x = < < < < 0 0 6 5 6 , / b) S x x = < < 0 3 2 3 , / c) S x x ou x = < < < < 0 0 3 2 3 , / d) S x x = < < 0 6 5 6 , / e) S x = 0, Complementares 1. (UFSJ-2012) Sendo x um arco tal que 0 x < 2 e 3 2 ⋅( )= tg x senx, é correto afirmar que a) a soma das soluções dessa equação é igual a . b) as extremidades de todos os arcos x que são solução dessa equação estão no terceiro quadrante. c) nesse intervalo, a equação tem dois arcos distintos como so- luções. d) para qualquer solução dessa equação, tgx = senx. 2. (PUC-Rio) Assinale o valor de para o qual sen2 = tg . a) 2 b) 3 c) 2 3 d) 4 3 e) 3 4 3. (UECE-2014–adap.)Usandoaexpressãoclássicadodesenvolvimen- to da potência (a + b)n , em que a e b são números reais e n é um número natural, pode-se resolver facilmente a equação sen4 x – 4sen3 x + 6sen2 x – 4sen x + 1 = 0. Então, para os valores de x encontrados, teremos cos x igual a a) 1 b) 3 2 c) 2 2 d) 0 4. (UFPE)Quantassoluçõesaequaçãotrigonométrica senx x = − 1 cos admite, no intervalo 0 80 , ? 5. (UCS) Suponha que, em determinado lugar, a temperatura média diária,T,em°C,possaserexpressa,emfunçãodotempot,emdiasde- corridos desde o início do ano, por T t sen t = + - 14 12 2 105 364 . Segundo esse modelo matemático, a temperatura média máxima nesse lugar ocorre no mês de a) julho. b) setembro. c) junho. d) dezembro. e) março. 6. (Mackenzie-2014) Em , o domínio da função f, definida por f x sen x senx ( )= 2 , é a) x x k k / , b) x k x k k < < + / , 2 2 c) x k x k k + + / , 2 2 3 2 2 d) x k x k k x k k < + + < + / , 2 2 2 3 2 2 2 2 e) x k x k k x k k + + < + / , 2 2 2 3 2 2 2 2 7. (UDESC-2012) A soma de todos os valores de x 0 2 , que satisfa- zem à equação cos2 (2x) – sen2 (x) = cos6 (x) é igual a a) . b) 2 . c) 5 . d) 3 . e) 4 . 8. (UECE-2010) O número de soluções da equação 3 3 0 2 2 sen x senx x − + = | | cos que estão no intervalo [0,2 ] é a) 2. b) 8. c) 4. d) 6. 9. (Fuvest) O triângulo AOB é isósceles, com OA OB = , e ABCD é um quadrado. Sendo a medida do ângulo AOB, pode-se garantir que área do quadrado é maior do que a área do triângulo se Dados os valores aproximados: tg14° 0,2493, tg15° 0,2679 tg20° 0,3640, tg28° 0,5317 a) 14° < < 28°. b) 15° < < 60°. c) 20° < < 90°. d) 25° < < 120°. e) 30° < < 150°. GABARITO ONLINE 1. Faça o download do aplicativo SAE Questões ou qualquer aplicativo de leitura QR Code. 2. Abra o aplicativo e aponte para o QR Code ao lado. 3. O gabarito deste módulo será exibido em sua tela. GABARITO E SOLUCIONÁRIO ONLINE 1. Faça o download do aplicativo SAE Questões ou qualquer aplicativo de leitura QR Code. 2. Abra o aplicativo e aponte para um dos QR Codes ao lado para acessar o gabarito ou o solucionário deste módulo. PG19LP424SDM0_MIOLO_PVE19_4_MAT_LP.indb 188 27/05/2019 16:53:10
  • 31. Frente B | Livro retrorocket/Shutterstock Frente B 189 PVE19_7_MAT_B_27 7 Definições iniciais • Juros simples Juros simples Juros simples Definições iniciais Podemos definir os juros como o rendimento de uma apli- cação financeira ou o valor referente ao atraso no pagamento de uma dívida, por exemplo. Em outras palavras, os juros estão diretamente ligados a valores financeiros. Dessa forma, antes de falarmos sobre os juros propriamente ditos, vamos saber um pouco mais sobre aumentos e descontos de mercadorias. Aumentos e descontos O preço inicial de um produto, cujo valor era Pi, foi aumen- tado em x%. Esse produto tem o preço final (Pf) expresso por Pf = Pi + Pi ⋅ x% = Pi ⋅ (1 + x%) Sendo um preço inicial Pi que sofreu um desconto de x%, o novo preço é: Pf = Pi – Pi ⋅ x% = Pi ⋅ (1 – x%) 9 Exemplos: 1) Um produto custava R$80,00 e sofreu um aumento de 30%. Qual o novo preço? Solução: 80,00 ⋅ (1 + 0,3) = R$104,00. 2) Um produto que custava R$80,00 foi vendido com um desconto de 30%. Qual o preço de venda? Solução: 80,00 ⋅ (1 - 0,3) = R$56,00. 3) Um produto que custava R$80,00 passou a custar R$110,00. Qual o percentual de aumento? Solução: 80 ⋅ (1 + x) = 110 ⇒ x = 0,375 = 37,5%. 4) Um produto que custava R$80,00 foi vendido com desconto por R$70,00. Qual o percentual do desconto? Solução: 80 ⋅ (1 - x) = 70 ⇒ x = 0,125 = 12,5%. 5) Uma mercadoria sofreu dois aumentos sucessivos de 10%. Qual o aumento resultante? Solução: Observe que, nesse caso, os acréscimos são sucessivos. Con- siderando Pf o preço final, Pi o preço inicial e x1, x2, x3,..., xn as taxas de acréscimos sucessivos em decimal, temos: Pf = Pi (1 + x1) ⋅ (1 + x2) + ... + (1 + xn) Pf = Pi ⋅ (1 + 0,1) ⋅ (1 + 0,1) = 1,21Pi Aumento: Pf -Pi = 1,21Pi - Pi = 0,21Pi = 21%. 6) Uma mercadoria sofreu um aumento de 20% e posteriormente um novo desconto de 20%. Qual o desconto final? Solução: Pf = Pi ⋅ (1 + 0,2) ⋅ (1 - 0,2) = 0,96Pi Desconto: Pf - Pi = 0,96Pi - Pi = -0,04Pi = 4%. Operações sobre mercadorias São operações que envolvem a compra e venda de mercadorias e o lucro ou prejuízo oriundos de tais operações. Vendas com lucro: o preço de venda (V) é obtido pelo pre- ço de custo (C) mais o lucro (L). V = C + L Vendas com prejuízo: o preço de venda é obtido pelo pre- ço de custo menos o prejuízo. V = C - P O lucro ou o prejuízo são comumente calculados com base no preço de custo, mas podem também ser calculados sobre o preço de venda. Observe o quadro a seguir, sendo i a taxa percentual. Cálculo sobre o preço de custo Cálculo sobre o preço de venda Com lucro V = C + i ⋅ C V = C + i ⋅ V Com prejuízo V = C - i ⋅ C V = C - i ⋅ V 9 Exemplos: 1) Uma mercadoria custou R$80,00. Por quanto deve ser vendida para que haja um lucro de 10% sobre o preço de custo? Solução: V = C + i · C ⇒ V = C + 0,10 · C ⇒ V = 1,1 · C ⇒ V = 1,1 · 80 = R$88,00. 2) Uma mercadoria custou R$80,00. Por quanto deve ser vendida para que haja um lucro de 10% sobre o preço de venda? Solução: V = C + i · V ⇒ V = C + 0,10 · V ⇒ V – 0,10 · V = C ⇒ 0,9 V = 80 ⇒ V = R$88,89. 3) Uma mercadoria foi vendida por R$180,00 com um prejuízo de 10% sobre o preço de venda. Qual o preço de custo dessa mer- cadoria? Solução: V = C – i · V ⇒ C = V + iV ⇒C = V(1 + i) ⇒ C = 180 · (1 + 0,10) ⇒ C = 180 · 1,10 ⇒ C = R$198,00. 4) Uma calça foi vendida por R$120,00 com um lucro de 20% sobre o preço de custo. Qual o preço de custo da calça? Solução: V = C + i · c ⇒ V = C(1 + i) ⇒ 120 = C(1 + 0,20) ⇒ 120 = 1,2 C ⇒ C = R$100,00. PG19LP424SDM0_MIOLO_PVE19_4_MAT_LP.indb 189 27/05/2019 16:53:36
  • 32. Matemática 190 PVE19_7_MAT_B_27 Juros simples O regime de capitalização simples é aquele em que os juros gerados em cada período são iguais e sobre eles não incidem novos juros, ou seja, os juros não são capitalizados. Juros simples é a remuneração recebida pela aplicação de um capital C a uma taxa de juros de i% durante certo tem- po t, cuja remuneração é calculada somente sobre o capital inicial C. Temos: J = C ⋅ i ⋅ t Em que: ● C é o capital inicial aplicado (principal); ● i é a taxa percentual de juros; ● t é o tempo de aplicação; ● J são os juros recebidos. É fundamental que, na fórmula acima, o tempo t seja ex- presso na mesma unidade a que estiver referenciada a taxa de juros i. Dessa forma, se a taxa de juros for ao ano, o tempo deve ser expresso em anos; já se a taxa de juros for ao mês, o tempo deverá estar em meses. Chama-se montante (M) o valor resgatado ao final da aplicação do capital C. M = C + J ⇒ M = C (1 + i · t) No regime de capitalização a juros simples os acréscimos ao capital em cada período são iguais, ou seja, o montante cresce segundo uma progressão aritmética, o que pode ser confirmado pela característica da expressão acima, que é uma função do 1.º grau em t. Vale citar que, para o cálculo de juros, normalmente é usa- do o ano comercial de 360 dias, no qual os meses são sempre considerados com 30 dias. 9 Exemplos: 1) SeR$3.000,00foramaplicadospor5mesesàtaxadejurossimples de 4% ao mês, determine os juros recebidos e o montante. Solução: = · i · t = 3 000 · 0,04 · = R 600,00 = + = 3 000 + 600 = R 3.600,00 2) Um capital de R$2.000,00 foi aplicado por 7 meses a uma taxa anualdejurossimplesde24%.Qualomontantedessaaplicação? Solução: Como o tempo está em meses e a taxa de juros ao ano, vamos determinar a taxa de juros mensal: 24 24 12 2 % . . % . . % . . a a a m a m = = M = C ⋅ (1 + i ⋅ t) M = 2 000 ⋅ (1 + 0,02 ⋅ 7) = 2 000 ⋅ 1,14 = R$2.280,00 3) Um capital de R$5.000,00 foi aplicado por 20 dias a juros simples a 9% ao mês. Qual o montante da aplicação? Solução: 20 dias = 20 30 mês = 2 3 mês i = 9% a.m. M = C · (1 + i · t) M = ⋅ + ⋅       = ⋅ 5000 1 0 09 2 3 5000 1 06 , , M = R$5.300,00 4) O capital de R$500,00 aplicado durante um ano e meio a juros simples rendeu R$180,00. Qual a taxa mensal? Solução: t = 1,5 ano = 18 meses J C i t i J C t i i i a m = ⋅ ⋅ ⇒ = ⋅ = ⋅ ⇒ = = 180 500 18 0 02 2 , % . . 5) A aplicação de R$3.000,00 a juros simples de 6% ao mês gerou montante igual a R$3.420,00. Determine o prazo da aplicação. Solução: M = C + J ⇒ J = M - C J = 3 420 - 3 000 = R$420,00 j C i t t J C i t t t meses t dias = ⋅ ⋅ ⇒ = ⋅ = ⋅ ⇒ = ⇒ = = 420 3 000 0 06 420 180 7 3 70 , Resolvidos 1. Em relação ao regime de capitalização de juros simples, resolva: a) UminvestidoraplicouR$50.000,00aumataxadejurosde6%ao ano por 7 anos. Imagine que o regime de capitalização é simples e determine o montante dessa aplicação. b) Uma pessoa realizou um empréstimo de R$220,00 para pagar após três meses, a taxa de 12% a.t. Determine quanto essa pessoa pagou de juros, sabendo que o regime de capitalização utilizado foi o simples. 9 Solução: a) M = montante Logo, M = 50000 · (1 + 0,06 · 7) = R$71.000,00 b) t meses i C J C i t R = = =      ⇒ = ⋅ ⋅ = ⋅ ⋅ = 3 0 04 220 220 0 04 3 26 40 , , $ , 2. (FGV)OSr.OliveiraaplicouR$20.000,00numacadernetadepoupan- ça e R$30.000,00 num fundo de ações por 1 ano. Neste período, a caderneta de poupança rendeu 8% e o fundo de ações apenas 2%. a) Qual a taxa de rendimento global do Sr. Oliveira no período? b) Quanto ele deveria ter aplicado no fundo de ações (mantida a aplicação de R$20.000,00 na caderneta de poupança) para que sua taxa global fosse de 6% ao ano? 9 Solução: a) Calculando os juros que a caderneta de poupança e o fundo de ações renderam, temos: • Caderneta de poupança: R$20.000,00 ⋅ 0,08 = R$1.600,00 • Fundo de ações: R$30.000,00 ⋅ 0,02 = R$600,00 A taxa de rendimento global será: i = + + = = = 1600 600 20000 30000 2200 50000 0 044 4 4 , , % PG19LP424SDM0_MIOLO_PVE19_4_MAT_LP.indb 190 27/05/2019 16:53:55
  • 33. Frente B | Livro 191 7 PVE19_7_MAT_B_27 b) 8 20000 2 20000 6 1600 0 02 20000 0 06 % % x % , , ⋅ + ⋅ + = + + = x x x 1200 + 0,06x = 1600 + 0,02x 0,04x = 400 x = 10 000 3. (FGV)Benedito,ummotoristadetáxiquepercorre5040kmpormês, analisa a hipótese de adquirir um veículo equipado com tecnologia flex fuel, bicombustível. No folheto de propaganda a montadora explica que o veículo bicombustível tanto pode usar álcool como gasolina, em qualquer proporção, apresentando a seguinte tabela de consumo, de acordo com as proporções de combustíveis utilizadas: Combustível Consumo (km por litro) Álcool Gasolina – 100% 18 40% 60% 16 60% 40% 15 70% 30% 14 100% – 10 a) Considerando que atualmente a gasolina custa R$2,00 por litro e que o preço do litro de álcool é 45% do preço do litro de ga- solina, que proporção de combustíveis Benedito deveria utilizar no veículo equipado com tecnologia flex fuel, para que tivesse o menor gasto mensal possível? b) Para comprar o carro bicombustível, Benedito despenderá R$3.000,00 a mais do que gastaria se adquirisse o mesmo modelo com motor movido a gasolina, que faz 18 km por litro. Nas duas hipóteses, o seu carro atual entrará como parte do pagamento. O nosso motorista está em dúvida, pois se comprar o carro a gasolina poderá aplicar os R$3.000,00 em um fundo de investimento que garante um rendimento de 30% de juros no período de 3 anos. Supondo que os preços dos combustíveis mantenham-se nos níveis atuais nos próximos 3 anos, qual a aquisição que proporcionará maior ganho a Benedito? 9 Solução: a) Supondo que Benedito possa usar apenas as opções apresenta- das na tabela do enunciado, podemos montar a tabela a seguir para estimar o gasto em cada uma das situações. Preço por litro (R$) Consumo mensal (L) Gasto mensal (R$) 0 · 0,90 + 1 · 2,00 = 2,00 040 : 18 = 280 2,00 · 280 = 560,00 0,4 · 0,90 + 0,6 · 2,00 =1,56 040 : 16 = 31 1,56 · 315 = 491,40 0,6 · 0,90 + 0,4 · 2,00 =1,34 040 : 1 = 336 1,34 · 336 = 450,24 0,7 · 0,90 + 0,3 · 2,00 =1,23 040 : 14 = 360 1,23 · 360 = 442,80 1 · 0,90 + 0 · 2,00 = 0,90 040 : 10 = 04 0,90 · 504 = 453,60 A análise dos resultados mostra que seu gasto mensal será o me- norpossívelcomaproporçãode70%deálcoole30%degasolina. b) Nas condições apresentadas, se Benedito adquirir um veículo com tecnologia flex fuel, ele poderá economizar, no máximo, 560,00–442,80=117,20reaispormêsnogastocomcombustível. Isso corresponde a 117,20 ⋅ 36 = 4.219,20 reais em 3 anos. Descontando R$3.000,00, que gastaria a mais na aquisição, ele economizaria R$1.219,20. Nesse mesmo período, a aplicação de R$3.000,00 renderia R$900,00 (30% de R$3.000,00). Logo, a me- lhor aquisição é a do veículo flex fuel. Praticando 1. UmcapitaldeR$540,00éaplicadoporumanoaumataxade5%a.m. Imaginando o regime de capitalização simples, determine o valor dos juros obtidos. 2. Determine o valor resultante de uma aplicação de R$180,00, por 8 meses, sendo a taxa de juros simples igual a 72% a.s. 3. (FGV – adap.) Fábio recebeu um empréstimo bancário de R$10.000,00 paraserpagoemduasparcelasanuais,aserempagasrespectivamente nofinaldoprimeiroedosegundoano,sendocobradosjurossimplesà taxa de 20% ao ano. Sabendo que o valor da 1.ª parcela foi R$4.000,00, podemos concluir que o valor da 2.ª foi de a) R$8.800,00. b) R$8.000,00. c) R$9.600,00. d) R$6.000,00. e) R$10.000,00. PG19LP424SDM0_MIOLO_PVE19_4_MAT_LP.indb 191 27/05/2019 16:54:09
  • 34. Matemática 192 PVE19_7_MAT_B_27 Desenvolvendo Habilidades 1. C5:H22 (FMP-2016) A seguir são apresentados termos gerais que definem cinco sequências de números reais, para n . an = 80 ∙ (24)n bn = 80 ∙ (1,30)n cn = 80 ∙ (0,3)n dn = 80 + 24n en = 80 + (2,4)n Um dos termos gerais apresentados indica o valor devido n meses após a tomada de um empréstimo de R$80,00, calculado após a incidência de uma taxa mensal de juros simples de 30% sobre o valor do empréstimo. Esse termo geral é a) an b) bn c) cn d) dn e) en 2. C5:H21 (UERJ-2015 – adap.) Considere uma mercadoria que teve seu preço elevado de x reais para y reais. Para saber o percentual de aumento, um cliente dividiu y por x obtendo quociente igual a 2,08 e resto igual a zero. Em relação ao valor de x, o aumento percentual é equivalente a a) 10,8%. b) 20,8%. c) 108,0%. d) 208,0%. e) 280,0%. 3. C5:H21 (Enem-2013) O contribuinte que vende mais de R$20mil de ações em Bolsa de Valores em um mês de- verá pagar Imposto de Renda. O pagamento para a Re- ceita Federal consistirá em 15% do lucro obtido com a venda das ações. (Disponível em: <www.folha.uol.com.br> Acesso em. 26 abr. 2010. Adaptado). Um contribuinte que vende por R$34 mil um lote de ações que custou R$26 mil terá de pagar de Imposto de Renda à Receita Federal o valor de a) R$ 900,00. b) R$ 1.200,00. c) R$ 2.100,00. d) R$ 3.900,00. e) R$ 5.100,00. 4. C5:H22 (Enem-2010) Uma empresa possui um sistema de controle de qualidade que classifica o seu desempenho financeiro anual, tendo como base o do ano anterior. Os conceitossão:insuficiente,quandoocrescimentoémenor que 1%; regular, quando o crescimento é maior ou igual a 1% e menor que 5%; bom, quando o crescimento é maior ou igual a 5% e menor que 10%; ótimo, quando é maior ou igual a 10% e menor que 20%; e excelente, quando é maior ou igual a 20%. Essa empresa apresentou lucro de R$132.000,00 em 2008 e de R$145.000,00 em 2009. De acordo com esse sistema de controle de qualidade, o desempe- nho financeiro dessa empresa no ano de 2009 deve ser considerado a) insuficiente. b) regular. c) bom. d) ótimo. e) excelente. 5. C5:H21 (IFSC-2017) Analise as seguintes situações: 1. Seu João fez um empréstimo de R$1.000,00 no Banco A, a uma taxa de juros simples; após 4 meses, pagou um montante de R$1.320,00 e quitou sua dívida. 2. Dona Maria fez um empréstimo de R$1.200,00 no Banco B, a uma taxa de juros simples; após 5 meses, pagou um montante de R$1.800,00 e quitou a dívida. Assinale a alternativa correta. A taxa mensal de juros simples cobrada pelo Banco A e pelo Banco B, respectivamente, é: a) 8% a.m. e 10% a.m. b) 18% a.m. e 13% a.m. c) 6,4% a.m. e 12,5% a.m. d) 13% a.m. e 18% a.m. e) 10% a.m. e 8% a.m. 6. C5:H21 (UFSM-2015) A chegada da televisão no Brasil facilitou o acesso à informação. Com o avanço da tecnologia, os aparelhos estão cada dia mais modernos e consequentemente mais caros. Um consumidor deseja adquirir uma televisão com tecnologia de última geração. Enquanto aguarda o preço da televisão baixar, ele aplica o capital disponível de R$3.000,00 a juros simples de 0,8% ao mês em uma instituição financeira, por um período de 18 meses. O montante, ao final desse período, é igual a a) R$7.320,00. b) R$5.400,00. c) R$4.320,00. d) R$3.432,00. e) R$3.240,00. 7. C5:H21(UEPA-2015)Umagricultorfinancioujuntoaumacooperativa os insumos utilizados na lavoura em 2014. Pagou 20% do valor dos insumos no ato da compra, utilizando parte do lucro obtido no ano anterior, e financiou o restante em 10 meses a uma taxa de 2% ao mês a juros simples. Observou que havia gastado o montante de R$208.800,00 com a parte financiada. Neste caso, o valor financiado dos insumos pelo agricultor foi de a) R$217.500,00. b) R$174.000,00. c) R$164.000,00. d) R$144.500,00. e) R$136.000,00. 8. C5:H21 (Enem-2011) Uma pessoa aplicou certa quantia em ações. No primeiro mês, ela perdeu 30% do total do investimento e, no segundo mês, recuperou 20% do que havia perdido. Depois desses dois meses, resolveu tirar o montante de R$3.800,00 gerado pela aplicação. A quantia inicial que essa pessoa aplicou em ações corresponde ao valor de a) R$4.222,22. b) R$4.523,80. c) R$5.000,00. d) R$13.300,00. e) R$17.100,00. 9. C5:H21 (Enem-2015)Umfornecedorvendiacaixasdeleite a um supermercado por R$1,50 a unidade. O supermerca- docostumavacomprar3000caixasdeleitepormêsdesse fornecedor.Umaforteseca,ocorridanaregiãoondeoleite éproduzido,forçouofornecedoraencareceropreçodevendaem40%. Osupermercadodecidiuentãocortarem20%acompramensaldessas caixas de leite. Após essas mudanças, o fornecedor verificou que sua receita nas vendas ao supermercado tinha aumentado. O aumento da receita nas vendas do fornecedor, em reais, foi de a) 540. b) 600. c) 900. d) 1260. e) 1500. 10. C6:H25 (Enem-2011) Um jovem investidor precisa esco- lher qual  investimento lhe trará maior retorno financei- ro em uma aplicação de R$500,00. Para isso, pesquisa o rendimento e o imposto a ser pago em dois investimen- tos: poupança  e CDB (certificado de depósito bancário). As infor- mações obtidas estão resumidas no quadro: Rendimento Mensal (%) IR (Imposto de Renda) Poupança 0,560 Isento CDB 0,876 4% (sobre o ganho) Paraojoveminvestidor,aofinaldeummês,aaplicaçãomaisvantajosaé a) a poupança, pois totalizará um montante de R$502,80. b) a poupança, pois totalizará um montante de R$500,56. c) o CDB, pois totalizará um montante de R$504,38. d) o CDB, pois totalizará um montante de R$504,21. e) o CDB, pois totalizará um montante de R$500,87. PG19LP424SDM0_MIOLO_PVE19_4_MAT_LP.indb 192 27/05/2019 16:54:10